Você está na página 1de 142

NOTAS DE AULA TEORIA DOS NMEROS

(Verso 01/06/2009)

Por Benedito Tadeu Vasconcelos Freire

SUMRIO

AULA
Aula 01 Aula 02 Aula 03 Aula 04 Aula 05

TTULO
Noes sobre o processo e o mtodo de induo Divisibilidade O algoritmo da diviso O teorema fundamental da aritmtica O mximo divisor comum, o mnimo mltiplo comum e as equaes diofantinas lineares

PGINA
03 18 27 35

46

Aula 06 Aula 07 Aula 08 Aula 09 Aula 10 Aula 11

Representao dos nmeros naturais e critrios de divisibilidade Congruncias O Teorema Chins de Restos e o Pequeno Teorema de Fermat A Funo de Euler Sequncias de Fibonacci Noes sobre o processo e o mtodo de criptografar

59 75 86 97 111 131

Aula 1 Noes sobre o processo e o mtodo de induo


Apresentao Esta a primeira aula da disciplina Teoria dos Nmeros, que pretende: introduz os resultados bsicos da Teoria Elementar dos Nmeros; mostra aplicaes da Teoria dos Nmeros; desenvolve mecanismos de reconhecimento de padres numricos; e introduz o rigor nas provas dos resultados. Nesta primeira aula, introduziremos o processo e o mtodo de induo, uma importante tcnica usada para provar resultados em Matemtica e especialmente na Teoria dos Nmeros. Tente entender tudo que est sendo explicado na aula. Estude com caneta e papel ao lado. Seja paciente e procure ter certeza de que voc entendeu o que (e por que) est fazendo.

Objetivos Compreender a essncia do processo indutivo. Usar o Princpio da Induo para provar a validade de certas frmulas envolvendo nmeros naturais.

O processo e o mtodo indutivo Em muitos problemas de Matemtica, especialmente da Teoria dos Nmeros, precisamos verificar a veracidade de uma afirmao, A(n), que depende de um nmero natural n. Se a afirmao A(n) de fato verdadeira, usamos o mtodo de induo para facilitar a sua prova. Os historiadores da Matemtica tm opinies diferentes sobre quem primeiro formulou o Princpio da Induo Matemtica. Mas, certo que os matemticos da antiga Grcia usaram argumentos indutivos, basta ver o Teorema IX-20 em Os Elementos, de Euclides, onde ele prova a existncia de infinitos nmeros primos. Ateno: ao longo de todas estas Notas de Aula, a letra maiscula N denotar o conjunto dos nmeros naturais:

N = {1, 2, 3, 4, 5, 6, ......, n, .........}. A letra maiscula Z denotar o conjunto dos nmeros inteiros: Z = {...., -3, -2, -1, 0, 1, 2, 3, 4, ..........}
Euclides de Alexandria (360 a.C. 295 a.C.) foi um professor, matemtico e escritor. Teria sido educado em Atenas e freqentado a Academia de Plato, em pleno florescimento da cultura helenstica. Convidado por Ptolomeu I para compor o quadro de professores da recm fundada Academia, que tornaria Alexandria o centro do saber da poca, tornou-se o mais importante autor de matemtica da Antiguidade greco-romana e talvez de todos os tempos, com seu monumental Stoichia (Os elementos, 300 a.C.), uma obra em treze volumes, sendo cinco sobre geometria plana, trs sobre nmeros, um sobre a teoria das propores, um sobre incomensurveis e os trs ltimos sobre geometria no espao. Escrita em grego, a obra cobria toda a aritmtica, a lgebra e a geometria conhecidas at ento no mundo grego, reunindo o trabalho de seus predecessores, como Hipcrates e Eudxio, e sistematizava todo o conhecimento geomtrico dos antigos e intercalava os teoremas j conhecidos ento com a demonstrao de muitos outros, que completavam lacunas e davam coerncia e encadeamento lgico ao sistema por ele criado. Aps sua primeira edio foi copiado e recopiado inmeras vezes e, versado para o rabe, tornouse o mais influente texto cientfico de todos os tempos e um dos com maior nmero de publicaes ao longo da histria. Depois da queda do Imprio Romano, os seus livros foram recuperados para a sociedade europia pelos estudiosos rabes da pennsula Ibrica. Escreveu ainda ptica (295 a.C.), sobre a ptica da viso e sobre astrologia, astronomia, msica e mecnica, alm de outros livros sobre matemtica. Entre eles citam-se Lugares de superfcie, Pseudaria e Porismas. Algumas das suas obras, como Os elementos, Os dados, outro livro de texto, uma espcie de manual de tabelas de uso interno na Academia e complemento dos seis primeiros volumes de Os Elementos, Diviso de figuras, sobre a diviso geomtrica de figuras planas, Os Fenmenos, sobre astronomia, e ptica, sobre a viso, sobreviveram parcialmente e hoje so, depois de A Esfera de Autlico, os mais antigos tratados cientficos gregos existentes. Pela sua maneira de expor nos escritos deduz-se que tenha sido um habilssimo professor. (Fonte: WIKIPDIA, 2008, extrado da Internet).

Alguns historiadores argumentam que a formulao precisa do mtodo e do processo de induo deveu-se a Jacob Bernoulli (1654-1705) e Blaise Pascal (1623 1662). Em 1889, quando estudava os nmeros naturais, Giuseppe Peano (18581932) introduziu o Princpio da Induo Matemtica como um dos axiomas dos nmeros naturais.

Em que consiste o mtodo de induo matemtica? A prova por induo pode ser pensada como a brincadeira de arrumar domins em fila e derrub-los como uma onda: derrubamos a primeira pea, que ao cair bate na segunda, que ao cair bate na terceira e assim por diante, at que todas elas estejam tombadas, conforme mostra a Figura 1.

Figura 1 Domins caindo como uma onda

Agora, em vez de peas de domins, pense numa seqncia de afirmaes A (1), A (2), A (3), ...., A(n), ......... Imagine que seja possvel provar as duas etapas seguintes: (i) (ii) a primeira afirmao, A (1), seja verdadeira; sempre que uma afirmao for verdadeira, a imediatamente seguinte tambm seja verdadeira.

Conclumos que todas as afirmaes A (1), A (2), A (3), ...., A(n), ........ so verdadeiras. Relacionando com as peas de domin, (i) seria a derrubada da primeira pea, (ii) seria a queda de uma pea de domin provocada pela queda da pea anterior. A parte (i) chamada a base da induo e (ii) a etapa indutiva.

Exemplo 1 (A soma dos primeiros n nmeros naturais) Vamos provar, usando induo, que para todo nmero natural n, temos:

1 + 2 + 3 + 4 + .... + n =
Soluo

n(n + 1) 2 n(n + 1) , para 2

Observe que a afirmao A(n) a ser provada : 1 + 2 + 3 + 4 + .... + n = cada nmero natural n.

Assim, A (1) ser entendida como a afirmao: 1 = A (2) : 1 + 2 = 2(2 + 1) ; 2 3(3 + 1) , e assim por diante. A (3): 1 + 2 + 3 = 2

1.(1 + 1) ; 2

Etapa 1 Vamos verificar a base da induo. Isto , (i), que o mesmo que verificar 1.(1 + 1) que A (1) verdadeira. Para isso, basta observar que: 1 = . 2 Etapa 2 Vamos supor que para n = k, onde k um nmero natural maior do que ou k (k + 1) igual a 1, a frmula dada seja verdadeira. Isto , 1 + 2 + 3 + 4 + ... + k = . Agora, 2 provaremos que para n = k +1 a igualdade tambm se verifica. De fato, somando k + 1 em ambos os membros da expresso anterior, obtm-se

1 + 2 + 3 + 4 + .... + k + (k + 1) =

n(n + 1) k + 2) [(k + 1) + 1] k para n = k , que o resultado = ( k + 1).( + 1) = (k + 1).( ) = (k + 1). 2 2 2 2 +1. Portanto, pelo Princpio da Induo, completamos a prova.
Exemplo 2 (A soma dos quadrados dos primeiros n nmeros naturais)

k (k + 1) + (k + 1) = 2

Vamos provar por induo que, para todo nmero natural n, temos: 12 + 2 2 + 3 2 + 4 2 + ... + n 2 =
Soluo

n(n + 1)(2n + 1) 6 n(n + 1)(2n + 1) , para 6

A afirmao, A(n), a ser provada : 12 + 2 2 + 3 2 + 4 2 + ... + n 2 = todo nmero natural n. Assim, A (1) ser entendida como a afirmao A (2): 12 + 2 2 =

2(2 + 1)(2.2 + 1) 2.3.5 = = 5; 6 6 3(3 + 1)(2.3 + 1) A (3): 12 + 2 2 + 3 2 = = 14 e assim por diante. 6

1(1 + 1)(2.1 + 1) = 12 = 1 ; 6

Etapa 1 Vai verificar a base da induo, isto , (i). Para isso, basta observar que: 1(1 + 1)(2.1 + 1) = 12 = 1 . 6

Etapa 2 Vamos supor que para n = k, onde k um nmero natural maior do que ou igual a 1, a frmula dada seja verdadeira. Isto , k (k + 1)(2k + 1) 12 + 2 2 + 3 2 + 4 2 + ... + k 2 = para todo nmero natural k. 6 Agora, provaremos que para n = k +1 a igualdade tambm se verifica. De fato, somando (k + 1)2 em ambos os membros da igualdade anterior, obtm-se:
2k 2 + k + 6k + 6 2k 2 + 7 k + 6 (k + 1)((k + 2)[2(k + 1) + 1] = (k + 1) = (k + 1). , esta = 6 6 6 n(n + 1)(2n + 1) quando n = k + 1, o que completa a prova. a expresso 6

Exerccio 1 Descubra o erro na demonstrao, por induo, do seguinte resultado: Proposio - Todos os objetos possuem a mesma cor. Demonstrao

O caso n = 1 bvio (numa coleo consistindo de um nico objeto, existe uma nica cor a ser observada). Assumindo, por hiptese, que em qualquer coleo de k objetos todos tm a mesma cor, ento, segue que toda coleo de k + 1 objetos ser formada totalmente por elementos monocromticos. De fato, se retirarmos um objeto de uma coleo de k + 1 objetos, os k restantes sero todos da mesma cor (pela hiptese). Agora, se colocamos de volta o que foi retirado e retiramos outro qualquer, ento, (novamente) pela hiptese de induo, ele tem a mesma cor dos objetos restantes e, com isso, conclumos que todos os objetos possuem a mesma cor.

Exemplo 3 (A soma dos cubos dos primeiros n nmeros naturais)

Vamos provar por induo que, para todo nmero natural n, temos:
n(n + 1) 1 + 2 + 3 + 4 + ... + n = 2 Soluo
3 3 3 3 3 3 3 2

n(n + 1) para todo A afirmao, A(n), a ser provada : 1 + 2 + 3 + 4 + ... + n = 2 nmero natural n.
3 3 3

1.(1 + 1) ; Assim, A (1) ser entendida como a afirmao 1 = 2 2(2 + 1) ; A (2): 1 + 2 = 2


3 3 2

3(3 + 1) , e assim por diante. A (3) : 1 + 2 + 3 = 2


3 3 3

Etapa 1 Vamos verificar a base da induo. Isto , (i). Assim, vamos verificar que

1.(1 + 1) 2 verdadeira. Para isso, basta observar que: 1 = = 2 = 1. 2


3

Etapa 2 Vamos supor que para n = k, onde k um nmero natural maior do que ou igual a 1, a frmula dada seja verdadeira. Isto ,

k (k + 1) 13 + 2 3 + 33 + 4 3 + ... + k 3 = , para todo nmero natural k. 2 Agora, provaremos que para n = k +1 a igualdade tambm se verifica. De fato, 2 k 2 k (k + 1) 3 3 3 3 3 3 1 + 2 + 3 + 4 + ... + k + (k + 1) = + (k + 1) 3 = (k + 1) 2 + (k + 1) = 2 4 k 2 + 4k + 4 (k + 1)(k + 2) 2 k + 2 = (k + 1) 2 = = (k + 1) . , que a frmula 4 2 2 n(n + 1) 2 para n = k + 1, o que completa a prova.
2 2 2

Observe que, pelo exemplo 1, temos 1 + 2 + 3 + 4 + .... + n = exemplo 2, curiosamente, nos permite concluir que: n(n + 1) 2 13 + 2 3 + 33 + 4 3 + ... + n 3 = = (1 + 2 + 3 + 4 + .... + n) , 2 um resultado, convenhamos, surpreendente!
2

n(n + 1) . Agora, o 2

Exerccio 2 Prove, por induo, que para todo nmero natural n, vale as seguintes igualdades:

(a)

1 1 1 1 n 1 + + + ... + = 1.2 2.3 3.4 (n 1).n n

x n +1 1 x 1 1 1 n +1 1 1 (c) 1 1 1 ...1 2 = 2n 4 9 16 n 1 1 1 1 1 1 1 1 1 = + + + ... + . (d) 1 + + ... + 2 3 4 2n + 1 2n n + 1 n + 2 n + 3 2n (b) 1 + x + x 2 + x 3 + ... + x n =

A Torre de Hani

A Torre de Hani um quebra-cabea que foi apresentado em 1883 por douard Lucas (1842-1891), um professor do Lyces Saint-Louis, na Frana, no seu livro Rcrations Mathmatiques, volume III, p. 56. Lucas anexou ao seu brinquedo uma lenda romntica sobre uma torre, a Torre de Brama, que supostamente tem 64 discos de ouro empilhados em trs agulhas de diamantes: No incio dos tempos, ele disse, Deus colocou estes discos de ouro na primeira agulha e mandou um grupo de sacerdotes transferir para a terceira agulha, movendo apenas um disco de cada vez e sem colocar um disco maior em cima de um menor. Os sacerdotes, ao que se saiba, trabalham dia e noite nesta tarefa. Quando eles terminarem, a Torre ruir e o mundo ir acabar. A primeira soluo do problema da Torre de Hani apareceu na literatura matemtica em 1884, num artigo de Allardice e Farser, La Tour dHanoi publicado em Proc. Edinburgh Math. Soc., v. 2, p. 50 53, 1884. A seguir, enunciamos o problema da Torre de Hani de forma mais geral.
Exemplo 4 (A Torre de Hani)

dada uma torre com n discos, inicialmente empilhados por tamanhos decrescentes em um dos trs pinos dados, conforme Figura 2. O objetivo transferir a torre inteira para um dos outros pinos, movendo apenas um disco de cada vez e sem colocar um disco maior em cima de um menor. a) Determine a menor quantidade de movimentos necessrios para transferir todos os discos de um dos pinos para outro.

Figura 2 - A Torre de Hani

b) Mais precisamente: prove que podemos realizar a transferncia dos n discos, de acordo com as regras de douard Lucas, com, no mnimo, 2n 1 movimentos.

Soluo

a) Na nossa notao anterior, a afirmao, A(n), a ser provada : a menor quantidade de movimentos para transferir os n discos igual a 2n 1.

Assim, A (1) ser entendida como a afirmao: A quantidade mnima de movimentos para transferir um disco igual a 21 1 = 1; A (2): a menor quantidade de movimentos para transferir dois discos igual a 22 - 1; A (3): a menor quantidade de movimentos para transferir trs discos igual a 23 1, e assim por diante.
Etapa 1 Vamos verificar a base da induo. Ou seja, vamos verificar que A (1) verdadeira. Para isso, observe que, para transferir um s disco, basta um nico movimento. Nesse caso, 1 = 21 1 e a frmula se verifica. Etapa 2 Vamos supor que para n = k, onde k o nmero de discos, a menor quantidade de movimentos para realizar a transferncia seja 2k - 1. Naturalmente, k um nmero natural maior do que ou igual a 1.

b) Agora, provaremos que, para n = k +1 discos, o nmero mnimo de movimentos que realizam a transferncia dado por 2n+1 1. De fato, se temos (k + 1) discos, podemos pensar em dois blocos de discos: um bloco com k discos, contendo todos os discos, com exceo do disco maior, que est embaixo da pilha, e outro, s com o disco maior. Pela hiptese de induo, podemos transferir os k primeiros discos com, no mnimo, 2k 1 movimento. Assim, transferimos o bloco contendo k discos para um dos pinos vazios, realizando 2k 1 movimentos e, em seguida, transferimos o disco maior para o outro pino vazio e, por ltimo, transferimos o bloco dos k discos para o pino em que se encontra o disco maior, com no mnimo 2k 1 movimentos. Em seguida, com um movimento transferimos o disco maior para o outro pino vazio. Portanto, o total mnimo de movimentos realizados foi: (2k 1) + 1 + (2k 1) = 2.2k 1 = 2k+1 1, o que conclui a prova. No caso de n = 64 discos, o nmero mnimo de movimentos ser 264 1, necessrios antes que o mundo se acabe... Agora, observe que o nmero 264 1 igual a 18.446.744.073.709.551.615. Se fizermos uma transferncia por segundo, 24 horas por dia, durante 365 dias no ano, levaramos 58.454.204.609 sculos e mais seis anos para terminar o trabalho! oportuno observar que a demonstrao por induo exige que comprovemos que uma dada afirmao A(n) sobre nmeros naturais seja verdadeira para n = 1. E, supondo verdadeira para n = k, possamos, atravs de meios legtimos, provarmos que ela seja verdadeira para n = k + 1. Somente aps essas duas etapas, podemos concluir que A(n) verdadeira para todo nmero natural n. Os dois exemplos a seguir ilustram bem o teor dessa observao.
Exemplo 5 A expresso n 2 + n + 41 representa um nmero primo para n = 1, 2, 3, ..., 39 (verifique essa afirmao, substituindo na expresso dada os valores: n = 1, n = 2, ...., n =39). Mas, para n = 40, temos: n 2 + n + 41 = 40 2 + 40 + 41 = 40(40 + 1) + 41 = 40 41 + 41 = 41 41 = 412 , que no um nmero primo. Esse clculo evidencia que a validade para n = 39 no implica a validade para n = 40. Esse um exemplo famoso que foi dado por Leonardo Euler.

10

Exemplo 6 Suponha verdadeira a seguinte afirmao: (n 1)(n + 2) A(n) = 1 + 2 + 3 + 4 + .... + n = . 2

Ento, a afirmao:

(n 1)(n + 2) +n+1= 2 (n 1)(n + 2) 2(n + 1) n 2 + n 2 + 2n + 2 n 2 + 3n n(n + 3) + = = = = 2 2 2 2 2 A (n + 1) = (1 + 2 + 3 + 4 + .... + n) + n + 1 =

verdadeira. Logo, a veracidade de A(n) implica a de A (n + 1). No entanto, A (1) = 1, (n 1)(n + 2) enquanto = 0, para n = 1. 2 Isso nos diz que A (1) no verdadeira. Portanto, no podemos concluir que A(n) verdadeira para todo n. Na verdade, como sabemos do exemplo 1, (n 1)(n + 2) 1 + 2 + 3 + 4 + .... + n . 2

Exerccio 3 Para cada uma das proposies, encontre os valores de n N para os quais a proposio verdadeira, e tambm o menor valor de n N para o qual a proposio falsa. (b) n2 + n + 1 um nmero primo. (a) 2n-1 n2

possvel demonstrar por induo a validade dessas duas proposies? Justifique.


Exemplo 7 (Jakob Steiner 1796-1863)

Mostre, usando induo, que o nmero mximo de regies definidas por n retas no n(n + 1) +1. plano Ln = 2 Soluo Para n = 1, o plano fica dividido em duas regies, veja a Figura 3 a seguir.

11

Figura 3 Reta dividindo o plano em duas regies

Desse modo, L1 = 2 =

1.(1 + 1) + 1 . O que mostra que a afirmao verdadeira para n = 2

1. Traando duas retas (no coincidentes), conforme a Figura 4, o plano fica dividido em 4 2.(2 + 1) regies. A expresso dada verdadeira: L2 = 4 = + 1. 2

Figura 4 Duas retas dividindo o plano em quatro regies

Agora, observe que, traando uma terceira reta, verificamos que esta divide no mximo trs das quatro regies j existentes, independentemente da posio das duas primeiras retas traadas, veja a Figura 5 a seguir.

Figura 5 Trs retas dividindo o plano em sete regies

Desse modo, com n = 3 retas, dividimos o plano em, no mximo, 7 regies e a frmula 3.(3 + 1) se verifica: L3 = 7 = +1. 2 Observe que, L2 = L1 + 2, L3 = L2 + 3.

12

Agora, para n 1, a n-sima reta aumenta o nmero de regies do plano de k se, e somente se, essa reta divide k das regies j existentes. Por outro lado, a n-sima reta intercepta k regies j existente se ela intercepta as retas anteriores em k 1 pontos. Mas, duas retas se interceptam em no mximo um ponto. Portanto, a n-sima reta s pode interceptar as n - 1 retas anteriores em no mximo n 1 pontos. Desse modo, como k n, Ln Ln-1 + n. Agora, desenhando a n-sima reta de tal maneira que ela no seja paralela a nenhuma das outras n - 1 retas e no passe por nenhum dos pontos de interseo j existentes, temos que Ln = Ln-1 + n. Essa igualdade foi verificada acima para n = 2 e n = 3. Observe que o passo da induo de n para n + 1 dado por: Ln+1 = Ln + (n + 1) = (n + 1)[(n + 1) + 1] n(n + 1) n(n + 1) n + 1 + (n + 1) = + (n + 1) + 1 = (n + 1) + 1 + 1 = +1 2 2 2 2 Portanto, o nmero mximo de regies definidas por n retas no plano Ln = n(n + 1) + 1 , para todo nmero natural n. 2

Exemplo 8 Desenha-se n crculos num dado plano. Eles dividem o plano em regies. Mostre que possvel pintar o plano com duas cores, azul e verde, de modo que regies com fronteira comum tenham cores distintas. Soluo

Se n = 1, ento, o plano fica dividido em duas regies, uma externa ao crculo, que pintada de azul, e a outra, regio interna, pintada de verde, veja a Figura 6.

Figura 6 O plano dividido por um crculo em duas regies, pintadas com cores distintas.

Se n = 2, fcil ver que podemos pintar o plano com duas cores, de modo que regies com fronteira comum tenham cores distintas, veja a Figura 7 a seguir.

13

Figura 7 O plano dividido em trs regies, pintadas de acordo com o problema.

Observe que, para fazer a pintura, no caso, de dois crculos, raciocinamos do seguinte modo: temporariamente, retiramos o segundo crculo e pintamos o plano de acordo com o caso de um nico crculo. Agora, recolocamos o segundo crculo, deixando fixa a cor da regio externa a ele e mudamos a cor da regio comum aos dois crculos, de acordo com a Figura 7. Vamos supor que, para o caso de n, seja possvel pintar o plano com duas cores, de modo que regies com fronteira comum tenham cores distintas. Para o caso de n + 1 crculos, raciocinamos como anteriormente. Isto , removemos temporariamente o (n + 1)-simo mais um- crculo. O que sobra est nas condies da hiptese de induo, para o caso de n crculos. Agora, recolocamos o (n + 1)-simo crculo e mudamos alternadamente as cores das regies internas a ele, deixando fixa a cor externa. Desse modo, as cores de regies adjacentes ficam distintas. Portanto, para todo nmero natural n, possvel pintar o plano com duas cores, de modo que regies com fronteira comum, determinadas pelos n crculos, tenham cores distintas.

Exerccios
(1) Mostre por induo que: (a) 1.2 + 2.3 + 3.4 + 4.5 + .... + n( n + 1 ) = (b) n( n + 1 )( n + 2 ) 3

1 1 1 1 1 n + + + + .... + = 1.2 2.3 3.4 4.5 n( n + 1 ) n + 1

(2) Prove, usando induo, que o nmero de diagonais, dn, de um polgono convexo de n( n 3 ) n lados dado pela expresso: d n = , n 4. 2 (3) Prove, usando induo, que a soma, S n , das medidas dos ngulos internos de um polgono convexo de n lados dado pela expresso: S n = ( n 2 ).180 o , n 3
(4) Considere uma Torre de Hani dupla contendo 2n discos de n tamanhos diferentes, dois de cada tamanho. Como usual, s podemos mover um disco de cada vez, sem colocar um maior sobre um menor.

14

Qual o nmero mnimo de movimentos necessrio para transferir a Torre de Hani dupla, de um pino para outro, se discos de mesmo tamanho so indistinguveis?
Sugesto Inicialmente, pense em mover a torre dupla com (n 1) discos, depois, mover e inverter a ordem dos dois maiores discos e, finalmente, mover a torre dupla com (n 1) discos.

(5) Num pas longnquo, a moeda local o cruzeiro. Nesse pas, um banco tem uma quantidade ilimitada de cdulas de 3 e 5 cruzeiros. Prove, por induo, que o banco pode pagar uma quantidade qualquer (inteira) de cruzeiros, maior do que sete. (Sugesto: faa induo sobre o nmero de cruzeiros que o banco tem de pagar. Mostre que: se o banco pode pagar k, k +1, k + 2 cruzeiros, ento, o banco pode pagar k + 3, k +4, k + 5 cruzeiros). (6) permitido cortar uma folha de papel em 4 ou 6 pedaos. Prove que, aplicando essa regra, pode-se cortar uma folha de papel num nmero qualquer de pedaos maior do que 8. (7) Prove, por induo, que todo nmero natural n pode ser representado como soma de diferentes potncias de 2, que a expanso de n na base 2.
Resumo

Nesta aula, estudamos o processo e o mtodo de induo, que consiste em provar afirmaes envolvendo nmeros naturais. Para tanto, temos que provar que a afirmao verdadeira para n = 1 e, supondo verdadeira para n = k, se pudermos provar que ela verdadeira para n = k + 1, ento, ela verdadeira para todo nmero natural n.

Problemas Suplementares
Problema 1 Usando o Princpio da Induo, mostre que:

(a) n3 3n + 1, para todo nmero inteiro maior do que ou igual a 2. (b) n3 3n2, para todo nmero inteiro maior do que ou igual a 3. (c) 3n n3, para todo nmero inteiro positivo. (d) sen. nos n.sen. x, para todo nmero real x e todo inteiro positivo n.
Problema 2 Prove, usando o Princpio da Induo, que todo polgono de n lados (convexo ou no) pode ser repartido em tringulos traando diagonais. Problema 3 Prove, usando o Princpio da Induo, que todo nmero inteiro positivo n pode ser escrito como uma soma do tipo 12 22 32 .... k2 , para algum inteiro positivo k e alguma escolha de sinais.

15

Problema 4 Cada vrtice de um polgono convexo de n lados pintado com uma cor. Para pintar todos os vrtices usam-se no mnimo trs cores, de tal maneira que vrtices consecutivos tm cores distintas. Prove, por induo, que podemos repartir o polgono em tringulo, usando diagonais que no se interceptam, de modo que os extremos das diagonais sejam pontos de cores distintas. Problema 5 Numa ilha, cinco piratas dispem de cem moedas de ouro para repartir entre si. Eles dividem o produto do saque da seguinte forma: o pirata mais velho prope uma diviso e todos votam sim ou no. Se pelo menos a metade dos piratas vota sim, eles dividem as moedas da forma proposta. Caso contrrio, matam o pirata mais velho e comeam de novo. Ento o pirata mais velho (sobrevivente) faz sua proposta de diviso e os outros votam de acordo com as mesmas regras, ou seja, repartem as moedas ou matam o mais velho, conforme o caso. O processo continua at que um plano seja aceito. Suponha que voc o pirata mais velho. Que diviso voc proporia, sabendo que todos os piratas so lgicos, gananciosos e querem continuar vivos? Problema 6 O professor de Matemtica escreve no quadro negro os nmeros 1, 2, 22, 23, ...., 2n e prope o seguinte desafio. Um estudante pode apagar quaisquer dois dos nmeros escritos e substitu-los pela diferena, tomada sempre maior do que ou igual a zero. Depois que este procedimento for repetido n vezes, restar um nico nmero. Que nmeros podem restar no final dos procedimentos?

16

Referncias
ANDREESCU, Titu; GELCA, Razvan. Putnam and Beyond. New York. Springer. 2007. FERNANDES, ngela Maria Vidigal et al. Fundamentos de lgebra. Belo Horizonte: Editora UFMG, 2005. FOMIN, Dmitri; GENKIN, Sergey; ITENBERG, Ilia. Mathematical circle: Russian experience. Roadiland. American Mathematical Society, 1996. GRAM, Ronald L.; KNUTH, Ronald E.; PATASHNIK, Oren. Matmtica concreta: fundamentos para a cincia da computao. Rio de Janeiro: LTC, 1995. HEFEZ, Abramo. Elementos de aritmtica. Rio de Janeiro: Sociedade Brasileira de Matemtica, 2005. POUNDSTONE, William. Como mover o Monte Fuji?. Rio de Janeiro. Ediouro. 2005. STOROZHEV, Andrei. International Mathematics Tournament of Towns 19972002. Book 5. Canberra. AMT Publishing. 2006 WIKIPDIA. Euclides. Disponvel em: <http://pt.wikipedia.org/wiki/Euclides>. Acesso em: 2 dez. 2008.

17

AULA 02 - Divisibilidade
Apresentao

Nesta aula, estudaremos o conceito de Divisibilidade no conjunto dos nmeros inteiros, bem como suas propriedades bsicas, que so importantes na Teoria dos Nmeros. Tente entender tudo que est sendo explicado na aula. Estude com caneta e papel ao lado. Seja paciente e procure ter certeza que voc entendeu o que (e por que) est fazendo.

Objetivos

Com esta aula espera-se que voc possa: Reconhecer quando um inteiro divide o outro; Usar as propriedades bsicas da diviso de inteiros; Saber o que um nmero primo.

PROPRIEDADES BSICAS DOS NMEROS INTEIROS

O conjunto dos nmeros naturais N = {1, 2, 3, 4, 5, 6, ....} o primeiro conjunto numrico que conhecemos, e o cenrio para os mais antigos e profundos problemas da Matemtica. Sabe-se que a soma (e o produto) de dois nmeros naturais resulta em um nmero natural. J a diferena entre dois nmeros naturais nem sempre um numero natural. Por exemplo, 5 8 = 3 e 4 4 = 0, que no so nmeros naturais. Para suprir esta deficincia, amplia-se o conjunto dos nmeros naturais para o conjunto dos nmeros inteiros:
Z = {...-5, -4, -3, -2, -1, 0, 1, 2, 3, 4, 5, ...},

que contm o conjunto N, o nmero zero e todos os inteiros negativos. Os nmeros naturais, 1, 2, 3, 4, 5, ... , so os inteiros positivos. Simbolicamente, dizemos que um nmero inteiro n positivo da seguinte maneira: n > 0 (leia-se: n maior do que zero ou n positivo). Os nmeros....,-5, -4, -3, -2, -1 so os inteiros negativos. Simbolicamente, dizemos que um nmero inteiro n negativo da seguinte maneira: n < 0 (leia-se: n menor do que zero ou n negativo). O nmero zero, 0, no negativo nem positivo. Como sabemos, os inteiros so munidos de duas operaes: + (adio) e x ( multiplicao, tambm representada por.). Se a, b e c so nmeros inteiros quaisquer, valem as seguintes propriedades para as duas operaes de adio e multiplicao na tabela a seguir:

18

Tabela 1 Propriedades bsicas da adio e multiplicao de inteiros


Propriedade Associatividade Comutatividade Identidade Inverso Distributividade Lei do Cancelamento Adio a + (b + c) = (a + b ) + c a+b=b+a a+0=0+a=a a + (-a) = 0 = (-a) + a a.(b + c) = a.b + a.c (a+b).c = a.c + b.c a+c=b+ca=b Multiplicao a.(b.c) = (a.b).c a.b = b.a a.1 = 1.a = a

a.c = b.c a = b, se c 0

Se a, b e c so nmeros inteiros quaisquer, dizemos que a > b (leia-se a maior do que b) se, e somente se, a b um nmero inteiro positivo.

Propriedades dos Inteiros

Duas importantes propriedades dos inteiros so: (a) Tricotomia Se a e b so nmeros inteiros quaisquer: ou a > b ou b > a ou a = b. (Ou seja, dados dois inteiros a e b, uma, e somente uma, das possibilidades ocorre: ou a maior do que b ou b maior do a ou a igual a b) (b) Transitividade Se a, b e c so nmeros inteiros quaisquer com a > b e b > c, ento a > c
OS CONCEITOS DE DIVISIBILIDADE E NMERO PRIMO

Por volta de 300 a.c., Euclides reconheceu que a divisibilidade e os nmeros primos so conceitos importantes para os nmeros naturais. A seguir, vamos tratar destes dois conceitos. Como podemos escrever 10 = 2 x 5, dizemos que 2 divide 10 (e que 5 divide 10). Ou seja, 2 um divisor de 10 (e 5 um divisor de 10) ou 2 um fator de 10 (e 5 um fator de 10).

Dizemos tambm que 10 um mltiplo de 2 (e 10 mltiplo de 5). Do mesmo modo, como 1001 = 11 x 91, dizemos que 11 divide 1001 (e que 91 divide 1001). Ou seja, 1001 um mltiplo de 11 (e 1001 mltiplo de 91).

19

De uma maneira geral, se a e b, com b diferente de zero, so nmeros inteiros e existe outro nmero inteiro c tal que a = b.c, dizemos que b divide a (e que c divide a). Neste caso, b (ou c) dito ser um divisor de a ou um fator de a. Dizemos, tambm, que a mltiplo de b e c. Vejamos um exemplo, como 28 = 7 x 4, dizemos que 7 divide 28 (e que, tambm, 4 divide 28). Portanto, 7 e 4 so divisores de 28 ou fatores de 28 e 28 mltiplo de 4 e de 7. De modo anlogo, -4 um divisor de 28 (ou um fator de 28), pois 28 = (-4) x (-7) e -3 um divisor de -12, pois -12 = (-3) x 4. Veja que os divisores de um nmero inteiro comparecem aos pares e quando b.c = a, a a ento b = e c= . c b Observe que a noo de divisibilidade restrita ao fato de o divisor ser diferente de zero. Esta restrio provm do fato de que para todo inteiro a, tem-se a.0 = 0, o que implica 0 a = , para todo a N , o que no faz sentido. Portanto, 0 no pode ser divisor de qualquer 0 inteiro. Observe que 7 no divide 12, pois no existe outro nmero inteiro c tal que 12 = 7 x c. Do mesmo modo, 20 no divide 30, pois no existe outro nmero inteiro c tal que 30 = 20 x c.

EXEMPLO 1 O conjunto S = {....., -12, -8, -4, 0, 4, 8, 12, 16, 20, ...} a coleo de todos os mltiplos inteiros de 4, pois: .., -12 = 4 .(-3); -8 = 4 .(-2); -4 = 4. (-1); 0 = 4.0; 4 = 4.1; 8 = 4.2; 12 = 4.3; 16 = 4.4; ..
Deste modo, podemos descrever S como sendo a coleo de todos os inteiros da forma 4s, onde s um nmero inteiro: S = {4s; s Z}. Do modo anlogo, U = {7, 14, 21, 28, 35, ....} o conjunto dos mltiplos inteiros positivos de 7. Ou seja, U = {7m; m Z} A seguir, vamos listar as propriedades bsicas da diviso, apresentando suas respectivas provas.

Propriedades Bsicas da Diviso


Se a, b e c so nmeros inteiros, valem as seguintes propriedades: (1) (2) (3) (4) (5) (6) Se a 0, ento a divide a e a divide 0. Para qualquer a, 1 divide a. Se a divide b e a divide c, ento a divide bm + cn, para todo m e n inteiros. Se a divide b e b divide c, ento a divide c. Se a > 0 e b > 0, a divide b e b divide a, ento a = b. Se a > 0 e b > 0, a divide b, ento a b.

20

Prova (1) e (2). Estas propriedades seguem do fato que a . 1 = a. e a . 0 = 0 (3) Se a divide b, ento existe x inteiro tal que b = a.x; Se a divide c, ento existe y inteiro tal que c = a.y. Assim, bm + cn = a.xm + a.yn = a.(xm + yn). Portanto, a divisor de bm + cn. (4) Se a divide b, ento existe x inteiro tal que b = a.x. Se b divide c, ento existe y inteiro tal que c = b.y. Assim, c = b.y = (a.x).y = a.(x.y). Portanto, a divide c. (5) Se a divide b, ento existe x inteiro positivo tal que b = a.x. Se b divide a, ento existe y inteiro positivo tal que a = b.y. Assim, b = a.x = (by).x = b.(yx) yx = 1 y = 1 e x = 1. Portanto, a = b. (6) Se a divide b, ento existe x inteiro positivo tal que b = a.x. Logo, a b, pois como a > 0 e x > 0 tem-se a ax = b.
comum a seguinte notao para dizer que um inteiro a divide o inteiro b: a b. Se a no divide b, notamos a b

EXEMPLO 2 Usando as propriedades bsicas da diviso acima, concluir que:


3 (12m + 21n), quaisquer que sejam m, n inteiros. Soluo fcil ver que 3 12 e 3 21, pois 12 = 3.4 e 21 = 3 7. Deste modo, (12m + 21n) = (3.4.m + 3.7.n) = 3.(4m + 7n). Portanto, 3 (12m + 21n).

EXERCCIO 1 (1) Verifique que 20 divisvel por cada um dos seguintes nmeros: 1, 2, 4, 5, 10, 20. (2) Verifique que 30 mltiplo de cada um dos seguintes nmeros: 1, 2, 3, 5, 6, 10, 15, 30. 3) Observe que 45 + 60 = 105 e 105 60 = 45. (a) Explique porque qualquer divisor de 45 e 60 tem de ser um divisor de 105. (b) Explique porque um divisor comum de 105 e 60 tem de ser um divisor comum de 45. 4) Verifique que 3 divide 228 e que 5 divide 725, mas 15 no divide 228 nem 725.

EXEMPLO 3 Verifique que a soma de trs mltiplos de 5 , tambm, um mltiplo de 5. Soluo


Observe que um mltiplo de 5 da forma 5s, onde s um nmero inteiro. Assim, se s, t e u so nmeros inteiros, ento 5s + 5t + 5u a soma de trs mltiplos de 5. Mas, 5s + 5t + 5u = 5(s + t + u), que um mltiplo de 5, pois (s + t + u) um inteiro. Portanto, a soma de trs mltiplos de 5 , tambm, um mltiplo de 5.

EXERCCIO 2

21

Explique porque a diferena entre dois mltiplos de 7 tem de ser um mltiplo de 7. E se em vez de 7 for 12? E se em vez de 12 for n? Observe que, por mais que tentemos, no vamos conseguir escrever o nmero 7 como produto de dois nmeros naturais a b , a menos que a = 7 e b = 1 ou a = 1 e b = 7. Isto quer dizer que: 7 s admite como divisores os nmeros naturais 1 e 7. Neste caso, Euclides chamou 7 (e todos os nmeros com essa propriedade) de nmero primo. fcil ver que 2, 3, 5, 7, 11, 13, 17, 19, 23, 29, 31, 37, 41, 43, 47, 53, 59, 61, 67, 71, 73, 79, 83, 89 e 97 so todos os nmeros primos menores do que 100.

Uma pergunta: Existem quantos nmeros primos?


Vamos demonstrar na Aula 5 que existem infinitos nmeros primos. Voc viu acima que existem 25 nmeros primos menores do que 100. De uma maneira geral, dizemos que um nmero natural p maior do que 1 primo se p no pode ser escrito como produto de dois naturais entre 1 e p. Observe que um nmero primo , por definio, um nmero inteiro maior do que 1. Se um nmero natural no primo, como, por exemplo, o nmero 10 = 2.5, dizemos que ele um nmero composto. fcil ver que 15 um nmero composto, pois 15 = 3 . 5. Tambm o nmero 1071 composto, pois 1071 = 3. 357. O nmero 2047 composto, pois 2047 = 23 . 89. fcil ver que, proposies simples sobre os nmeros naturais podem envolver mtodos delicados de prova. De fato, para provar a proposio seguinte (aparentemente fcil): (*) (EUCLIDES) Se um nmero primo p divide o produto de dois nmero naturais a.b, ento p divide a ou p divide b Euclides introduziu o conceito de Mximo Divisor Comum (MDC) e usou o Algoritmo da Diviso para expressar o MDC numa forma conveniente. Estes assuntos sero tratados nas aulas seguintes. Para encerrar esta aula, vamos provar o seguinte resultado:

Dado um nmero inteiro positivo n, ento o menor nmero natural maior do que 1 que divide n um nmero primo.
Vamos supor que p o menor nmero natural maior do que 1 que divide o nmero inteiro n. Assim, existe um nmero inteiro c tal que n = p.c. Se p no primo, ento p = a.b, onde 1 < a < p e 1 < b < p. Isto , n = p.c = a.b.c. Deste modo, a um divisor de n e a menor do que p. Contradio, pois p o menor nmero natural maior do que 1 que divide n. Portanto, p um nmero primo.

22

Exemplificando, o menor nmero natural maior do que 1 que divide 75 3, que primo: 75 = 3.25. O menor nmero natural maior do que 1 que divide 87 3, que primo: 87 = 3. 29. O menor nmero natural maior do que 1 que divide 100 2, que primo: 100 = 2.50.

EXEMPLO 4 Qual o menor nmero natural maior do que 1 que divide o nmero 91? Soluo
fcil ver que 91 = 7 . 13. Portanto, o menor nmero natural maior do que 1 que divide 91 7 e, de fato, 7 um nmero primo.

EXEMPLO 5 Se a e b so inteiros positivos e ab = n, ento podemos concluir que a n ou b n . Soluo


Basta observar que: se a b, ento n = ab a.a = a2. Logo, a n . Por outro lado, se a b, ento n = ab b.b = b2. Logo, b n . Conclumos que, o resultado acima indica que, um nmero inteiro n no primo se ele tem um divisor positivo menor do que ou igual n .

EXEMPLO 6 Diga, justificando, se o nmero 377 primo. Soluo


Como 377 = 19,416..... , pelo Exemplo 5, basta procurar um divisor para 377 dentre os inteiros primos de 1 at 19. fcil ver que 13 divide 377 e, portanto, 377 no primo.

EXERCCIO 3 Diga, justificando, se os nmeros 187 e 211 so primos.

EXERCCIO 4 Identifique qual dos seguintes inteiros primo: (a) 91 (b) 191 (c) 791 (d) 771

EXEMPLO 6 A Teoria dos Nmeros nos permite tratar do lado ldico da Matemtica.
Vivencie a seguinte brincadeira: Como adivinhar a idade do amigo (a) ? Instrues: Pea que ele (ela) escreva dois dgitos cuja diferena seja maior do que um. Que entre os dois dgitos escreva um algarismo qualquer.

23

Pea que inverta a ordem dos algarismos do nmero obtido. Pea que diminua o menor nmero obtido do maior. Pea que inverta a ordem dos dgitos da diferena acima obtida. Pea que some o ltimo nmero obtido ao resto anterior. Pea que some o nmero obtido idade do amigo (a). Pea para ele dizer qual o ltimo resultado obtido. Voc ento dir a idade do amigo (a). Qual o truque?

Soluo
Vamos imaginar que seu amigo escreveu os dgitos 7 e 2 e entre os dois colocou o nmero 4, formando o nmero 742. Seguindo as instrues, invertendo o nmero, obtm-se 247. Diminuindo o menor nmero do maior, tem-se: 742 247 = 495. Invertendo a ordem dos dgitos da diferena obtida, encontramos 594, que somado a 495 nos d: 594 + 495 = 1089. Se a idade do amigo for, por exemplo, 17, voc soma 1089 + 17 = 1106. Qual o truque? O truque o seguinte: qualquer que seja a escolha dos dgitos, antes de somar a idade do amigo, encontramos sempre o resultado 1089. Quando ele diz o resultado final, voc subtrai 1089, obtendo a idade do amigo. Experimente com outros valores e comprove!

EXERCCIOS
1) Prove que: se a b e a c, ento: (i) a (b +c) (ii) a (b c)

2) Na tabela a seguir, identifique cada nmero primo, pintando de vermelho o quadradinho em que ele se encontra. Tabela 2 Os nmeros naturais de 1 a 100. 1 11 21 31 41 51 61 71 81 91 2 12 22 32 42 52 62 72 82 92 3 13 23 33 43 53 63 73 83 93 4 14 24 34 44 54 64 74 84 94 5 15 25 35 45 55 65 75 85 95 6 16 26 36 46 56 66 76 86 96 7 17 27 37 47 57 67 77 87 97 8 18 28 38 48 58 68 78 88 98 9 19 29 39 49 59 69 79 89 99 10 20 30 40 50 60 70 80 90 100

3) Responda as seguintes perguntas: a) Qual o menor inteiro positivo, maior do que 1, que divide 4189? b) Qual o menor inteiro positivo, maior do que 1 que divide 2627? 4) Dois amigos, A e B, se divertem com o seguinte jogo. O jogador A inicia o jogo escolhendo um dos inteiros de 1 a 8 (inclusive) e diz ao segundo jogador, que escolhe

24

um nmero qualquer de 1 a 8 e soma esse nmero com o que A escolheu, falando o resultado ao primeiro jogador, que, por sua vez, o soma a qualquer nmero escolhido de 1 a 8, e assim eles vo jogando, alternadamente. O jogador que primeiro obtiver o nmero 46 perde o jogo. Quem vence: A ou B? Qual a estratgia para vencer? (Sugesto: O vencedor na sua ltima jogada, para ter certeza que vence o jogo, deve atingir que nmero? Qual a primeira jogada do vencedor?) 5) Encontre trs divisores inteiros positivos de 30. 6) Diga, justificando, se 39 um nmero primo ou composto. 7) D exemplo de quatro pares de nmeros naturais (p, q), onde p e q so primos, com p > q e p q = 2. 8) Escreve-se no quadro-negro os nmeros inteiros de 1 a 15. Voc escolhe quaisquer dois destes nmeros, apaga-os, e junta lista a soma deles. Depois de quatorze operaes deste tipo, resta somente um nmero sobre o quadro-negro. Em cada operao realizada, possvel fazer escolhas de modo que o nmero final seja 105? (Sugesto: Quanto a soma: 1 + 2 + 3 + 4 + ... + 15?) 9) Dois amigos, A e B, se divertem com o seguinte jogo. O jogador A inicia o jogo escolhendo um dos inteiros de 1 a 8 (inclusive) e diz ao segundo jogador, que escolhe um nmero qualquer de 1 a 8 e soma esse nmero com o que A escolheu, falando o resultado ao primeiro jogador, que, por sua vez, o soma a qualquer nmero escolhido de 1 a 8, e assim eles vo jogando, alternadamente, at que um deles obtenha o nmero 46, vencendo o jogo. a) Quem vence o jogo: A ou B? b) Qual a estratgia para vencer? (Sugesto: O vencedor na sua penltima jogada, para ter certeza que vence o jogo, deve atingir que nmero? Qual a primeira jogada do vencedor?)

RESUMO
Nesta aula estudamos os conceitos de divisibilidade e suas propriedades e o de nmero primo, indispensveis e fundamentais no estudo da Teoria dos Nmeros.

Problemas Suplementares
Problema 1 Encontre todos os nmeros inteiros positivos n para os quais os nmeros n10 + 1 sejam divisveis por n + 1. Problema 2 Mostre que: para todo inteiro positivo n, o nmero n4 + 64 um nmero composto.

25

Problema 3 Seja p um nmero primo maior do que ou igual a 5. Mostre que p2 + 2 um nmero composto. Problema 4 Encontre o menor nmero inteiro positivo n para o qual os nmeros:
n, n + 1, n + 2, n + 3, n + 4, n + 5, n + 6, sejam todos compostos.

Problema 6 Mostre que no existe polinmio com coeficientes inteiros, P(x), satisfazendo as igualdades: P(7) = 5 e P(15) = 9.

Referncias
ANDREESCU, Titu; GELCA, Razvan. Putnam and Beyond. New York. Springer. 2007. Burton, David M. Elementary Number Theory. The McGraw-Hill Companies, Inc. New York. USA. 1998 Coutinho, S. C. Nmeros Inteiros e Criptografia RSA. Instituto de Matemtica Pura e Aplicada IMPA & Sociedade Brasileira de Matemtica SBM. Rio de Janeiro. Brasil. 1997 Hefez, Abramo Elementos de Aritmtica. Sociedade Brasileira de Matemtica. Rio de Janeiro. Brasil. 2005 Perelmn, Ya I. Problemas y Experimentos Recreativos. Editorial MIR. Mosc. URSS. 1975

26

Aula 3 O algoritmo da diviso


Apresentao
Nesta aula, estudaremos o Algoritmo da Diviso, proposto por Euclides, e seus usos nas questes de divisibilidade dos nmeros inteiros. O que nesta aula voc vai aprender no so somente fatos, imaginamos poder trilhar, juntos, caminhos amigveis para voc aprender a Teoria dos Nmeros. Para obter sucesso, a partir desta aula, voc tem que ler e compreender o contedo. Leia devagar, gastando alguns minutos numa nica linha, se isso for necessrio. No se impaciente. Avance quando voc se achar preparado.

Objetivos
Fazer uso do Algoritmo da Diviso na soluo de problemas envolvendo nmeros inteiros. Compreender que o uso do Algoritmo da Diviso permite expressar um nmero inteiro de forma unvoca a partir da sua diviso por outro nmero.

Noes bsicas
Observe que o inteiro 5 no divide o inteiro 42 e que 7 no divide 12. Por outro lado, podemos escrever 42 = 8.5 + 2 e 12 = 1.7 + 5. fcil criar vrios exemplos de dois nmeros inteiros onde um nmero no divide outro. Assim, conclumos que a diviso bastante restritiva no conjunto dos nmeros inteiros. Existe um processo de diviso de um nmero natural qualquer por outro, que amplia o conceito de divisibilidade e pelo qual se determina o quociente e o resto da diviso, sendo eles determinados unicamente. Esse processo conhecido como Algoritmo da Diviso (apresentado por Euclides), e se estende de modo natural para o conjunto de todos os inteiros, com a restrio do divisor ser diferente de zero (ou divisor positivo, para facilitar).

O Algoritmo da Diviso
Sabemos que 5 no divide 38, mas, no entanto, podemos escrever 38 = 7.5 + 3. Nesse caso, 7 o quociente e 3 o resto da diviso de 38 por 5. Outro exemplo 42 = 5.8 + 2, nesse caso, 5 o quociente e 2 o resto da diviso de 42 por 8. Outro exemplo, -26 = (-7).4 + 2, nesse caso, -7 o quociente e 2 o resto da diviso de -26 por 4. Agora, observe que, como 5 divide 35, podemos escrever 35 = 7.5 + 0; nesse caso, 7 o quociente e 0 o resto da diviso de 35 por 5. nesse sentido que dizemos que o Algoritmo de Euclides amplia o conceito de divisibilidade. De uma maneira geral temos:

27

Algoritmo da Diviso
Dados dois nmeros inteiros n e d, com d > 0, existem dois nmeros inteiros qer tais que nb = q.d + r, com 0 r < d. Alm disso, os nmeros q e r so nicos, para cada par de nmeros n e d dados. Antes de demonstrar o Algoritmo da Diviso, de Euclides, vamos apresentar uma propriedade muito interessante do conjunto dos nmeros inteiros, que utilizaremos na demonstrao do Algoritmo de Euclides. Trata-se do Princpio do Menor Inteiro (tambm chamado de Princpio da Boa Ordenao ou Princpio da Boa Ordem).

Princpio do Menor Inteiro


Todo subconjunto no nulo de nmeros inteiros positivos possui um menor elemento. O Princpio do Menor Inteiro aceito sem demonstrao porque ele intuitivo e se comprova facilmente com qualquer exemplo. Alm disso, pode-se mostrar que o Princpio do Menor Inteiro equivalente ao Princpio da Induo. Se A = {5, 7, 9, ..., 99}, ento, o menor elemento de A o 5. Se B = {x Z x = n2 + 1, onde n um inteiro qualquer}, ento o menor elemento de B 1 (que ocorre quando n = 0). Voc pode criar vrios outros exemplos para verificar a veracidade do Princpio do Menor Inteiro. Agora, vamos apresentar duas demonstraes do Algoritmo da Diviso. interessante observar que a demonstrao do Algoritmo da Diviso tem duas partes distintas: uma a existncia e a outra a unicidade.

Demonstrao 1 (i) Existncia


a) Suponha inicialmente que n um nmero natural. Vamos usar o Princpio da Induo, estudado na aula 1 Noes sobre o processo e o mtodo de induo fazendo induo sobre n. Para n = 1, tem-se q = 1 e r = 0 no caso d = 1, pois 1 = 1.1+ 0. No caso d > 1, tem-se q = 0 e r = 1, uma vez que 1 = 0.d + 1. Suponhamos o algoritmo vlido para n = k, isto , k = q.d + r, com 0 r < d. Desse modo, k + 1 = q.d + r + 1. Como 0 r d 1, analisemos os casos 0 r d 2 e r = d 1 separadamente. Se for r = d 1, ento, r + 1 = d, o que d k + 1 = q.d + d = (q + 1).d. Logo, k + 1 dividido por d tem q + 1 como quociente e resto zero. Agora, se 0 r d 2, ento, 1 r + 1 d 1. Desse modo, fica k + 1 = q.d + (r + 1), onde 1 (r + 1) d 1. Portanto, o algoritmo tambm vlido para n = k + 1. Pelo Princpio da Induo, o algoritmo vlido para todo nmero natural n. b) Ora, de n = q.d + r, com 0 r < d, segue que: Se r = 0, temos: -n = (-q)d + 0. Caso contrrio, - n = (-q).d r = (-q).d d + d r = (-q 1).d + (d r). Como 0 r < d, ento, 0 < d r < d. Desse modo, o algoritmo vlido para todo inteiro negativo.

28

c) Para n = 0, tem-se zero como quociente e resto, pois 0 = 0.d + 0. Concluso: de a), b) e c), segue que o Algoritmo da Diviso vlido para todo inteiro n, o que conclui a prova.

(ii) Unicidade Resta mostrar que os nmeros inteiros q e r so nicos, para cada par de nmeros inteiros n, d dado. Vamos supor que existam dois inteiros u e v, tais que n = q.d + r e n = u.d + v, com 0 r < d e 0 v < d. Vamos supor que u < q. Logo, u + 1 q, pois u e q so inteiros. Podemos concluir que: r = n q.d n (u + 1).d = n u.d d = v d < 0. Contradio, pois r 0. O mesmo raciocnio pode ser usado para o caso em que u > q, obtendo uma contradio. Pela propriedade da Tricotomia (vista na aula 1), s resta u = q. Portanto, temos n = q.d + r e n = q.d + v, o que implica r = v. Logo, a unicidade est provada.

Demonstrao 2 (i) Existncia Suponhamos por absurdo que o Algoritmo da Diviso no vlido. Isto , existe n Z e d N tais que, para todo f Z, se r = n f.d, ento, r < 0 ou r d. Essa hiptese nos remete a considerar o conjunto S = {n fd n fd 0}. Mostremos que S no o conjunto vazio. De fato, se n 0, tomando f = -1, fica n f.d = n + d d. Isso nos diz que n + d S. Se, por outro lado, n < 0, tomando f = n 1, temos que n f.d = n (n 1).d = n nd + d > d, pois n nd > 0. Logo, n (n 1).d S. Sendo S e limitado inferiormente por d, pelo Princpio do Menor Inteiro, S possui um menor elemento; n qo .d. Por outro lado, n (qo + 1) .d = n qo .d d < n qo .d. Como n qo .d o menor elemento de S, ento, n (qo + 1) .d no pertence a S. Pela hiptese inicial, dever ser n (qo + 1) .d < 0. Isso implica que n qo .d < d, o que uma contradio, pois n qo .d d. Portanto, a hiptese inicial falsa, o que acarreta que o Algoritmo da Diviso vlido para todo nmero n Z e todo d N. (ii) Unicidade Veja a demonstrao anterior.

No Algoritmo da Diviso, b = q.a + r, a dito divisor, b o dividendo, q o quociente e r o resto. Assim, podemos escrever: dividendo = quociente divisor + resto. Certamente, voc j foi apresentado a esse algoritmo com essa verso simplificada. Uma das mais importantes conseqncias do Algoritmo da Diviso que qualquer inteiro m ou divisvel por a (sendo a inteiro maior do que 1) ou deixa resto 1 ou 2 ou 3 ou... ou a 1 na diviso por a. Logo, no caso em que a = 2, conclui-se que todo inteiro ou da forma 2q ou da forma 2q + 1, sendo q um nmero inteiro, pois os possveis restos na diviso por 2 so: 0 ou 1.

29

Quando um inteiro b da forma 2q, com q um nmero inteiro (ou seja, deixa resto zero na diviso por 2), dizemos que b um nmero par. Quando um inteiro b da forma 2q + 1, com q um nmero inteiro (ou seja, deixa resto 1 na diviso por 2), dizemos que b um nmero mpar. Usando a diviso por 3, podemos concluir que todo nmero inteiro m da forma 3q ou 3q + 1 ou 3q + 2 (ou seja, ou deixa resto zero ou deixa resto 1 ou deixa resto 2, quando dividido por 3).

Exemplo 1 O quadrado de qualquer inteiro ou da forma 4q ou 4q + 1, onde q um inteiro. Soluo


Dado um inteiro b qualquer, temos que: ou b par ou b mpar. Logo, ou b = 2k ou b = 2k + 1, onde k um inteiro. Portanto, ou b2 = 4k2 = 4q, onde q = k2, ou b2 = 4k2 + 4k + 1 = 4(k2 + k) + 1 = = 4q + 1, onde q = k2 + k Z. Portanto, o quadrado de qualquer inteiro da forma 4q ou 4q + 1, onde q um inteiro.

Exemplo 2 A soma dos quadrados de dois inteiros da forma 4q ou 4q + 1 ou 4q + 2, onde q um inteiro. Soluo
Pelo exemplo 1, dados dois inteiros a e b, seus quadrados, a2 e b2, s podem ser da forma 4q ou 4q + 1, onde q um inteiro. Portanto, a soma dos quadrados, a2 + b2, s pode ser da forma: 4m + 4n = 4q, com m + n = q, ou 4m + (4n + 1) = 4q + 1, com m + n = q ou (4m + 1) + (4n + 1) = 4q + 2, com m + n = q.

Exerccio 1
Se n um inteiro positivo qualquer, verifique que o nmero

n( n + 1) um inteiro. 2

Exemplo 3 Prove que nenhum inteiro da forma 4q + 3, onde q um inteiro, pode ser escrito como soma de dois quadrados. Soluo
uma conseqncia imediata do exemplo 2, pois a soma dos quadrados de dois inteiros tem que ser da forma 4q ou 4q + 1 ou 4q + 2, onde q um inteiro.

Exemplo 4 Nenhum nmero da lista abaixo um quadrado perfeito, isto , um quadrado de um nmero inteiro:
11, 111, 1111, 11111, 111111, 1111111, ...........

30

Soluo
Basta observar que todo nmero da lista da forma 4q + 3, com q Z. Veja, por exemplo, que: 11 = 4.2 + 3; 111 = 4.27 + 3; 1111 = 4.277 + 3 etc. Mas, de acordo com o exemplo 1, o quadrado de qualquer nmero inteiro da forma 4q ou 4q + 1, onde q um inteiro. Logo, nenhum nmero da lista dada um quadrado perfeito.

Exerccio 2 1) Encontre o quociente e o resto na diviso de: (a) 227 por 143 (b) 1479 por 272

(c) 2378 por 1769

2) Quantos inteiros entre 100 e 200 deixam resto 5 quando divididos por 7? Observe que no enunciado do Algoritmo da Diviso no exigido que o dividendo b seja um nmero positivo, mas que apenas o divisor seja. Por exemplo, b = -123 e a = 15, ento, q = -9 e r = 12. Ou seja, -123 = (-9).15 + 12.

Exemplo 5 Os nmeros inteiros positivos so arrumados em 7 colunas conforme a disposio a seguir.


1 2 8 9 15 16 22 23 ........ ........ ........ ........ 3 10 17 24 ....... ....... 4 11 18 25 ....... ....... 5 12 19 26 ....... ....... 6 13 20 27 ....... ....... 7 14 21 28 ....... .......

Qual a linha e coluna em que se encontra o nmero 1500? Soluo Observe na disposio anterior que, na primeira coluna, contando da esquerda para a direita, esto colocados todos os nmeros inteiros positivos que deixam resto 1 quando divididos por 7. Na segunda coluna, esto os nmeros inteiros positivos que deixam resto 2 quando divididos por 7, e, assim por diante, at a stima coluna, onde esto os nmeros inteiros positivos que deixam resto zero quando divididos por 7. Agora, para identificar a coluna em que se encontra o nmero 1500, basta calcular o resto da diviso de 1500 por 7. Como 1500 = 214. 7 + 2, temos que 1500 encontra-se na segunda coluna, contado da esquerda para a direita

Exerccios

31

1) Liste todos os inteiros entre 0 e 50 que podem ser expressos na forma 8n para algum inteiro n. 2) Liste todos os inteiros entre 0 e 50 que podem ser expressos na forma 8n + 3 para algum inteiro n. 3) Liste todos inteiros entre 0 e 50 que podem ser escritos na forma 8n + 7 para algum inteiro n. 4) Ache o menor mltiplo positivo de 5 que deixa resto 2 quando dividido por 3 e 4. 5) Use o Algoritmo da Diviso para concluir que: (a) O cubo de qualquer inteiro tem uma das formas seguintes: 9q, 9q + 1 ou 9q + 8. (b) Nenhum nmero da forma 3q2 1 um quadrado perfeito. 6) Dado um inteiro qualquer n, conclua, usando o Algoritmo da Diviso, que o nmero n(n + 1)(2n + 1) um inteiro. 6 7) Encontre o quociente e o resto na diviso de: (a) 22009 por 3 (b) 152008 por 7 (c) 20082009 por 1601

8) Quantos inteiros entre 100 e 200 deixam resto 5 quando divididos por 7? 9) Quantos inteiros entre 0 e 200 deixam resto 4 quando divididos por 6? 10) Encontre o maior nmero inteiro de trs dgitos que deixa resto 5 quando dividido por 8. 11) Encontre o menor nmero natural mltiplo de 7 que deixa resto 1 na diviso por 3 e 4. 12) Arrumam-se os nmeros inteiros positivos mpares 1, 3, 5, 7, 9, 11, ......., em cinco colunas, conforme a disposio a seguir. 1 15 13 17 31 29 33 47 45 49 63 61 ......... ......... ......... ......... 3 11 19 27 35 43 51 59 ......... ......... 5 9 21 25 37 41 53 57 ......... ......... 7 23 39 55 ........ ........

Contando da esquerda para a direita, qual a coluna em que se encontra o nmero 2009?

Sugesto Observe que todos os nmeros positivos mpares entre 0 e 8 esto na primeira fila e, de um modo geral, todos os nmeros inteiros positivos mpares entre 8(n 1) e 8n esto na

32

n-sima fila. Agora, verifique se o nmero 2009 est numa fila par ou mpar e se os nmeros crescem ou decrescem naquela fila. 13) Quais so os nmeros que, quando divididos por 7, deixam resto igual: (a) metade do quociente? (b) ao quociente? (c) ao dobro do quociente? (d) ao triplo do quociente? 14) O resto da diviso de um nmero inteiro n por 15 5. Qual o resto da diviso de n por 7? 15) Como adivinhar o dia e ms do seu nascimento ? Escreva numa folha de papel o dia do ms em que voc nasceu e faa as operaes seguintes: duplique o nmero escrito; multiplique por 10 o nmero obtido; some 73 ao produto; multiplique por 5 a soma; Ao total adicione, o nmero de ordem do ms em que voc nasceu (por exemplo, se voc nasceu em agosto esse nmero 8); diga o resultado final de todas as operaes. Com esse nmero posso dizer exatamente o dia e o ms em que voc nasceu. Como posso fazer isso?

Resumo Nesta aula, estudamos o importante Algoritmo da Diviso e vimos como ele indispensvel no estudo da divisibilidade dos nmeros inteiros.

Problemas Suplementares
Problema 1 Trs escolas pediram a uma mesma livraria as respectivas quantidades de certo livro didtico: 90, 126 1 198. A livraria pretende entregar esses livros, numa nica viagem, por meio de pacotes iguais, isto , todos com a mesma quantidade de livros. Qual o nmero mnimo de pacotes que essa livraria conseguir formar para atender os tais pedidos? Problema 2 Num luminoso de rua, uma lmpada amarela pisca de 6 em 6 segundos e uma lmpada vermelha pisca de 9 em 9 segundos. Se s 8 horas da noite elas piscam juntas, desse instante at as 11 horas da mesma noite, quantas vezes elas piscam simultaneamente? Problema 3 Sabe-se que m, n e p so trs nmeros inteiros positivos, tais que: m < n e m.n + p.n = 58. Descubra que nmero o p.

Problema 4

33

Joozinho exagerou na baguna na sala de aula e o professor, como forma de castigo, mandou que ele resolvesse o seguinte problema: Encontre um nmero natural, maior do que 100, cujo quadrado ao ser dividido por 3 deixa resto 2. Qual foi a resposta de Joozinho?

Problema 5 Considere todos os nmeros naturais do conjunto {500, 501, 502, 503, ..., 1999}. Qual a soma de todos os restos das divises, por 5, de todos os nmeros do conjunto dado? Problema 6 No subconjunto de nmeros inteiros S = {1881, 1882, 1883, ..., 2009}, quantos so os nmeros divisveis por 117? Problema 7 Mostre que, para quaisquer que sejam os inteiros m e n, o produto (36a + b).(a + 36b) no pode ser uma potncia de 2.

Referncias
BURTON, David M. Elementary number theory. New York: McGraw-Hill, 1998. COUTINHO, S. C. Nmeros inteiros e criptografia RSA. Rio de Janeiro: Instituto de Matemtica Pura e Aplicada IMPA/ Sociedade Brasileira de Matemtica SBM, 1997. HEFEZ, Abramo. Elementos de aritmtica. Rio de Janeiro: Sociedade Brasileira de Matemtica, 2005. NERY, Chico. Para gostar de Matemtica, Vols 1 e 2. Ribeiro Preto. Editora So Francisco. 2008 PERELMN, Ya I. Problemas y experimentos recreativos. Mosc: Editorial MIR, 1975.

34

Aula 4 - O teorema fundamental da aritmtica


Apresentao
Na aula 2 (Divisibilidade), voc teve o primeiro contato com os nmeros primos. Eles constituem um dos objetos mais fundamentais da Matemtica. O aspecto de indivisibilidade que carrega consigo cada nmero primo, tem despertado o interesse e a admirao dos matemticos ao longo dos sculos. A importncia dos primos se deve capacidade que eles tm de gerar todos os nmeros inteiros, veremos adiante quando abordarmos o Teorema Fundamental da Aritmtica. Tal importncia tem motivado o estudo dos nmeros primos desde a antiguidade grega at os nossos dias. Tente entender tudo que est sendo explicado na aula. Estude com caneta e papel ao lado. Leia com ateno. Se for preciso, leia vrias vezes uma linha ou um pargrafo. Seja paciente e procure ter certeza que voc entendeu o que (e por que) est fazendo. importante que tente resolver cada um dos problemas que aparece no final da aula, na auto-avaliao, pois ser um teste para avaliar seu entendimento acerca do contedo apresentado.

Objetivos

Decompor um nmero inteiro positivo em seus fatores primos. Usar a decomposio de dois nmeros inteiros positivos em fatores primos para encontrar o Mximo Divisor Comum e o Mnimo Mltiplo Comum desses dois nmeros. Encontrar o nmero de divisores de um nmero inteiro positivo.

O papel fundamental dos nmeros primos


Desde a Grcia antiga, os qumicos se esforaram para identificar os elementos bsicos da natureza. Tal esforo culminou com a elaborao da tabela peridica de Dimitri Mendeleev (1834 -1907), professor da Universidade de So Petersburgo, na Rssia. Cada uma das molculas do mundo fsico pode ser decomposta por tomos da tabela peridica de elementos qumicos. Para os matemticos, os nmeros primos so os elementos de nossa tabela peridica. Mas, apesar do sucesso que os gregos antigos tiveram na identificao de blocos de nmeros que permitem um amplo domnio da aritmtica, os matemticos tm dificuldade de entender a tabela dos nmeros primos. O matemtico que primeiro construiu uma tabela de primos foi Eratstenes, que foi diretor da biblioteca de Alexandria no sculo III a. C. A lista de matemticos que se esforaram para entender a tabela dos nmeros primos imensa, contando com nomes como Euclides, Fibonacci, Gauss, Euler, Goldbach, Riemann, Fourier, Jacobi, Legendre, Cauchy, Hilbert, Hardy, Littlewood, Ramanujan, Minkowski, Landau etc. At os dias de hoje ainda se procura entender a tabela dos primos. Em 1970, , trs pesquisadores que trabalhavam no Massachussets Institute of Tecnology MIT, nos Estados Unidos, Ron Rivest, Adi Shamir e Leonard Adleman, explorando os

35

trabalhos de Pierre de Fermat, feitos no sculo XVII, descobriram um modo de usar os nmeros primos para proteger nossos cartes de crditos, quando fazemos compras pela Internet. Sem o poder dos nmeros primos, esse tipo de comrcio jamais poderia existir. Os trs pesquisadores citados usaram um processo para manter o nmero de nossos cartes de crdito em segurana, usando nmeros primos com 100 dgitos. O sistema inventado se chama RSA, sendo R a primeira letra do segundo nome do primeiro cientista, S a primeira letra do segundo nome do segundo cientista e A a primeira letra do segundo nome do terceiro. Hoje em dia, para aumentar a segurana, j se usa nmeros primos com 600 dgitos. Eratstenes, astrnomo e matemtico grego que foi diretor da biblioteca de Alexandria na poca de Ptolomeu III, inventou uma tcnica para achar todos os primos menores do que ou iguais a um dado nmero n, que ficou conhecida como Crivo de Eratstenes. A tcnica consistia em listar todos os nmeros de 2 at n; em seguida, riscar todos os mltiplos de 2, maiores do que 2; logo aps, riscar todos os mltiplos de 3, maiores do que 3; depois, riscar todos os mltiplos de 5, maiores do que 5, e assim por diante. Eratstenes sabia que um dos fatores primos de um nmero composto era menor do que ou igual raiz quadrada do nmero. Assim, ele continuaria o processo at que o maior nmero primo menor do que ou igual a n fosse atingido. Nessa altura, todos os nmeros compostos de 2 at n j teriam sido riscados, restando somente os nmeros primos de 2 at n. Eratstenes tambm foi atleta, poeta, filsofo e historiador. Como atleta, fez sucesso nos III Jogos Olmpicos, da Grcia antiga. Agora, vamos responder completamente pergunta feita na aula 2:

Existem quantos nmeros primos?


Conforme afirmamos antes, Euclides, em sua obra Os Elementos, demonstrou o seguinte:

Teorema 1 Existem infinitos nmeros primos. Demonstrao 1


Antes de dar a demonstrao de Euclides, vamos entender a sua idia. O que Euclides fez foi construir um nmero que no pudesse ser gerado por qualquer lista finita de primos que lhe fosse apresentada. Por exemplo, considere a lista dos seis primeiros primos: 2, 3, 5, 7, 11 e 13. Euclides multiplicou-os, obtendo o nmero 2 x 3 x 5 x 7 x 11x 13 = 30030, que um mltiplo dos seis primeiros primos. A seguir, usando sua genialidade, somou 1 ao produto, obtendo 30031, que no divisvel por nenhum dos cinco primos da lista, pois a diviso de 30031 por qualquer um dos cinco primos deixa resto 1. Euclides sabia que o nmero criado poderia no ser primo, mas se no fosse, deveria ser divisvel por um primo que no estava na lista dada. Assim, Euclides disse que qualquer que fosse a lista finita dos primos, era possvel criar um nmero que s poderia ser formado a partir de primos que no estavam na lista dada. O nmero do exemplo, 2 x 3 x 5 x 7 x 11x 13 + 1 = 30031 = 59 x 509, portanto, no primo. Agora que voc j entendeu a idia de Euclides, vamos fazer a demonstrao do Teorema 1. Vamos imaginar que a quantidade de primos no seja infinita. Significa que, ao listar todos os primos, essa lista terminaria em algum primo, ou seja, teria um primeiro primo, um segundo, ..., e um ltimo. Assim, vamos supor que existem somente n nmeros primos: p1 , p 2 , p3 , p 4 ,....., p n

36

de maneira tal que eles estejam ordenados em ordem crescente: p1 < p 2 < p3 < p 4 < ..... < p n . Isso seria o mesmo que colocar 2 < 3 < 5 < 7 < 11 < 13 < 17 < 23 < ..... < p n , sendo p n o maior de todos. Agora, vamos tomar o seguinte nmero, M, construdo a partir de todos os n primos: M = p1 . p 2 . p3 . p 4 ....... p n + 1 . Observe que M um inteiro positivo maior do que qualquer um dos primos. De fato, M > 2pn + 1, pois 2 o menor nmero primo. Logo, M maior do que pn, o maior de todos os primos e, portanto, maior do que todos os primos p1 , p 2 , p3 , p 4 ,....., p n . Logo, por hiptese, M no primo. Assim, M divisvel por algum primo. Mas, esse primo deveria ser um dos primos p1 , p 2 , p3 , p 4 ,....., p n . Escolha um desses, digamos pi, para ser o divisor de M. Nesse caso, teramos nmero M p1 . p 2 . p3 . p 4 ... pi .... p n = 1 e como pi divide M e pi divide p1 . p 2 . p3 . p 4 ... pi .... p n , segue que pi divide 1, o que implica que pi = 1, o que uma contradio.

Demonstrao 2 Suponha que a quantidade dos primos seja finita e p1 = 2 < p2 = 3 < p3 = 5 < p4 = 7 < .... < pn os primos. Considere m o menor nmero inteiro positivo maior do que pn. Agora, considere o nmero inteiro positivo m! + 1. Como m! + 1 > pn, ele no pode ser primo. Logo, m! + 1 divisvel por um dos primos j enumerados anteriormente. Por outro lado, o produto 1.2.3.....m = m! tem esse primo como um de seus fatores. Logo, esse primo divide 1, o que constitui uma contradio.

Exemplo 1 Diga, justificando, se o nmero M = 2.3.5.7 + 1 primo. Soluo


M = 2.3.5.7 + 1 = 211. Voc viu, no exemplo 5 da aula 2 (Divisibilidade) que se um nmero inteiro positivo n no primo, ento, n possui um divisor menor do que ou igual a n . Como M = 211 = 14,525839... , basta procurar um divisor primo menor do que ou igual a 13. Como 2, 3, 5, 7, 11 e 13 no dividem M, ento, M um nmero primo.

Exerccio 1 Qual o maior divisor primo do nmero inteiro 1 + 2 + 3 + 4 + .... + 50? O teorema fundamental da aritmtica
O Teorema Fundamental da Aritmtica coloca em evidncia o papel dos nmeros primos na estrutura dos inteiros. Ele nos assegura que um nmero pode ser expresso como um produto de nmeros primos de modo nico, a menos da ordem desses fatores primos. Considere o nmero 90. Esse nmero s pode ser expresso como produto de primos usando somente os primos 2, 3 e 5, a menos da ordem. De fato, podemos escrever:

37

90 = 2 x 3 x 3 x 5 = 3 x 3 x 2 x 5 = 2 x 5 x 3 x 3 = 2 x3 x 5 x 3 = = 5 x 3 x 2 x 3 = 3 x 2 x 3 x 5 = 3 x 2 x 5 x3 = 3 x 5 x 2 x 3 etc. Resumidamente, 90 = 2 x 32 x 5. De modo anlogo, considere o nmero 24. Ele s pode ser expresso como produto de primos usando somente o 2 e o 3, a menos da ordem: 24 = 2 x 2 x 2 x 3 = 2 x 2 x 3 x 2 = 2 x 3 x 2 x 2 = 3 x 2 x 2 x 2. Em resumo, 24 = 23 x 3

Teorema 2 ( Teorema Fundamental da Aritmtica) Todo nmero inteiro maior do que 1 se escreve como o produto nico de nmeros primos, a menos da ordem desses fatores primos. Demonstrao 3
Vamos imaginar que o Teorema Fundamental da Aritmtica no seja verdadeiro. Desse modo, existiriam alguns (ou algum) nmeros inteiros maiores do que 1 que no se escreveriam com o produto de primos, de modo nico. Seja n o menor inteiro maior do que 1 para o qual o teorema no fosse verdadeiro. O nmero n, ele prprio, no pode ser primo, pois, nesse caso, ele seria a sua prpria decomposio em fatores primos (um s fator). Portanto, n seria composto e poderamos escrever n = ab, com 0 < a < n e 0 < b < n. Nesse caso, a e b podem ser decompostos em produtos de primos, pois ambos so menores do que n, que , por hiptese, o menor nmero que no pode ser decomposto como produto de primos. Logo, teramos: a = p1 . p 2 . p3 . p 4 ....... p n , onde necessariamente distintos, e b = q1 .q 2 .q3 .q 4 .......q m , onde necessariamente distintos. p1 , p 2 , p3 , p 4 ,....., p n q1 , q 2 , q3 , q 4 ,......, q m so nmeros primos no so nmeros primos no

Portanto, n = ab = p1 . p 2 . p3 . p 4 ....... p n .q1 .q 2 .q3 .q 4 .......q m , e teramos n escrito como um produto de primos. Isso uma contradio com a escolha de n.. A contradio vem do fato de assumirmos que existia algum inteiro maior do que 1 para o qual o teorema no fosse vlido. Logo, todo nmero inteiro maior do que 1 se escreve como produto de nmeros primos. Resta mostrar que os primos que comparecem na decomposio do nmero n so nicos, a menos da ordem com que comparecem nessa decomposio. Vamos supor que a decomposio no seja nica. Seja n o menor inteiro maior do que 1 que no se escreve de forma nica como produto de primos. Ou seja, existem nmeros primos p1 , p 2 , p3 , p 4 ,....., p n e q1 , q 2 , q3 , q 4 ,....., q m , tais que n = p1 . p 2 . p3 . p 4 ....... p n e n = q1 .q 2 .q3 .q 4 .......q m , com q1 q 2 ... q m . Logo, p1 . p 2 . p3 . p 4 ....... p n = q1 .q 2 .q3 .q 4 .......q m . Se algum pi for igual a algum qj, poderamos dividir ambos os lados da igualdade por pi, obtendo duas fatoraes iguais e contradizendo a minimalidade de n . Assim, todos os pi qj, para todo i {1, 2, 3, ...n} e todo j {1, 2, 3, ..., m}. Podemos supor que p1 < q1. Chamando Q = q 2 .q3 .q 4 .......q m , tem-se p1Q < q1Q = n , pois p1 < q1. Logo, 0 < n p1Q < n . Como n suposto ter duas decomposies distintas, o nmero A seguinte pode ser escrito de duas maneiras, a saber: A = q1 q 2 ....q m p1Q = q1Q p1Q = (q1 p1 )Q e como

38

q1 .q 2 .q3 .q 4 .......q m = n = p1 . p 2 . p3 . p 4 ....... p n , ento, tambm:

A = p1 p 2 ... p n p1Q = p1 ( p 2 p3 .... p n Q) .


Como p1 < q1 e p1 q j , j = 1,2,3,...., m , ento, Q no tem p1 como fator primo. Tambm
q1 p1 no tem p1 como primo, do contrrio p1 dividiria q1 , o que uma contradio, pois os dois so nmeros primos distintos. Desse modo, o nmero 0 < A < n admite na segunda decomposio anterior p1 como fator primo e na primeira decomposio no tem, contrariando a escolha do nmero n.

Observe que o Teorema 2 fundamental para a Aritmtica justamente porque assegura que todo nmero inteiro maior do que 1 se escreve, de modo nico, como produto de primos, a menos da ordem desses fatores primos.

Exemplo 2 Decomponha em fatores primos o nmero inteiro 120. Soluo


O nmero dado se escreve (ou se decompe) como produto de primos da seguinte maneira: 120 = 2.2.2.3.5 . Na prtica, escrevemos: 120 = 23.3.5, onde 2 < 3 < 5.

Exemplo 3 Decomponha em fatores primos o nmero inteiro 4.667.544. Soluo


O nmero dado se escreve (ou se decompe) como produto de primos da seguinte maneira: 4.667.544 = 2.2.2.3.3.3.3.3.7.7.7.7. Na prtica, escrevemos: 4.667.544 = 23.34.74, onde 2 < 3 < 7.

Exerccio 2 Escreva 63 com produto de nmeros primos.


De um modo geral, como conseqncia do Teorema Fundamental da Aritmtica, temos que, se n um inteiro maior do que 1, que se decompe em fatores primos distintos p1 , p 2 , p3 , p 4 ,....., p k , com p1 aparecendo a1 vezes, p 2 aparecendo a2 vezes,......., p k aparecendo ak vezes, ento, podemos escrever n, de forma nica, como
a a n = p1a1 . p 2 2 . p3 3 ... p kak , onde

p1 < p 2 < p3 < .... < p k .

Exemplo 4 Escreva cada um dos nmeros 360, 540 e 700, de forma nica, como produto de primos a a p1a1 . p 2 2 . p3 3 ... p kak , onde p1 , p 2 , p3 , p 4 ,....., p n so nmeros primos com p1 < p 2 < p3 < .... < p k . Soluo

39

360 = 2.2.2.3.3.5. = 2 3.3 2.5 ; 540 = 2.2.3.3.3..5 = 2 2.33.5 ; 700 = 2.2.5.5.7 = 2 2.5 2.7

Exerccio 3 a a Escreva 80 como produto de nmeros primos p1a1 . p 2 2 . p3 3 ... p kak , onde p1 , p 2 , p3 , p 4 ,....., p k so nmeros primos com p1 < p 2 < p3 < .... < p k .
Na aula 5 O Mximo Divisor Comum, o Mnimo Mltiplo Comum e as equaes diofantinas lineares estudaremos o conceito e as propriedades do Mximo Divisor Comum de dois inteiros. Veremos que o Mximo Divisor Comum de dois inteiros m e n, que notaremos por MDC(m, n), um inteiro d maior do que 1, tal que: (i) d divide m e d divide n (Isto , d um divisor comum); (ii) se existe um inteiro c maior do que 1, tal que c divide m e c divide n, ento, c d (isto , d o maior divisor comum). Podemos usar a decomposio de um nmero inteiro maior do que 1 como produto de nmeros primos para encontrar o Mximo Divisor Comum de dois inteiros positivos. A ttulo de ilustrao, consideremos os nmeros 1890 e 360 e suas decomposies em fatores primos 1990 = 2 . 3 3 .5.7 360 = 2 3 .3 2 .5

Observamos que os fatores primos comuns na decomposio dos dois nmeros so 2 , 3 e 5. Para encontrar o MDC(1890, 360), basta agora multiplicar os fatores primos comuns elevados aos menores expoentes, isto , MDC(1890, 360) = 2.3 2 .5 = 90. Outro exemplo, para calcular MDC(150, 280), decompomos os dois nmeros em fatores primos: 150 = 2.3.5 2 e 280 = 2 3.5.7 ,

e consideramos o produto dos fatores primos comuns elevados aos menores expoentes: MDC(150, 280) = 2.5 = 10. De um modo geral, se MDC(m, n) = d, ento, na decomposio de d em fatores primos comparecem os fatores primos comuns aos nmeros inteiros m e n. Como d o maior divisor comum a m e n, ento, cada fator primo comum comparece com o menor expoente.

Exemplo 5 Use o Teorema Fundamental da Aritmtica para calcular o MDC dos nmeros: 68 e 120. Soluo
Decompondo ambos os nmeros dados, obtemos: 68 = 22 x 17 e 120 = 23 x 3 x 5. Logo, o nico primo comum na decomposio dos dois nmeros dados 2 e o menor expoente 2. Portanto, MDC(68, 120) = 22 = 4.

40

Usualmente, para decompor um nmero inteiro positivo n em fatores primos, usamos o algoritmo para fatores primos, que a diviso por todos os primos que dividem n. No algoritmo, aparece o nmero, seus divisores e o respectivo quociente. Os divisores esto na coluna direita, em ordem decrescente do nmero e na coluna do nmero esto os respectivos quocientes. Por exemplo, a decomposio em fatores primos de 12 feita do modo seguinte: 12 2 6 2 3 3 1

que significa 12 = 2.2.3 = 22.3.

Veja a decomposio de 360 em fatores primos: 360 2 180 2 90 2 45 3 15 3 5 5 1

que significa 360 = 2.2.2.3.3.5 = 23.32.5.

Exerccio 4 Use o Teorema Fundamental da Aritmtica para calcular o MDC dos nmeros: (a) 258 e 828 (b) 60 e 144

Uma pergunta: quantos so os divisores positivos de um nmero natural n?


Para facilitar o entendimento, vamos particularizar a pergunta: quantos so os divisores positivos de 12? Para responder, precisamos descrever um divisor positivo de 12. Para isso, vamos utilizar o Teorema Fundamental da Aritmtica no caso em que n = 12. Isto , 12 = 2 2.3 . Um divisor positivo de 12 um nmero d para o qual s comparece na sua decomposio em fatores primos os nmeros primos 2 e 3. Alm disso, d s pode ter o 2 como fator primo se 2 estiver elevado a: 0,1 ou 2. Do mesmo modo, o divisor d s pode ter o fator 3 elevado a: 0 ou 1. Desse modo, a quantidade de divisores positivos de 12 3.2 = 6. Ou seja, a quantidade de divisores positivos depende dos expoentes dos primos que comparecem na decomposio do nmero em fatores primos. Assim, podemos concluir que so os seguintes os divisores positivos de 12: 1 = 20.30; 2 = 21.30; 3 = 20.31; 4 = 22.30; 6 = 21.31; 12 = 2 2.3 . De uma maneira geral, se n um nmero inteiro maior do que 1 e a3 ak a1 a2 n = p1 . p 2 . p3 ... p k , onde p1 , p 2 , p 3 , p4 ,....., p k so nmeros primos com p1 < p 2 < p3 < .... < p k , ento, pelo Princpio Multiplicativo (veja na aula 1 Aprendendo a

41

contar da disciplina Anlise Combinatria) o nmero de divisores positivos de n igual a: (a1+1).(a2 +1)(a3 +1)...(ak + 1).

Exemplo 6 Quantos so os divisores positivos de 540? Soluo


Pelo que vimos anteriormente, a decomposio de 540 em fatores primos : 540 2 270 2 135 3 45 3 Ou seja, 540 = 22.33.5 e todo divisor positivo de 540 da forma d = 2 a .3b.5 c , 15 3 5 5 1 onde a {0, 1, 2}, b {0, 1, 2, 3} e c {0 ,1}. Desse modo, usando o Princpio Multiplicativo, a quantidade de divisores positivos de 540 igual a (2 + 1).(3 + 1).(1 + 1) = 24.

Exerccio 5 Quantos divisores positivos possui o nmero 2100 ?

Exerccios
1) Com relao ao nmero 375: (a) escreva 375 como produtos de nmeros primos; (b) descreva a expresso geral dos divisores de 375; (c) quantos e quais so os divisores de 375? 2) Um nmero inteiro chamado livre de quadrado se ele no divisvel pelo quadrado de algum primo. (a) D 5 exemplos de inteiros positivos compostos, cada um sendo livre de quadrado. (b) Usando o Teorema Fundamental da Aritmtica, diga a forma geral na qual pode ser escrito cada inteiro positivo maior do que 1 que seja livre de quadrado. 3) Considere o seguinte nmero inteiro maior do que 1: z = n 4 + 4, onde n um nmero natural. Diga, justificando, se z um nmero primo.
Sugesto Pense numa fatorao do binmio n 4 + 4 como produto de dois trinmios do segundo grau.

42

4) So dados dois nmeros naturais, m e n, tais que MDC(m, n) = 15. (a) Quais so os primos comuns que fazem parte da decomposio de m e n em fatores primos? (b) Qual o menor expoente de cada um dos primos comuns que comparecem na decomposio de m e n? 5) Maria escreve num pedao de papel os cubos de trs inteiros positivos. Antnio verifica que cada um deles mltiplo de 18 e que o Mximo Divisor Comum dos cubos desses nmeros n. Encontre o menor valor possvel para n. 6) Com relao ao nmero 160, faa o que se pede. (a) Escreva 160 como produto de nmeros primos. (b) Descreva a expresso geral dos divisores de 160. (c) Quantos e quais so os divisores de 160? 7) Use o Crivo de Eratstenes para encontrar todos os primos menores do que ou iguais a 200. 8) Qual a soma de todos os inteiros positivos menores do que 100 que tm exatamente 12 divisores? 9) Sabendo que MDC(m, n) = 1 e que m e n possuem, respectivamente, 8 e 12 divisores positivos, quantos divisores possui o nmero mn? 10) Certo nmero natural K tem exatamente oito divisores, dentre os quais esto 35 e 77. Encontre o nmero K. 11) O nmero natural (2 48 - 1) divisvel por dois nmeros entre 60 e 70. Encontre esses nmeros.
Sugesto Use produtos notveis para decompor (2 48 - 1).

12) Responda as questes a seguir. (a) Usando o Teorema Fundamental da Aritmtica, prove que 2 no um nmero racional. (b) Usando o Teorema Fundamental da Aritmtica, prove que 3 no um nmero racional. (c) Adaptando seus argumentos usados nos subitens (a) e (b), possvel provar que o nmero p , com p nmero primo, no um racional? 13) Leia o enunciado a seguir e responda as questes. Escreve-se no quadro-negro os nmeros inteiros de 1 a 100. Dois jogadores disputam o seguinte jogo, em que jogam alternadamente. Uma jogada consiste em apagar um dos nmeros escritos. O jogo termina quando restam somente dois nmeros no quadro-negro. O primeiro jogador vence se a soma desses dois nmeros divisvel por 3; o segundo jogador ganha caso ocorra o contrrio. a) Quem vence: o primeiro ou o segundo jogador?

43

b) Qual a estratgia usada para vencer?


Sugesto O nmero 101 no divisvel por 3. Veja como 101 pode ajud-lo a definir uma estratgia vencedora.

Resumo Nesta aula, estudamos o papel importante dos nmeros primos dentro do conjunto dos nmeros inteiros, culminando com o Teorema Fundamental da Aritmtica, o qual diz que cada nmero natural maior do que 1 pode ser decomposto, de forma nica, como produto de fatores primos, a menos da ordem desses fatores.

Problemas Suplementares
Problema 1 O nmero 27.000.001 possui exatamente quatro fatores primos. Encontre a soma desses fatores primos. Sugesto: 27x6 + 1 = (3x2)3 + 1 = (3x2 + 1).(3x2 + 3x + 1). (3x2 - 3x + 1). Problema 2 Existem quantos inteiros positivos com 2 dgitos (na base 10), tendo um nmero par de divisores? Resp. 84 Problema 3 Encontre o menor inteiro positivo que duas vezes um quadrado perfeito e trs vezes um cubo perfeito. Resp. 648 Problema 4 Um inteiro positivo n chamado cintilante se seu menor inteiro positivo maior do que 1 igual quantidade de divisores positivos de n. O nmero 9 cintilante, pois os divisores positivo de 9 so: 1, 3 e 9, sendo 3 o menor divisor positivo maio do que 1. Quantos so os nmeros cintilantes? Resp. 3 Problema 5 Encontre todos os nmeros inteiros positivos n para os quais 3512 1 seja divisvel por 2n. Resp. 11 Problema 6 Existem quantos nmeros inteiros positivos a, com 1 a 200, tais que aa um quadrado? Resp. 107 Problema 7 Existem quantos pares de inteiros (a, b) , com 1 a b 60 , tendo a propriedade de que b divisvel por a e b + 1 divisvel por a + 1? Resp. 30

44

Problema 8 Identifique o inteiro positivo n, menor do que 1000, que possui exatamente 29 divisores positivos, sem contar ele prprio. Resp. 720 Problema 9 Cada noite, trs pessoas de um grupo de n pessoas saem juntas para jantar. Depois de certo perodo de tempo se observa que cada para de pessoas jantam junto exatamente uma vez. Demonstre que n deixa resto 1 ou 3 na diviso por 6.

Referncias

BURTON, David M. Elementary number theory. New York: McGraw-Hill, 1998. COUTINHO, S. C. Nmeros inteiros e criptografia RSA. Rio de Janeiro: Instituto de Matemtica Pura e Aplicada IMPA/ Sociedade Brasileira de Matemtica SBM, 1997. DU SAUTOY, Marcus. A msica dos nmeros primos: a histria de um problema no resolvido. Rio de Janeiro: Jorge Zahar, 2008. HEFEZ, Abramo. Elementos de aritmtica. Rio de Janeiro: Sociedade Brasileira de Matemtica, 2005. PERELMN, Ya I. Problemas y experimentos recreativos. Mosc: Editorial MIR, 1975.

45

Aula 5 O mximo divisor comum, o mnimo mltiplo comum e as equaes diofantinas lineares

Apresentao

Nesta aula, estudaremos o Mximo Divisor Comum entre dois inteiros e veremos como Euclides usou esse conceito para provar resultados da Teoria dos Nmeros. No espere encontrar um grau de dificuldade uniforme. Umas coisas so mais difceis que outras. Nelas, voc tem que gastar mais tempo. No fique impaciente. Leia vrias vezes o que julgar mais difcil. Veja cuidadosamente os exemplos, pois eles servem de referncia para voc consolidar a aprendizagem.

Objetivos

Calcular o Mximo Divisor Comum e o Mnimo Mltiplo Comum. Calcular o Mximo Divisor Comum usando o Algoritmo de Euclides. Escrever o Mximo Divisor Comum de dois nmeros inteiros m e n como combinao linear de m e n. Resolver Equaes Diofantinas Lineares como aplicaes do Mximo Divisor Comum.

O mximo divisor comum

O nmero inteiro 4 divide 12 e 4 tambm divide 20 e, alm disso, como voc pode verificar facilmente, 4 o maior nmero inteiro positivo com essa propriedade. Dizemos, ento, que 4 o Mximo Divisor Comum de 12 e 20, e notamos 4 = MDC(12, 20). De uma maneira geral, o nmero inteiro positivo d o Mximo Divisor Comum de dois nmeros inteiros a e b se d o maior nmero positivo que divide a e b ao mesmo tempo. Isso pode ser precisado nos itens abaixo: (i) d divide a e d divide b (i.e. d um divisor comum); (ii) se existe um nmero natural c tal que c divide a e c divide b, ento c d (i.e. d o maior divisor comum). Notaremos isso por: MDC(a, b) = d. Veja que, por definio, o MDC sempre um nmero positivo.

46

Observe que os inteiros 5 e 7 no possuem divisores positivos comuns, exceto o 1. Nesse caso, MDC(5, 7) = 1 e dizemos que 5 e 7 so primos entre si (ou que 5 e 7 so relativamente primos). De uma maneira geral, dois nmeros naturais a e b so primos entre si (ou relativamente primos) se MDC(a, b) = 1. Podemos usar o Algoritmo da Diviso para calcular o Mximo Divisor Comum de dois nmeros naturais. O mtodo consiste em fazer sucessivas aplicaes do Algoritmo da Diviso at obter o resto nulo. Ilustramos o mtodo com o exemplo seguinte.
Exemplo 1 Usando o Algoritmo da Diviso, calcular o MDC(360, 126). Soluo

Usando o Algoritmo da Diviso, dividimos 360 por 126 escrevendo 360 = 2 x 126 + 108 (2 o quociente e 108 o resto).

Agora, como o resto da diviso no nulo, aplicamos novamente o Algoritmo da Diviso para o divisor inicial e o resto da diviso anterior, ou seja, 126 = 1 x 108 + 18 (1 o quociente e 18 o resto).

Como o resto da ltima diviso ainda no nulo, procedemos como da vez anterior, dividindo o divisor acima pelo resto obtido, isto : 108 = 6.18 + 0 (6 o quociente e 0 o resto).

Como o resto da ltima diviso nulo, ento o divisor 18 o Mximo Divisor de 360 e 126, isto MDC(360, 126) = 18, conforme veremos na Proposio 1. Ilustramos os clculos anteriores no diagrama a seguir: 1 126 18 108 0
18

quocientes restos

Exemplo 2 Usando o Algoritmo da Diviso, calcular o MDC(32, 12) = 4. Soluo

Usando o Algoritmo da Diviso, o clculo correspondente 32 = 2 x 12 + 8 12 = 1 x 8 + 4

47

8 = 2 x 4 + 0, ou seja:
2 32 8 12 4 8 0 1 4 restos 2 quocientes

Como no Exemplo 1, conclui-se que o MDC(32, 12) = 4.

Exerccio 1 (a) Encontre o quociente q e o resto r na diviso de 72 por 20. (b) Ache o MDC(72, 20). (c) Ache o MDC(20, r). (d) Compare o MDC(72, 20) e MDC(20, r).

O procedimento descrito nos Exemplos 1 e 2 correto, pois uma aplicao repetida do fato seguinte, que verdadeiro, como veremos a seguir:
Proposio 1 Se a e b so dois nmeros naturais, de acordo com o Algoritmo da Diviso, existem naturais, q e r tais que b = q. a + r, com 0 r < a. Nesse caso, MDC(a, b) = MDC(a, r). Demonstrao

Se MDC(a,b) = d, ento da e db. Logo, a = d.u e b = d.b, com u, v nmeros inteiros. Como existem inteiros q e r tais que b = q.a + r, segue que r = b qa = d.v qdu = (v qu).d. Portanto, dr. Assim, d um divisor comum de b e r. Resta mostrar que d o maior divisor comum de b e r. Suponha que existe um inteiro positivo c tal que ca e cr. Como b = q. a + r, segue que cb . Como d = MDC(a, b) e c um divisor comum de a e b, ento d c. Portanto, d = MDC(b, r). De acordo com a Proposio 1, para encontrar o MDC(a,b) fazemos repetidas aplicaes do Algoritmo da Diviso at obter um resto zero, caso em que o penltimo resto o MDC(a, b). Outra concluso que poderemos tirar, a partir do uso do Algoritmo da Diviso, : o Mximo Divisor Comum de a e b uma combinao linear de a e b, isto : MDC(a, b) = ax + by, com x, y inteiros. Para exemplificar, quando calculamos anteriormente o MDC(360, 126) = 18, aplicamos trs vezes o Algoritmo da Diviso. Agora, explicitando o valor de 18 na segunda igualdade, obtemos: 18 = 126 + (-1).108. Isolando o valor de 108 na primeira aplicao do Algoritmo da Diviso e substituindo na igualdade acima, obtemos:

48

MDC(360, 126) = 18 = 3.126 + (-1).360 ou seja, para expressar o Mximo Divisor Comum entre dois nmeros, usamos o Algoritmo da Diviso ao inverso, isto , aplicamos o Algoritmo da Diviso at encontrar o resto 0 e vamos desfazendo at encontrar os dois nmeros. De uma maneira geral, podemos afirmar:
Teorema 1 ( Bachet Bezout) Dados a e b nmeros inteiros (ambos no nulos), ento existem inteiros x e y tais que MDC(a, b) = ax + by. Demonstrao

Para facilitar a prova, considere S a coleo de todas as combinaes lineares inteiras positivas de a e b. Ou seja, S = { ax + by x e y so inteiros e ax + by > 0}. Vamos mostrar que S no vazio e, portanto, possui um menor elemento m. Provaremos que este menor elemento m S o MDC(a, b). Se a > 0, podemos escrever a = a.1 + 0.b, o que mostra que a est em S. Por outro lado, se a < 0, podemos escrever -a = a.(-1) + b.0, o que mostra que -a est em S. Assim, podemos concluir que S um conjunto no vazio de inteiros positivos. Pelo Princpio do Menor Inteiro, S possui um menor elemento. Seja m este menor elemento de S. Logo, existem u e v inteiros tais que m = au + bv. O que vamos fazer a seguir , utilizando o Algoritmo da Diviso, dividir m por a e b e mostrar que em ambos os casos o resto zero. Ou seja, ma e mb. De fato, como a = q.m + r, onde 0 r < m., temos que r = a q.m= a q.(au +bv) = a(1 qu) + b(qv). Se r 0, temos que r est no conjunto S. Mas, r < m, e m o menor elemento de S. Contradio. Logo, r = 0. E, portanto, ma. Argumentando de maneira anloga, prova-se que mb. Agora, segue que m d = MDC(a, b). Por outro lado, como m = au + bv, com u, v inteiros e d divide a e b, segue que dm. Portanto, d m. Assim, m = d = MDC(a, b). A demonstrao do Teorema de Bachet-Bezout no fornece uma pista de como encontrar x e y inteiros para que se tenha d = ax + by. Apesar dessa deficincia, esse resultado muito til, como veremos no restante desta aula. Segue do Teorema de Bachet-Bezout, os seguintes corolrios:
Corolrio 1 Se d = MDC(a, b), ento qualquer divisor c de a e b divide d. Demonstrao

49

Se d = MDC(a, b), ento existem x e y inteiros para os quais d = ax + by. Como c divide a e c divide b, ento a = c.t e b = c.v, com t, v inteiros, tem-se d = ax + by = ctx + c.vy = c(tx + vy). Em resumo, d = ck, onde k = tx + vy, o que implica que c divide d.

Corolrio 2 Os inteiros a e b no nulos so relativamente primos se, e somente se, x e y pertencem aos inteiros tais que ax + by = 1. Demonstrao

Se a e b so dois nmeros inteiros no nulos e relativamente primos, ento MDC(a, b) =1. Pelo resultado acima, existem x e y pertencentes aos inteiros tais que ax + by = MDC(a, b) = 1. Reciprocamente, se existem x e y inteiros, com ax + by = 1, ento todo divisor d de a e b divide ax + by = 1, isto , d divide 1. Logo, d = 1. Portanto, a e b so relativamente primos.

Exemplo 3 Encontrar inteiros x e y tais que MDC(3, 20)= 3x +20y. Soluo

Usando o Algoritmo da Diviso, 20 = 6.3 + 2; 3 = 1.2 + 1 e 2 = 2.1 + 0. Logo, MDC(3, 20) = 1. Assim, existem x e y pertencentes aos inteiros tais que 3x +20y = 1. Veja que x = 7 e y = -1 verifica a igualdade.
Exerccio 2 Em cada item a seguir, encontre quatro pares de inteiros (m, n) tais que : (a) MDC(2, 3) = 2m + 3n (b) MDC(24, 51) = 24m + 51n (c) MDC(72,164) = 72m + 164n

Sendo o Mximo Divisor Comum definido para dois nmeros inteiros, entendemos MDC (a, b, c), com a, b e c nmeros inteiros, como sendo: MDC (a, MDC(b,c)) = MDC( MDC( a, b), c) = (MDC(a, c), b). Ilustrando, MDC(39, 42, 54) = MDC( MDC(39, 42), 54) = MDC(3, 54) = 3 ou MDC (39, 42, 54 ) = MDC( 39, MDC(42, 54)) = MDC (39, 6) = 3. Agora, vamos provar o seguinte resultado, citado na aula 2 (Divisibilidade):
Teorema 2 (Euclides) Se um nmero primo p divide o produto de dois nmeros naturais a.b, ento p divide a ou p divide b. Demonstrao

Vamos supor que p no divide a. Nesse caso, como p um nmero primo, p e a so relativamente primos. Logo, MDC(a, p) = 1. Portanto, existem nmeros inteiros x e y tais que MDC(a, p) = 1 = ax + py. Multiplicando ambos os lados da (ltima) igualdade por b, obtemos

50

abx + bp y = b. Como, por hiptese, p| ab, temos que: ab = pk, onde k um nmero inteiro. Assim, b = abx + bp y = pkx + bpy = p(kx + by). Portanto, p divide b. De modo anlogo, supondo que p no divide b, conclui-se que p divide a.
Exemplo 4 O nmero 7 divide o produto 84 x 12 = 1.008. Como 7 no divide 12, segue que 7, obrigatoriamente, divide 84. De fato, 84 = 12 x 7.

Observe que, se o nmero p no for primo, o Teorema 1 no verdadeiro. De fato, 12 no primo e 12 divide o produto 8 x 15. Mas, 12 no divide 8 nem divide 15. Ou seja, no Teorema 1, a hiptese de p ser um nmero primo indispensvel.

O mnimo mltiplo comum

Observe que 6 divide 60 e 15 divide 60. Nesse caso, 60 dito um mltiplo comum de 6 e 15. Do mesmo modo, 8 divide 120 e 20 divide 120. Assim, dizemos que 120 um mltiplo comum de 8 e 20. De uma maneira geral, se a um nmero inteiro que divide o inteiro m, dizemos que m um mltiplo de a. Se a e b so nmeros inteiros no nulos, ento podemos concluir que tanto ab como -ab so mltiplos comuns de a e b e um deles tem de ser positivo (pelo Axioma da Tricotomia, visto na aula 2). Portanto, pelo Princpio do Menor Inteiro, existe um menor mltiplo comum de a e b. Seja m este nmero positivo que o menor mltiplo comum de a e b. Chamamos m de Mnimo Mltiplo Comum de a e b, e notamos por m = MMC( a, b ). Resumindo, m = MMC( a, b ) se, e somente se, (i) m um inteiro positivo; (ii) a divide m e b divide m (m um mltiplo comum de a e b); (iii) Se n um mltiplo de a e b, ento n m (m o menor mltiplo comum).
Exemplo 5 O MMC (12, 30) 60, pois (i) 60 um inteiro positivo; (ii) 12 divide 60, pois 60 = 12 . 5, e 30 divide 60, pois 60 = 30 . 2; (iii) 60 o menor mltiplo comum de 12 e 30.

Exerccio 3 Ache: (a) MMC(12, 42) (b) MMC(18, 32)

Existe uma relao entre o MDC e o MMC dada por:

51

Proposio 2 Se a e b so dois nmeros inteiros positivos, ento: MDC (a, b) x MMC (a, b) = ab. Demonstrao

De fato, seja d = MDC (a, b). Como d um divisor de a e b, podemos escrever: a = dm e b = dn, com m, n Z. Se s = ab/d, ento s = dmb/d = bm e s = adn/d = na. Portanto, s um inteiro positivo que mltiplo de a e b. Precisamos mostrar que s o Mnimo Mltiplo Comum de a e b. Para isso, suponha que outro nmero inteiro positivo r seja um mltiplo comum de a e b. Assim, podemos escrever r = au e r = bv, com u, v Z. Por outro lado, sabemos, pelo Teorema 1, que existem inteiros x e y tais que d = ax + by. Logo, podemos escrever: r rd r (ax + by ) r r = = = x + y = (vx + uy ) Z . Portanto, s divide r. Em particular, s s sd ab b a ento ds = ab, isto , r. Logo, podemos concluir que s = MMC (a, b). Como MDC(a, b). MMC(a, b) = ab

Corolrio 3 Se MDC (a, b) = 1, temos, MMC (a, b) = ab. Demonstrao

Da Proposio 2, temos MDC (a, b) x MMC (a, b) = ab. Como, por hiptese, MDC (a, b) = 1, segue que MMC (a, b) = ab.

Exemplo 6 Achar MMC(72, 20). Soluo

Basta observar que MDC(72, 20) = 4 e aplicar a Proposio 2: MDC(72, 20) x MMC(72, 20) = 72 x 20. Assim, 4 x MMC(72, 20) = 72 x 20. Portanto, MMC(72, 20) =
Exerccio 4 Encontre MMC(120, 36).

72 x 20 = 360. 4

Equaes diofantinas lineares

usual chamar equaes diofantinas as equaes com coeficientes inteiros e com uma ou mais incgnitas a serem procuradas no conjunto dos nmeros inteiros. As mais simples so as equaes diofantinas lineares: ax + by = c, com a, b, c constantes e a, b, c Z. A soluo de uma equao diofantina desse tipo so dois inteiros x0, y0, tais que ax0 + by0 = c.

52

As equaes seguintes so diofantinas: 3x + 8y = 9; 4x + 30y = 42; x + y = 1. O nome dessas equaes em homenagem a Diofanto, que iniciou estudos no sentido de resolv-las e que tomamos conhecimento atravs de Os Elementos, de Euclides. Diofanto viveu em Alexandria por volta de 250 depois de Cristo.
Diofanto de Alexandria considerado como o maior algebrista grego. Na histria da Aritmtica, esse autor desempenha um papel semelhante ao que Euclides (360-295 aC) tem na Geometria e Ptolomeu (85-165) na Astronomia. Sabe-se relativamente pouco sobre a sua vida. Desconhece-se a data precisa em que Diofanto nasceu. No entanto, atravs da leitura dos seus escritos, nos quais cita Hipsicles (240-170 a.C.) e tambm por uma passagem de Thon de Alexandria (335-395), que cita Diofanto como um clssico, possvel marcar limites temporais que permitem situar a vida desse autor entre o Sculo II a.C. e o princpio do Sculo IV da nossa Era. De acordo com P. Tannery, deve-se considerar Diofanto como contemporneo de Papus (290-350) e pertencendo segunda metade do Sculo III. Por outro lado, atendendo ao que na parte da aritmtica da mutilada obra de Papus no mencionado o nome de Diofanto, sendo, no entanto, citados, no s diversos outros gemetras da poca, mas tambm quase todos os matemticos do seu tempo como Hron (10-75), Nicmaco (60-120), Thon e Ptolomeu, Diofanto possa ser um pouco posterior a Papus.

Entre os vrios livros que escreveu, o mais importante "Aritmtica". Neste, ele introduz uma notao simblica com smbolos diferentes para o quadrado de uma incgnita, para o cubo e assim sucessivamente. Em sua tumba estava escrito o seguinte enigma: "Aqui jaz o matemtico que passou um sexto da sua vida como menino. Um doze avos da sua vida passou como rapaz. Depois viveu um stimo da sua vida antes de se casar. Cinco anos aps, nasceu seu filho, com quem conviveu metade da sua vida. Depois da morte de seu filho, sofreu mais 4 anos antes de morrer". De acordo com esse enigma, Diofanto teria 84 anos. (Fonte: http://pt.wikipedia.org/wiki/Diofanto_de_alexandria)
Exemplo 7 2x + 8y = 10 uma equao diofantina e x = 1 e y = 1 uma soluo, pois 2.1 + 8.1 = 10. Observe que x = 5 e y = 0 outra soluo. Assim, de uma maneira geral, as solues de uma equao diofantina no so nicas Ateno! Quando estudamos as equaes diofantinas natural surgir os seguintes questionamentos: Pergunta 1: toda equao diofantina admite soluo? Resposta: nem toda equao diofantina admite soluo. Por exemplo, 4x + 6y = 13 no admite soluo, uma vez que poderamos escrever, 13 = 4x + 6y = 2(2x + 3y). Isso nos diz que 13 par, o que uma contradio. Pergunta 2: que condies devem satisfazer os nmeros inteiros a, b e c para que a equao diofantina ax + by = c, admita soluo?

53

Resposta: a equao diofantina ax + by = c, com a, b, c constantes e a, b, c Z admite soluo se, e somente se, o MDC (a, b) divide c. De acordo com esse critrio, a equao 4x + 6y = 13 no admite soluo porque MDC (4, 6) = 2 e 2 no divide 13. Pergunta 3: como justificar o critrio de solubilidade das equaes diofantinas? Resposta: vamos supor que a equao diofantina ax + by = c, com a, b, c constantes e a, b, c Z admita soluo. Isso quer dizer que existem inteiros x0, y0 tais que ax0 + by0 = c. Seja d = MDC (a, b). Assim, pelo Teorema 1, existem r, s Z tais que a = dr e b = ds. Desse modo, podemos escrever:

c = ax0 + by0 = drx0 + ds.y0 = d(rx0 + s.y0). Portanto, d = MDC (a, b) divide c. Por outro lado, se d = MDC (a, b) divide c, ento existe t Z tal que c = dt. J sabemos que existem dois inteiros x0, y0 tais que d = MDC (a, b) = ax0 + by0 . Multiplicando ambos os membros desta ltima igualdade por t, obtemos: c = dt = (ax0 + by0).t = a(tx0) + b(ty0). Portanto, a equao diofantina admite uma soluo x = (tx0) e y = (ty0), o que conclui a justificativa de que a equao diofantina ax + by = c, com a, b, c constantes e a, b, c Z admita soluo se, e somente se, o MDC (a, b) divide c.

Exemplo 8 Considere a seguinte equao diofantina 5x + 12 y = 18. Essa equao diofantina admite soluo, pois MDC (5, 12) = 1 e 1 divide 18, enquanto a equao diofantina 18x + 12 y = 25 no admite soluo, pois MDC (18, 12) = 6, pois 6 no divide 25. Pergunta: supondo que a equao diofantina ax + by = c, com a, b, c constantes e a, b, c Z admite soluo, quais so todas as suas solues?

A resposta dada pela Proposio 3.


Proposio 3 Se x0, y0 uma soluo particular da equao diofantina ax + by = c, ento todas as solues desta equao so dadas por:

b x = x0 + t e y = y0 d Demonstrao

a t , com t um nmero inteiro. d

Sejam x e y outra soluo qualquer da equao. Assim, temos: ax + by = c e ax0 + by0 = c.

Portanto, ax + by = ax0 + by0, que o mesmo que:

54

a(x - x0) = b(y0 y). (*) Seja d = MDC (a, b). Sabe-se que existem inteiros, r e s, relativamente primos, tais que a = dr e b = ds. Logo, a igualdade (*) pode ser reescrita como: a(x - x0) = b(y0 y) = dr(x - x0) = ds(y0 y) ou ainda: r(x - x0) = s(y0 y). Como s divide r(x - x0) e s relativamente primo com r, temos que s divide (x - x0). Logo, b x - x0 = st ou ainda x = x0 + st,= x0 + t , com t um nmero inteiro. d Por outro lado, como r(x x0) = s(y0 y1) e x x0 = st, ento rst = s(y0 y). Logo, a y0 y= rt, ou ainda y= y0 rt = y0 - t , com t um nmero inteiro. d

b Para concluir, basta verificar que, x = x0 + t d inteiro, satisfaz a equao diofantina dada, de fato: b ax + by = a[x0 + t ] + b[y0 d

a y = y0 - t , com t um nmero d

a ab ab t ] = (ax0 + by0) + t = (ax0 + by0) + 0 = c. d d d

Exemplo 9 Por exemplo, 2x + 8y = 10 uma equao diofantina e x = 1 e y = 1 uma soluo particular. Como MDC (2, 8) = 2 = d, a = 2 e b = 8, todas as solues so dadas por:

8 x = 1 + t = 1 + 4t 2

2 y = 1 - t = 1 t, com t um nmero inteiro. 2

Exerccio 5 Encontre todas as solues da equao diofantina 7x 12y = 9. Uma pergunta: dada a equao diofantina 172x + 20y = 1000, como encontrar uma soluo particular? A resposta : aplicando o Algoritmo da Diviso para encontrar o MDC (172, 20) e trabalhando no sentido inverso para encontrar a soluo particular. Ou seja:

172 = 8 x 20 + 12; 20 = 1 x 12 + 8; 12 = 1 x 8 + 4; 8 = 2 x 4 + 0.

55

Agora, como MDC (172, 20) = 4 e 4 divide 1.000, podemos concluir que a equao diofantina dada tem soluo e percorrendo os clculos no sentido inverso vamos encontrar uma soluo particular: 4 = 12 8 = 12 (20 12) = 2 x. 12 20 = 2 x (172 8 x 20) = 2 x 172 + (-17) x 20. Finalmente, multiplicando 250 a cada membro da ltima igualdade, obtemos: 4 x 250 = 1000 = 500 x 172 + (-4250) x 20. Ou seja, x = 500 e y = -4250 uma soluo particular da equao diofantina dada.
Observao

b a x = x0 + t e y = y0 - t , com t um nmero inteiro, so todas as solues da d d equao diofantina ax + by = c, com d = MDC (a, b) dividindo c. Se d = 1, ento a soluo geral da equao dada dada por: x = x0 + bt e y = y0 at, para t um nmero inteiro.

Exemplo 10 Encontrar a soluo geral da equao diofantina 3x + 8y = 5. Soluo

A equao dada tem soluo particular x = - 1 e y = 1 e como o MDC(3, 8) = 1, a soluo geral dada por x = -1 + 8t e y = 1 3t, com t um nmero inteiro.

Exemplo 11 Um fazendeiro deseja comprar filhotes de pato e de galinha, gastando um total de R$ 1.770,00. Um filhote de pato custa R$ 31,00 e um de galinha custa R$ 21,00.

Quantos de cada um dos dois tipos o fazendeiro poder comprar? Soluo Chamemos de x o nmero de patos comprados e y o nmero de galinhas. Assim, podemos modelar o problema da seguinte maneira 31x + 21y = 1770. Observe que MDC (31, 21) = 1 e que 1 divide 1.770. Logo, a equao tem soluo. Vamos encontrar uma soluo particular. Para isso, usamos o Algoritmo da Diviso: 31 = 1.21 + 10; 21 = 2.10 + 1; 1 = 21 + (-2) .10 = 21 + (-2).[31 + (-1). 21] = 3.21 + (-2).31. Multiplicando ambos os lados por 1.770, obtemos:

56

1770 = (-3540).31 + (5310).21. Portanto, uma soluo particular x = -3540 dada por: x = -3540 + 21t e y = 5310 31t. e y = 5310. A soluo geral da equao

Observe que estamos interessados somente nas solues positivas ou nulas, pois representam quantidades de animais. Assim, temos que impor as condies seguintes: -3540 + 21t 0 e 5310 - 31t 0. e t 171,29.

Portanto, 21t 3540 e 31t 5310, que o mesmo que: t 168,57 Assim, como t um nmero inteiro, temos que 169 t 171. Desse modo, as solues so: x = -3540 + 21.169 = 9 e y = 5310 31.169 = 71; ou x = -3540 + 21.170 = 30 e y = 5310 31.170 = 40; ou x = -3540 + 21.171 = 51 e y = 5310 31.171 = 9. Essas solues dizem que o fazendeiro tem trs alternativas de comprar: 9 patos e 71 galinhas ou 30 patos e 40 galinhas, ou 51 patos e 9 galinhas.

Exerccio 6 Se um estudante tem em seu cofre muitas moedas de 10 e de 25 centavos, de quantas maneiras distintas pode pagar seu lanche que custou R$ 2,65?

EXERCCIOS
1) Ache o MDC(1000, 14400) e o MMC ( 12, 42). 2) (a) Encontre o quociente q e o resto r na diviso de 72 por 20. (b) Ache MDC(72, 20). (c) Ache MDC(20,r). (d) Compare MDC(72, 20) e MDC(20,r). 3) (a) Calcule o MDC( 990, 720). (b) Calcule o MDC(990 720, 720). (c) Compare o resultado do subitem (a) com o do subitem (b). 4) (a) Encontre o valor do inteiro positivo d tal que d = MDC(12, 28). (b) Expresse d como uma combinao linear de 12 e 28. Isto , encontre nmeros inteiros u e v tais que d = 12u + 28v. (c) Verifique que o conjunto S = {12x + 28y| x, y Z} a coleo dos mltiplos inteiros de d.. 5) Usando o fato: se a, b N, com M.D.C.(a, b) = 1, existem x, y Z tais que ax + by = 1, verifique que a raiz quadrada de dois no um nmero racional. (Nota: um nmero racional da forma a/b, onde a e b so nmeros inteiros, com b diferente de zero).

57

6) Encontre, se possvel, as solues de cada uma das equaes diofantinas: (a) 28x + 35y = 91 (b) 24x + 15y = 9.

Resumo Nesta aula, vimos que o MDC entre dois nmeros o maior divisor comum a esses nmeros, enquanto o MMC entre eles o menor mltiplo comum de ambos. Vimos tambm como encontrar solues de Equaes Diofantinas, teis em modelagem de problemas do cotidiano.

Problemas Suplementares
Problema 1 Uma sequncia de inteiros positivos dada por: a1 = 1 e a n = MDC (a n 1 , n) + 1 , para n > 1. Calcule a 2002 . Resp. 3 Problema 2 Arranjam-se 10 nmeros inteiros positivos em torno de um crculo. Cada nmero 1 mais do que o MDC dos dois vizinhos. Qual a soma dos dez nmeros? Resp. 28 Problema 3 Os nmeros naturais m e n so relativamente primos. Prove que MDC(m + n, m2 + n2) ou 1 ou 2. Sugesto: Se d divide m + n, ento d divide (m + n)2 (m2 + n2) = 2mn. Portanto, d divide 2m(m + n) 2mn = 2m2 e divide 2n(m + n) 2mn = 2n2 Referncias

BURTON, David M. Elementary number theory. New York: McGraw-Hill, 1998. COUTINHO, S. C. Nmeros inteiros e criptografia RSA. Rio de Janeiro: Instituto de Matemtica Pura e Aplicada IMPA/ Sociedade Brasileira de Matemtica SBM, 1997. CRAWFORD, Mathew. Introduction number theory: the art of problem solving. Alphine: AoPS Incorporated, 2006. HEFEZ, Abramo. Elementos de aritmtica. Rio de Janeiro: Sociedade Brasileira de Matemtica, 2005

58

Aula 6 Representao dos nmeros naturais e critrios de divisibilidade


Apresentao

Nesta aula, estudaremos a representao dos nmeros naturais numa base qualquer e destacaremos os casos especiais nas bases 10 e 2. Em seguida, estudaremos os critrios de divisibilidade, que nos permitem decidir se um determinado inteiro ou no divisvel por outro, sem precisar efetuar a diviso. Esses critrios de divisibilidades foram estudados por vrios matemticos como al-Khwarizmi, cujo nome completo era Abu Abdullah Mohammed ben Musa al-Khwarizmi (nasceu por volta do ano 780 depois de Cristo e morreu entre 830 e 850 depois de Cristo), e Leonardo de Pisa (cerca de 1180 1250), mais conhecido como Fibonacci, que significa filho de Bonacci, matemtico e comerciante da idade mdia, que escreveu, em 1202, um livro denominado Liber Abacci que chegou a ns, graas sua segunda edio de 1228. Tente entender tudo que est sendo explicado na aula. Estude com caneta e papel ao lado. Leia com ateno. Se for preciso, leia vrias vezes uma linha ou um pargrafo. Seja paciente e procure ter certeza que voc entendeu o que (e por que) est fazendo.
Objetivos

Expandir um nmero natural numa base qualquer b. Dada a representao de um nmero natural na base b, identificar esse nmero na base 10. Decidir, sem efetuar a diviso, quando um inteiro divisvel por: 2, 3, 4, 5, 6, 7, 8, 9, 10, 11...

Sistema de numerao decimal

Para representar os nmeros naturais, os babilnios usavam o sistema sexagesimal, desenvolvido na China e na India, h cerca de 1700 anos antes de Cristo. A partir da publicao do livro de Fibonacci, Liber Abacci, o sistema decimal posicional, hoje universalmente adotado, passou a ser difundido na Europa. No sistema decimal posicional, todo nmero natural representado por uma seqncia dos dgitos: 0, 1, 2, 3, 4, 5, 6, 7, 8, 9, o smbolo 0 (zero) representa a ausncia de qualquer outro dgito. O sistema chamado decimal pelo fato de ter dez smbolos ao todo: 0, 1, 2, 3, 4, 5, 6, 7, 8, 9. Num nmero natural escrito na base decimal ou base 10, cada dgito tem, alm de seu valor prprio, um peso que lhe atribudo em funo da posio que ele ocupa nesse nmero. Esse peso sempre uma potncia de 10, que varia de acordo com as regras abaixo: a) o algarismo da extrema direita tem peso 1; b) da direita para esquerda, o algarismo seguinte tem peso 10; c) o terceiro algarismo, da direita para a esquerda, tem peso 100;

59

d) o quarto algarismo, da direita para a esquerda, tem peso 1000, e assim por diante. Seguindo as regras acima, como se representa na base 10 cada nmero natural? Vamos comear pela representao dos nmeros de 1 at 9. Cada um dos dgitos de 1 at 9 representado por ele mesmo. O nmero dez representado por 10; o nmero cem representado por 100; o nmero mil representado por 1000 etc. O nmero 1789 tem a seguinte representao decimal: 1 x 1000 + 7 x 100 + 8 x 10 + 9 = 1 x 103 + 7 x 102 + 2 x 10 + 9. No nmero mil setecentos e vinte e nove, 1729, o dgito 9, pela posio que ocupa, chamado o dgito das unidades, enquanto o dgito 2 chamado o dgito das dezenas (pois seu peso 10), o 7 o dgito das centenas (pois seu peso 102 = 100) e o 1 o dgitos dos milhares (seu peso 103 = 1000). O nmero dois mil e sete, 2007, tem a seguinte representao decimal: 2 x 103 + 0 x 102 + 0 x 10 + 7. O dgito 7 o dgito das unidades, o primeiro 0, contado da direita para a esquerda, o dgito das dezenas e o segundo zero das centenas. O dgito 2 o dgito dos milhares. O nmero 173.648 tem a seguinte representao decimal: 1 x 105 + 7 x 104 + 3 x 103 + 6 x 102 + 4 x 10 + 8.
Uma questo: dado um nmero natural, como obter a sua expanso decimal?

Para exemplificar, suponha que o nmero dado 173.648. Inicialmente, o que fazemos usar o Algoritmo da Diviso, dividindo o nmero por 10. Em seguida, dividimos o quociente por 10 e assim por diante: 173.648 = 17364 x 10 + 8 = (1736 x 10 + 4) x 10 + 8 = 1736 x 102 + 4 x 10 + 8 = = (173 x 10 + 6) x 102 + 4 x 10 + 8 = 173 x 103 + 6 x 102 + 4 x 10 + 8 = = (17 x 10 + 3) x 103 + 6 x 102 + 4 x 10 + 8 = = 17 x 104 + 3 x 103 + 6 x 102 + 4 x 10 + 8 = = (1 x 10 + 7) x 104 + 3 x 103 + 6 x 102 + 4 x 10 + 8 = = 1 x 105 + 7 x 104 + 3 x 103 + 6 x 102 + 4 x 10 + 8. A expanso do nmero 173.648, acima, chamada expanso relativa base decimal, ou seja, relativa base 10. fcil ver que, dados os nmeros naturais m e a, com a > 1, existe uma expanso do nmero m na base a. Isto , existem inteiros no negativos cn, cn-1,....., c2, c1, c0 (a), todos menores do que a, univocamente determinados, tais que: m = cnan + cn-1an - 1 + .....+ c3a3+ c2 a2 + c1a + c0 (*)

Basta proceder como fizemos no exemplo acima, quando encontramos a expanso do nmero 173.648 na base 10, s que agora troca-se o 10 por a. A expanso (*) acima

60

chamada a expanso do nmero m relativo base a e c0c1c2...cn (a) a representao de m na base a.


Exemplo 1 Seja K um nmero natural, cuja representao na base 10 :

K = cn10n + cn-110n-1 + .... + c3103 + c2 102 + c110 + c0, com cn, cn-1, ..., c3, c2, c1, c0 inteiros no negativos, todos menores do que 10. Mostre que K pode ser escrito como K = c0 + 10M, onde M um nmero natural. Soluo Basta olhar para a representao decimal de K como sendo a soma de duas parcelas: K = c0 + (c110 + c2 102 + c3103 + ... + cn10n), ou ainda: K = c0 + 10(c1 + c2 10 + c3102 + ... + cn10n-1). Chamando M = c1 + c2 10 + c3102 + ... + cn10n-1, podemos escrever K como sendo K = c0 + 10M.

Exerccio 1 Seja K um nmero natural. Mostre que na representao de K2 na base 10 o dgito das unidades s pode ser 0, 1, 4, 5, 6 ou 9.

O sistema de base dois

O Sistema de Base Dois ou Sistema Binrio um sistema numrico que expande valores numricos usando somente dois smbolos: os dgitos 0 e 1. Isso decorre do fato de que os possveis restos na diviso por 2 so: 0 e 1. O Sistema Binrio usado por todos os computadores modernos, pois devido tecnologia empregada o zero representa a ausncia de corrente e o 1 a presena de corrente.
Pergunta: Usando somente os dois dgitos, 0 e 1, como fazer para representar todos os nmeros naturais? Resposta: Do mesmo modo que usamos na expanso de um nmero natural na base 10, a representao de um nmero natural na base 2, cada dgito, 0 ou 1, tem um peso que lhe atribudo em funo da posio que ele ocupa nesse nmero. Esse peso sempre uma potncia de 2, que varia do seguinte modo:

a) o algarismo da extrema direita tem peso 1; b) da direita para esquerda, o algarismo seguinte tem peso 2; c) o terceiro algarismo, da direita para a esquerda, tem peso 22 = 4; d) o quarto algarismo, da direita para a esquerda, tem peso 23 = 8 , e assim por diante. Para escrever os nmeros naturais de um s dgito na base 2, fazemos 0=0

61

1=1 Esses dois nmeros so nicos nmeros naturais representados usando um s dos dgitos 0 e 1. Para escrever os dgitos 2 e 3 na base 2, usamos dois dos dgitos 0 e 1: Como 2 = 1.2 + 0 e 3 = 1.2 + 1, escrevemos 2 = 10(2) e 3 = 11(2). Para escrever na base 2 os nmeros de 4 at 7, usamos trs dos dgitos 0 e 1: 4(2) = 100(2), 5(2) = 101(2), 6(2) = 110(2), 7(2) = 111(2), pois pois pois pois 4 = 1.22 + 0.2 + 0; 5 = 1.22 + 0.2 + 1; 6 = 1.22 + 1.2 + 0; 7 = 1.22 + 1.2 + 1.

Ainda para exemplificar, para escrever na base 2 os nmeros de 8 a 15, usamos quatro dos dgitos 0 e 1: 8(2) = 1000(2), 9(2) = 1001(2), 10(2) = 1010(2), 11(2) = 1011(2), 12(2) = 1100(2), 13(2) = 1101(2), 14(2) = 1110(2), 15(2) = 1111(2), pois pois pois pois pois pois pois pois 8 = 1.23 + 0.22 + 0.2 + 0; 9 = 1.23 + 0.22 + 0.2 + 1; 10 = 1.23 + 0.22 + 1.2 + 0; 11 = 1.23 + 0.22 + 1.2 + 1; 12 = 1.23 + 1.22 + 0.2 + 0; 13 = 1.23 + 1.22 + 0.2 + 1; 14 = 1.23 + 1.22 + 1.2 + 0; 15 = 1.23 + 1.22 + 1.2 + 1.

Costuma-se escrever 15 = 1.23 + 1.22 + 1.2 + 1 = 1111(2). Ou seja, 1111(2) a representao de 15 na base 2. Do mesmo modo, 1001(2) a representao de 9 na base 2.
Exemplo 2 Como representamos na base 2 o nmero 54? Soluo

A representao de 54 na base 2 feita de modo anlogo sua representao na base 10, porm mudando 10 por 2. Assim, dividimos 54 por dois e, em seguida, dividimos o quociente por 2 e assim por diante, at obter o dividendo como sendo 1: 54 = 27.2 + 0 = (13.2 + 1).2 + 0 = 13.22+ 1.2 + 0 = = (6.2+ 1).22 + 1.2 + 0 = 6.23 + 1.22 + 1.2 + 0 = = (3.2 + 0).23 + 1.22 + 1.2 + 0 = 3.24 + 0.23 + 1.22 + 1.2 + 0 = = (1.2 + 1).24 + 0.23 + 1.22 + 1.2 + 0 = = 1.25 + 1.24 + 0.23 + 1.22 + 1.2 + 0 Portanto, a representao de 54 na base dois 110110(2).
Exerccio 2 Verifique que a representao na base 2 de todo nmero natural par termina em zero e a representao na base 2 de todo nmero natural mpar termina em 1.

62

Uma pergunta: Como identificar na base 10 o nmero 10101001101?

representado na base 2 por

Resposta: basta atribuir os pesos para cada dgito que comparece na representao do nmero na base 2 e efetuar a soma:

1.210 + 0.29 + 1.28 + 0.27 + 1.26 + 0.25 + 0.24 + 1.23 + 1.22 + 0.2 + 1 = 1357. Portanto, 1357 = 10101001101(2).
Exerccio 3 Se a representao na base 2 do nmero natural K 11101011(2), qual a representao na base 2 do nmero (K +5)? Exemplo 3 Mostre que se b um nmero natural maior do que 2, ento, o nmero 121(b) um quadrado perfeito. Soluo

De fato, 121(b) = 1.b2 + 2.b + 1 = (b + 1 )2.


Exemplo 4 Num pas distante, os nmeros so escritos na base r e a moeda local o potiguar, abreviada Poti. Um homem comprou um boi por 440 potis. Para efetuar a compra, ele deu ao vendedor uma cdula de 1000 potis e recebeu de troco 340 potis. Qual o valor da base r? Soluo

Pelos dados do problema, temos 1000 potis 440 potis = 340 potis, na base r. Portanto, (1.r3 + 0.r2 + 0.r + 0) (4.r2 + 4.r + 0) = 3.r2 + 4.r + 0, que o mesmo que r3 7r2 8r = 0. Como a base r um nmero positivo, podemos dividir ambos os lados por r, obtendo r2 7r 8 = 0. Resolvendo a equao, temos: r = 8 ou r = -1. Como a base r positiva, r tem que ser igual a 8. Observe que, na base r, tem-se 440 + 340 = 1000. Isso significa que 4 + 4 = 0 no sistema de base r. Portanto, de fato, a base r = 8.
Exerccio 4 O nmero natural K tem como representao na base 2 o nmero 111000110(2). Qual a representao binria do nmero M = 2.K? Exemplo 5 O nmero 15! = 1 x 2 x 3 x ... x 14 x 15 ( fatorial de 15) quando escrito na base decimal termina com a zeros. Quando escrevemos 15! na base 12, termina com b zeros. Qual o valor de a.b? Soluo

63

O que produz um zero no final de um nmero na base 10 um fator de 10. Ou seja, a representao de um nmero natural M na base 10 termina com um nico zero se M = 10 x K, onde K um nmero natural e 10 no divide K. De modo anlogo, a representao de um nmero natural Q na base 10 termina com dois zeros se Q = 102 x J, onde J um nmero natural e 10 no divide J. Usando o raciocnio acima, a representao de 15! na base 10 termina com quantos zeros quantos forem seus fatores de 10. Mas, 15! = 1 x 2 x 3 x ... x 14 x 15 = 1 x 2 x 3 x 4 x 5 x 6 x 7 x 8 x 9 x 10 x 11 x 12 x 13 x 14 x 15 = = (2 x 5 x 10 x 2 x 5) x (1 x 3 x 4 x 6 x 7 x 4 x 9 x 11 x 12 x 13 x 14 x 3) = = 103 x (1 x 3 x 4 x 6 x 7 x 4 x 9 x 11 x 12 x 13 x 14 x 3). Portanto, a = 3. De modo anlogo ao que fizemos acima, podemos concluir que: o que produz um zero no final de um nmero na base 12 um fator de 12. Ou seja, a representao de um nmero natural M na base 12 termina com um nico zero se M = 12 x K, K um nmero natural e 12 no divide K. De modo anlogo, a representao de um nmero natural Q termina com dois zeros se Q = 122 x J, onde J um nmero natural e 12 no divide J. Assim, a representao de 15! na base 12 termina com quantos zeros quantos forem seus fatores de 12. Mas, 15! = 1 x 2 x 3 x ... x 14 x 15 = 1 x 2 x 3 x 4 x 5 x 6 x 7 x 8 x 9 x 10 x 11 x 12 x 13 x 14 x 15 = = (2 x 6 x 3 x 4 x 12 x 4 x 3 x 2 x 3 x 2) x (1 x 5 x 7 x 3 x 10 x 11 x 13 x 7 x 5) = = 125 x (1 x 5 x 7 x 2 x 3 x 10 x 11 x 13 x 7x 5). Portanto, b = 5. Logo, a.b =3 x 5 = 15.
Exemplo 6 Um nmero K tem trs dgitos na sua representao na base 7. Quando representamos K na base 9 os dgitos so os mesmos da representao na base 7 s que invertidos. Qual a representao de K na base 10? Soluo

Sejam a, b e c os dgitos de K. Ou seja, K = a x 72 + b x 7 + c = c x 92 + b x 9 + a . Assim, temos: b x 7 - b x 9 = (c x 92 + a) (a x 72 + c), que o mesmo que -2b = 80c 48a , ou seja b = 24a - 40c = 8(3a 5c). Conclumos que o dgito b um mltiplo de 8. Por outro lado, o dgito b satisfaz as desigualdades 0 b < 7. Logo, necessariamente temos 3a 5c = 0. Portanto, b = 0. Como 3a = 5c, segue que c divisvel por 3 e 5 divide a. Mas, na base 7, K tem trs dgitos. Portanto, a no pode ser zero e satisfaz as desigualdades 0 < a < 7. Logo, a = 5 e c = 3. Conclumos que K = 503(7) = 305(9) = 248 na base 10.

64

Exerccio 5 Se o nmero 46(b) igual ao triplo de 15(b), qual o valor de b? Exemplo 7 O adivinho indiscreto Convide um colega para dizer, dentre os 6 cartes abaixo, de 32 nmeros cada, em quais deles est a sua idade. Imediatamente voc advinha idade dele. Onde est o segredo?

1 3 5 7 9 1 1 1 3 1 5 1 7 1 9 2 1 2 3 2 5 2 7 2 9 3 1 3 3 3 5 3 7 3 9 4 1 4 3 4 5 4 7 4 9 5 1 5 3 5 5 5 7 5 9 6 1 6 3 3 6 7 1 0 1 1 1 4 1 5 1 8 1 9 2 2 2 3 2 6 2 7 3 0 3 1 3 4

2 3 5 3 8 3 9 4 2 4 3 4 6 4 7 5 0 5 1 5 4 5 5 5 8 5 9 6 2 6 3 5 6 7 12 13 14 15 20 21 22 23 28 29 30 31 36

4 3 7 3 8 3 9 4 4 4 5 4 6 4 7 5 2 5 3 5 4 5 5 6 0 6 1 6 2 6 3 9 1 0 1 1 1 2 1 3 1 4 1 5 2 4 2 5 2 6 2 7 2 8 2 9 3 0 3 1 4 0

8 4 1 4 2 4 3 4 4 4 5 4 6 4 7 5 6 5 7 5 8 5 9 6 0 6 1 6 2 6 3 1 7 1 8 1 9 2 0 2 1 2 2 2 3 2 4 2 5 2 6 2 7 2 8 2 9 3 0 3 1 4 8

1 6 49 50 51 52 53 54 55 56 57 58 59 60 61 62 63 3 3 3 4 3 5 3 6 3 7 3 8 3 9 4 0 4 1 4 2 4 3 4 4 4 5 4 6 4 7 4 8

3 2 4 9 5 0 5 1 5 2 5 3 5 4 5 5 5 6 5 7 5 8 5 9 6 0 6 1 6 2 6 3

Soluo

Um nmero natural K, entre 1 e 63, pode ser escrito na base 2 como K = a5 .25 + a4 .24 + a3.23 + a2.22 + a1.2 + a0 ,

65

com cada um dos nmeros a5 , a4 , a3 , a2 , a1 , a0 sendo 0 ou 1. Nesse caso, a representao do nmero K na base 2 , exatamente, a5 a4 a3 a2 a1 a0, onde ai = 0 ou 1, i = 1,2,3,4,5. Pois bem, na primeira lista do adivinho esto os nmeros para os quais a0 = 1, isto , aqueles que terminam em 1 quando escritos na base 2; na segunda lista, esto os nmeros para os quais a1 = 1, ou seja, aqueles, entre 1 e 63, que tm 1 na segunda casa da direita para esquerda, quando escritos na base 2; na terceira, esto aqueles para os quais a2 = 1 e assim por diante. Fica claro, agora, porque cada idade igual soma dos primeiros nmeros de cada lista em que ela esteja!
Exemplo 8 Uma base de numerao fatorial aquela pela qual se expressaria um nmero inteiro positivo K como sendo:

K = a1 + a2 . 2! + a3 . 3! + a4 . 4! + .... + an . n!,

(*)

onde a1, a2, a3, a4,...., an so inteiros no negativos com 0 ak K. Expresse os seguintes nmeros na base de numerao fatorial: a) 7 b) 15 c) 85 d) 695 Soluo a) Observe que, como 4! = 24 > 7, s podemos expressar 7 como uma expresso do tipo a1 + a2 . 2! + a3 . 3!. fcil ver que 7 = 1 + 0 . 2! + 1. 3! b) Veja que 4! = 24 > 15. Logo a expresso para 15 na base fatorial deve ser do tipo a1 + a2 . 2! + a3 . 3!. fcil ver que 15 = 1 + 1 . 2! + 2. 3! c) Observe que 5! = 120 > 85. Logo, 85 = a1 + a2 . 2! + a3 . 3! + a4 . 4!. Agora, fcil ver que 85 = 1 + 0 . 2! + 2 . 3! + 3 . 4! d) Como 6! = 720, a expresso para 695 do tipo a1 + a2 . 2! + a3 . 3! + a4 . 4! + a5 . 5!. Agora, observe que a5 = 5, caso contrrio no atingiremos 695 e a4 no pode ser 4, pois 4 . 4! + 5 . 5! > 695. Por outro lado, a4 no pode ser menor do que 3, pois se a4 = 2, teremos 1 + 2 . 2! + 3 . 3! + 2 . 4! < 95 e no conseguiramos atingir 695. Logo, a4 = 3. Agora, fcil ver que 695 = 1 + 2 . 2! + 3 . 3! + 3 . 4! + 5 . 5!.

Critrios de divisibilidade

Olhando somente para os nmeros na base 10, temos vrios critrios de divisibilidade. Ou seja, testes para saber se um nmero natural divisvel por outro sem ser preciso efetuar a diviso. Esse estudo ser mais completo com a noo de congruncia, a ser estudada na prxima aula. Como essa noo no normalmente estudada no Ensino Mdio, pretendemos adiantar alguns critrios de divisibilidade que possam ser provados facilmente sem o uso do conceito de congruncias. Assim, a seguir vamos estudar os critrios de divisibilidade mais comuns.

66

a) Divisibilidade por 2

Um nmero natural K divisvel por 2 se, e s se, na sua representao na base 10, seu algarismo das unidades divisvel por 2. Ou seja, um nmero natural K divisvel por 2 se, e somente se, na sua representao na base 10, termina em 0, 2, 4, 6 ou 8. De fato, representemos K na base 10 por: K = cn10n + cn-110n-1 + .... + c3103 + c2 102 + c110 + c0, com cn, cn-1, ..., c3, c2, c1, c0 inteiros no negativos, todos menores do que 10 e c0 sendo o dgito da unidades. Se K divisvel por 2, ento, explicitamos o valor de c0 como sendo c0 = K (cn10n + cn-110n-1 + .... + c3103 + c2 102 + c110). Agora, observe que o nmero natural (cn10n + cn-110n-1 + .... + c3103 + c2 102 + c110) divisvel por 2 porque uma soma de nmeros pares. Logo, c0 divisvel por 2, como diferena de dois nmeros divisveis por 2. Se c0 divisvel por 2, nesse caso, K = cn10n + cn-110n-1 + .... + c3103 + c2 102 + c110 + c0, divisvel por 2, pois uma soma finita de nmeros divisveis por 2.

b) Divisibilidade por 3

Um nmero natural K divisvel por 3 se, e somente se, na sua representao na base 10, a soma de seus dgitos um nmero divisvel por 3. De fato, representemos K na base 10 por: K = cn10n + cn-110n-1 + .... + c3103 + c2 102 + c110 + c0, com cn, cn-1, ..., c3, c2, c1, c0 inteiros no negativos, todos menores do que 10. Agora, observe que: 10 = 9 + 1 e 10k = (9 + 1)k. Usando o desenvolvimento do Binmio de Newton para (9 + 1)k, temos que (9 + 1)k = 9mk + 1, onde mk um nmero inteiro. Assim, temos que: ck10k = ck(9 + 1)k = ck(9mk + 1) = 9mkck + ck. Logo, K= = = = cn10n + cn-110n-1 + .... + c3103 + c2 102 + c110 + c0 = c0 + (c19m1 + c1)10 + (c2 9m2 + c2) + (c39m3 + c3) + ... + (cn9cn + cn) = (c19m1 + c2 9m2 + c39m3 + . + cn9cn) + (c0 + c1 + c2 + c3 + ... + cn) = 9(c1m1 + c2m2 + c3m3 + . + cncn) + (c0 + c1 + c2 + c3 + ... + cn).

Suponha que K seja divisvel por 3. Como (c0 + c1 + c2 + c3 + ... + cn) = K - 9(c1m1 + c2m2 + c3m3 + . + cncn),

67

segue que (c0 + c1 + c2 + c3 + ... + cn) divisvel por 3, por ser a diferena de dois nmeros divisveis por 3. Por outro lado, se (c0 + c1 + c2 + c3 + ... + cn) divisvel por 3, segue que K = 9(c1m1 + c2m2 + c3m3 + . + cncn) + (c0 + c1 + c2 + c3 + ... + cn) divisvel por 3, como soma de dois nmeros divisveis por 3.

Exemplo 9 Verifique, sem efetuar a diviso, se o nmero 187134574 divisvel por 3. Soluo

Basta calcular a soma 1 + 8 + 7 + 3 + 4 + 5 + 7 + 4 = 41. Como 41 no divisvel por 3, conclumos que o nmero 187134574 no divisvel por 3.
Exemplo 10 Escreva a seqncia crescente de todos os nmeros inteiros comeados por 10 e terminados por 99 para formar o nmero inteiro

K = 10111213141516............979899 Qual a maior potncia de 3 que divide K? Soluo Observe que a soma 1 + 2 + 3 + ... + 9 = 45. Portanto, a soma dos dgitos do nmero K igual: a soma dos nmeros de 10 a 19 mais a soma dos nmeros de 20 a 29, ...., soma dos nmeros de 90 a 99, que dada por : (1 x 10 + 45) + (2 x 10 + 45) + (3 x 10 + 45) + (4 x 10 + 45) + .... + (9 x 10 + 45) = = (1 + 2 + 3 + 4 + ... + 9) x 10 + 9 x 45 = 45 x 10 + 9 x 45 = 19 x 45 = 32 x 5 x 19. Conclumos que o nmero divisvel por 9, pois na decomposio em fatores primos, o 3 comparece elevado ao quadrado. Logo, a maior potncia de 3 que divide K dois.

c) Divisibilidade por 4

Um nmero natural K divisvel por 4 se, e somente se, na sua representao na base 10, o nmero formado pelos dois ltimos dgitos (contados da esquerda para a direita) forma um nmero divisvel por 4. Observe que 4 = 04 e 8 = 08, ambos divisveis por 4. Agora, suponha que K > 8 e a representao de K na base 10 seja: K = cn10n + cn-110n-1 + .... + c3103 + c2 102 + c110 + c0, com cn, cn-1, ..., c3, c2, c1, c0 inteiros no negativos, todos menores do que 10.

68

Olhando para K como sendo K = (cn10n + cn-110n-1 + .... + c3103 + c2 102 )+ c110 + c0, fcil ver que a parcela (cn10n + cn-110n-1 + .... + c3103 + c2 102 ) = 102(cn10n-2 + cn-110n-3 + .... + c310 + c2 )) divisvel por 4, pois podemos escrever 102 = 4 x 25. Assim, fcil concluir que K divisvel por 4 se, e s se, o nmero inteiro c110 + c0 divisvel por 4.
Exerccio 6 Verifique, sem efetuar a diviso, se o nmero 2008 divisvel por 4. d) Divisibilidade por 5

Um nmero natural K divisvel por 5 se, e s se, na sua representao na base 10, o dgito das unidades zero ou cinco. De fato, representemos K na base 10 por: K = cn10n + cn-110n-1 + .... + c3103 + c2 102 + c110 + c0, com cn, cn-1, ..., c3, c2, c1, c0 inteiros no negativos, todos menores do que 10 e c0 sendo o dgito da unidades. Observe que a parcela cn10n + cn-110n-1 + .... + c3103 + c2 102 + c110 um nmero inteiro divisvel por 5, pois mltiplo de 10 e 10 = 2 x 5. Portanto, K divisvel por 5 se, e s se, c0 divisvel por 5. Mas, 0 c0 9. Logo, c0 0 ou 5.
e) Divisibilidade por 6

Um nmero natural K divisvel por 6 se, e somente se, na sua representao na base 10, for divisvel simultaneamente por 2 e por 3. De fato, se K divisvel por 6, ento, K = 6k, com k um nmero inteiro. Mas, podemos escrever K = 6k = 2(3k) = 3(2k). Logo, K mltiplo simultaneamente de 2 e 3. Como 2 e 3 so primos, na decomposio em fatores primos do nmero K aparece o 2 e o 3. Logo, K mltiplo do produto 2.3 = 6.

f) Divisibilidade por 7

Antes de enunciar o critrio de divisibilidade por 7, vamos fazer algumas consideraes. Seja K um nmero natural, cuja representao na base 10 : K = cn10n + cn-110n-1 + .... + c3103 + c2 102 + c110 + c0, com cn, cn-1, ..., c3, c2, c1, c0 inteiros no negativos, todos menores que 10. Agora, olhemos para a representao decimal de K como sendo a soma de duas parcelas: K = cn10n + cn-110n-1 + .... + c3103 + c2 102 + c110 + c0, ou seja: K = 10(cn10n-1 + cn-110n-2 + .... + c3102 + c2 10 + c1) + c0 Chamando M = cn10n-1 + cn-110n-2 + .... + c3102 + c2 10 + c1, podemos escrever K como sendo

69

K = 10M + c0. Enunciamos a seguir o teste de divisibilidade por 7. Um nmero natural K = 10M + c0 divisvel por 7 se, e somente se, o nmero natural M + 5.c0 divisvel por 7. De fato, se K =10M + c0 divisvel por 7, ento, 5K = 5.( 10M + c0) tambm divisvel por 7. Mas, podemos escrever: 5K = 5.( 10M + c0) = 50M + 5.c0 = 49M + M + 5.c0. Portanto, M + 5.c0= 5K 49M um mltiplo de 7, como diferena de dois mltiplos de 7. Por outro lado, se M + 5.c0 um mltiplo de 7, ento, 10(M + 5.c0) um mltiplo de 7. Mas, 10(M + 5.c0) = 10M +50c0 = 49c0 + (10M + c0) = 49.c0 + K. Portanto, K = 10(M + 5.c0) - 49.c0 um mltiplo de 7, pois a diferena de dois mltiplos de 7.

Exemplo 11 Verifique, sem efetuar a diviso, se o nmero 1729 divisvel por 7. Soluo

Pelo que vimos anteriormente, escrevemos K = 1729 = 9 + 10 x (172) e K ser divisvel por 7 se, e somente se, o nmero 5 x 9 + 172 = 217 for divisvel por 7. Aplicando o critrio para o nmero 217, temos que 217 = 7 + 10 x (21) ser divisvel por 7 se, e somente se, o nmero 5 x 7 + 21 = 56 for divisvel por 7. Como 56 divisvel por 7, pois 56 = 7 x 8, conclumos que 217 divisvel por 7. Agora, observe que: 1729 divisvel por 7 217 divisvel por 7 56 divisvel por 7. Portanto, 1729 divisvel por 7.
Exerccio 7 Verifique, sem efetuar a diviso, se o nmero 23921083 divisvel por 7. g) Divisibilidade por 8

Um nmero natural K divisvel por 8 se, e somente se, na sua representao na base 10, o nmero formado pelos trs ltimos dgitos (contados da esquerda para a direita) forma um nmero divisvel por 8. K = cn10n + cn-110n-1 + .... + c3103 + c2 102 + c110 + c0, com cn, cn-1, ..., c3, c2, c1, c0 inteiros no negativos, todos menores do que 10. Olhamos para K da seguinte maneira:

70

K = 103(cn10n-3 + cn-110n-4 + .... + c310) + c2 102 + c110 + c0 Agora, fcil ver que a parcela 103(cn10n-3 + cn-110n-4 + .... + c310) divisvel por 8, pois 103 = 1000 = 8 x 125. Portanto, fcil ver que N divisvel por 8 se, e s se, o nmero c2 c1c0 = c2 102 + c110 + c0 um nmero divisvel por 8. A justificativa a mesma feita para o caso da divisibilidade por 2 e 4.

h) Divisibilidade por 9

Um nmero natural K divisvel por 9 se, e somente se, na sua representao na base 10, a soma de seus dgitos um nmero divisvel por 9. A justificativa a mesma feita para o caso da divisibilidade por 3.
i) Divisibilidade por 10

Um nmero natural K divisvel por 10 se, e somente se, na sua representao na base 10, divisvel simultaneamente por 2 e por 5. Ou, que o mesmo: um nmero natural K divisvel por 10 se, e somente se, na sua representao na base 10, o algarismo das unidades zero. A justificativa a mesma feita para o caso da divisibilidade por 6, mudando os primos 2 e 3 para 2 e 5.
j) Divisibilidade por 11

Um nmero natural K divisvel por 11 se, e somente se, na sua representao na base 10, a soma de seus dgitos nas posies pares menos a soma dos dgitos nas posies mpares um nmero divisvel por 11. De fato, representemos K na base 10 por: K = cn10n + cn-110n-1 + .... + c3103 + c2 102 + c110 + c0, com cn, cn-1, ..., c3, c2, c1, c0 inteiros no negativos, todos menores que 10. Agora, observe que: 10 = 11 - 1 e 10k = (11 - 1)k. Usando o desenvolvimento do Binmio de Newton para (11 - 1)k, temos que: (11 - 1)k = 11mk + 1, se k um inteiro par e mk um nmero inteiro e (11 - 1)k = 11mk - 1, se k um inteiro mpar e mk um nmero inteiro. Assim, temos que: ck10k = ck(11 - 1)k = ck(11mk + 1) = 11mkck + ck, se k um inteiro par e ck10k = ck(11 - 1)k = ck(11mk - 1) = 11mkck - ck, se k um inteiro mpar. Portanto, temos que

71

K = c0 + c110 + c2 102 + c3103 + ... + cn10n = = c0 + 11m1c1 c1 + 11m2c2 + c2 + 11m3c3 - c3 + 11m4c4 + c4 + .... + 11mncn+ (-1)k cn, = (c0 - c1+ c2 c3+ c4 - ......+ (-1)k cn) +(11m1c1 + 11 m2c2 + 11m3c3 + 11 m4c4 + 11mncn) = (c0 - c1+ c2 c3+ c4 - ......+ (-1)k cn) + 11s, com s Z. Portanto, fcil ver que K divisvel por 11 se, e somente se, (c0 - c1+ c2 c3+ c4 - ......+ (-1)k cn) divisvel por 11.

Exemplo 12 Verifique, sem efetuar a diviso, se o nmero 90806375 divisvel por 11. Soluo

Basta calcular a soma 9 0 + 8 0 + 6 3 + 7 5 = 22, que divisvel por 11. Logo, o nmero 90806375 divisvel por 11.
Exerccio 8 Verifique, sem efetuar a diviso, se o nmero 28382607 divisvel por 11. Exemplo 13 Considere a sequncia de nmeros inteiros dada por

1, 2, 2, 3, 3, 3, 4, 4, 4, 4, 5, 5, 5, 5, 5, 6, 6, 6, 6, 6, 6, .......... na qual o n-simo nmero inteiro positivo aparece n vezes. Qual o termo de ordem 2007? Soluo Observe que o 1 s aparece uma vez e na posio 1. A ltima apario do segundo inteiro, 2, na terceira posio: 1 + 2 = 3. A ltima apario do 3 na sexta posio: 1 + 2 + 3 = 6. A ltima apario do 4 na sexta posio: 1 + 2 + 3 + 4 = 10. fcil ver que a ltima apario do n-simo nmero inteiro positivo na sexta posio: 1 + 2 + 3 + 4 + ... + n = n(n+1)/2. Observe agora que: (62 x 63)/2 = 1953 < 2007 e (63 x 64)/2 = 2016 > 2007. Isso significa que a ltima apario de 62 na sequncia na posio de nmero 1953 e que a ltima posio de 63 a de nmero 2016. Portanto, o termo de ordem 2007 63.

Exerccios
1) Escreva na base 7 o nmero cuja representao na base 5 14432(5). 2) Escreva os primeiros 25 inteiros positivos na base 12.

72

3) Encontre o dgito das unidades do nmero (a) 1072007 (b) 24100 (c) 1 + 2 + 3 + + 100 4) Ana, uma adolescente, tem o quadrado da idade dela igual ao nmero da casa, na rua em que mora. A idade dela e o nmero da casa tm o mesmo dgito das unidades, mas a soma desses nmeros no um mltiplo de 10. Qual a idade de Ana? 5) Se o nmero 81(b) igual ao triplo de 15(b), qual o valor de b? 6) Qual o maior nmero de trs dgitos na base 14? 7) Encontre os nmeros naturais a e b tais que: 1727 = a.83 + b, onde 0 b < 83. 8) Verifique que: quando se subtrai de um nmero natural, na sua representao na base 10, a soma de seus dgitos, o resultado um nmero divisvel por 9. 9) Verifique, sem efetuar a diviso, se o nmero 174557974 divisvel por 7. 10) Na sua representao na base 10, quantos dgitos tem o nmero 7 x 541?
Sugesto Seja K = 7 x 541. Verifique que o logaritmo na base 10 de K , aproximadamente, igual 29,5.

11) Teste de Divisibilidade por 13 D uma justificativa para o teste de divisibilidade por 13, apresentado abaixo. Seja K um nmero natural, cuja representao na base 10 : cn10n + cn-110n-1 + .... + c3103 + c2 102 + c110 + c0, com cn, cn-1, ..., c3, c2, c1, c0 inteiros no negativos, todos menores que 10. Agora, olhemos para a representao decimal de K como sendo a soma de duas parcelas: K = (cn10n + cn-110n-1 + .... + c3103 + c2 102 + c110) + c0, ou seja: K = 10(cn10n-1 + cn-110n-2 + .... + c3102 + c2 101 + c1) + c0. Chamando M = cn10n-1 + cn-110n-2 + .... + c3102 + c2 101 + c1, podemos escrever K como sendo K = 10M + c0. Um nmero natural K divisvel por 13 se, e somente se, o nmero natural M + 4.c0 divisvel por 13. 12) Encontre o maior nmero primo (escrito na base 10) que divide a soma: 12 + 102 + 1002 + 10002 + + 1000000002.

73

13) Reduza a frao

116.690.151 427.863.887

na sua forma mais simples. em qualquer

14) Mostre que o (1110).(1111).(1112).(1113) = (1.235.431)2 1 sistema de numerao de base maior do que 5. 15) Em que base b o nmero M = 11111b um quadrado perfeito?

16) Um juiz resolve dar uma chance de liberdade para um condenado morte. O condenado tem que adivinhar uma senha que o livrar da sentena de morte. A senha formada por trs nmeros distintos, x, y e z, de dois dgitos cada um. O condenado tem de identificar trs nmeros A, B e C, de tal modo que permita encontrar o nmero Ax + By + Cz fornecido pelo juiz e da encontrar a senha x, y e z. Como voc pode ajudar o condenado a obter a liberdade?
Sugesto O condenado tem que descobrir um mtodo para distinguir os trs nmeros de dois dgitos. A base 100 a sada. Escreva A = 1002, B = 100 e C = 1 e Ax + By + Cz = x.1002 + y.100 + z.1. Resumo Nesta aula, estudamos a representao dos nmeros naturais numa base qualquer e os critrios de divisibilidade, que em alguns casos nos permitem decidir se um determinado inteiro ou no divisvel por outro, sem precisar efetuar a diviso.

Referncias

BURTON, David M. Elementary number theory. New York: McGraw-Hill, 1998. COUTINHO, S. C. Nmeros inteiros e criptografia RSA. Rio de Janeiro: Instituto de Matemtica Pura e Aplicada IMPA/ Sociedade Brasileira de Matemtica SBM, 1997. CRAWFORD, Mathew. Introduction number theory: the art of problem solving. Alphine: AoPS Incorporated, 2006. EARL, M. James; SALKIND, Charles T. The contest problem book III. Washington: The Mathematical Association of American, 1973. HEFEZ, Abramo. Elementos de aritmtica. Rio de Janeiro: Sociedade Brasileira de Matemtica, 2005. SALKIND, Charles T. The contest problem book II. Washington: The Mathematical Association of American, 1966

74

Aula 7 Congruncias
Apresentao

Nesta aula, estudaremos o conceito de congruncia, que revolucionou o estudo da Aritmtica, permitindo tratar as questes de divisibilidade com um enfoque mais fcil e mais eficiente. Foi Carl Friederich Gauss (1777- 1855) quem introduziu este conceito, em 1801, no seu livro Disquisitiones aritmeticae (Investigaes na Aritmtica). Gauss escreveu este livro quando ele tinha apenas 24 anos. Tente entender tudo que est sendo explicado na aula. Estude com caneta e papel ao lado. Leia com ateno. Se for preciso, leia vrias vezes uma linha ou um pargrafo. Seja paciente e procure ter certeza que voc entendeu o que (e por que) est fazendo.

Objetivos

Decidir quando dois nmeros so congruentes. Aplicar corretamente as propriedades de congruncia. Decidir se um dado nmero inteiro positivo divide outro nmero inteiro, usando propriedades das congruncias Resolver congruncias lineares.

A definio de congruncia

Dados os nmeros inteiros 33, 18 e 3, temos que 33 18 = 15 = 3 x 5. Portanto, a diferena 33 18 divisvel por 3. Dizemos ento que 33 cngruo a 18 mdulo 3. Johann Carl Friedrich Gauss (1777-1855), conhecido como Gauss, famoso matemtico, astrnomo e fsico alemo, sugeriu uma notao para esse fato, a qual ficou mundialmente aceita: 33 18 (mod 3) 33 18 divisvel por 3. L-se: 33 cngruo a 18 mdulo 3 se, e somente se, a diferena 33 18 divisvel por 3. De um modo equivalente, escrevemos: 33 18 (mod 3) 33 18 = 3k, com k Z. Dizemos, tambm, que 33 e 18 so cngruos entre si mdulo 3. Vejamos outro exemplo: 27 e 13 so cngruos mdulo 7 (e tambm cngruos mdulo 2), pois a diferena 27 13 = 14 = 2 x 7. Ou seja, 13 27 (mod 7) e 13 27 (mod 2). fcil ver que: 18 -1 (mod 19);

75

31 1 (mod 2); -3 4 (mod 7); 12 0 (mod 3).

Exemplo 1 Mostre que quaisquer dois nmeros inteiros pares so cngruos entre si mdulo 2. Soluo

Sejam 2m e 2n dois nmeros pares. A diferena 2m - 2n = 2.(m n). Portanto, divisvel por 2. Na notao de Gauss: 2m 2n (mod 2), para quaisquer m e n inteiros.
Exerccio 1 Mostre que quaisquer dois nmeros inteiros mpares so cngruos entre si mdulo 2.

De uma maneira geral, se a e b so dois nmeros inteiros quaisquer e n um nmero inteiro maior do que ou igual a 2, dizemos que a cngruo a b mdulo n se a b = nk, com k um nmero inteiro. Na notao de Gauss: a b(mod n) a b = nk, com k um nmero inteiro. Uma propriedade muito til, que decorre imediatamente da definio de congruncia, a seguinte:
Proposio 1 Dizer que a b (mod n ) equivalente a dizer que diviso por n. Demonstrao

a e b deixam o mesmo resto na

Se a b (mod n ), ento, a b = n.k, onde k um inteiro. Fazendo a diviso de a e b por n, temos: a = q1.n + r1, onde q1 e r1 so nmeros inteiros, com 0 r1 < n, b = q2.n + r2, onde q2 e r2 so nmeros inteiros, com 0 r2 < n. Suponha que r2 r1. Nesse caso, 0 r1 - r2 r1 < n. Agora, fazemos a diferena: a b = (q1 q2)n + (r1 r2) (*) Desse modo, 0 r1 - r2 < n o resto da diviso de a b por n. Logo, r1 - r2 = 0, pois a b mltiplo de n. Reciprocamente, se a e b deixam o mesmo resto na diviso por n, ento: a = q1.n + r, onde q1 e r so nmeros inteiros, com 0 r < n, b = q2.n + r, onde q1 e r so nmeros inteiros, com 0 r < n. Da, segue que a b = (q1 q2)n. Portanto, a diferena a b um mltiplo de n, o que equivale a dizer que a b (mod n ).

76

Quando dois nmeros, a e b, no so cngruos (ou no so congruentes) mdulo n, dizemos que a e b so incongruentes (ou incngruos) mdulo n.

Propriedades bsicas das congruncias

Se a, b, c e d so inteiros quaisquer e n um inteiro maior do que ou igual a 2, so verdadeiras as seguintes propriedades: I. a a (mod n) (Reflexividade) II. Se a b (mod n), ento b a (mod n) (Simetria) III. Se a b (mod n) e b c (mod n), ento, a c (mod n) (Transitividade) IV. Se a b (mod n) e c d (mod n), ento, a +c b + d(mod n) (Soma) V. Se a b (mod n) e c d (mod n), ento, a - c b - d (mod n) (Diferena) VI. Se a b (mod n) e c um inteiro no negativo, ento, ac bc (mod n) VII. Se a b (mod n) e c d (mod n), ento, a .c b.d (mod n) (Produto) VIII. Se a b (mod n) e k um inteiro positivo, ento, ak bk (mod n) (Potncia) IX. Se a + c b + c (mod n), ento, a b (mod n) (Cancelamento para a soma) Voc pode verificar facilmente que as propriedades anteriores so de fato verdadeiras. Por exemplo, para verificar a veracidade da propriedade IV, temos que mostrar que a diferena (a + c) (b + d) mltiplo de n. Para isso, basta observar que: (a + c) (b + d) = (a b) + (c d) e que (a b) e (c d) so, por hiptese, ambos mltiplos de n. Logo, (a + c) (b + d) um mltiplo de n, como soma de dois mltiplos de n. Portanto, a + c b + d (mod n) . Para verificar a veracidade da propriedade VII, temos que mostrar que a.c - b.d um mltiplo de n. Para isso, basta observar que: a .c - b.d = a.c b.c + b.c b.d = (a b).c + b.(c d). Como (a b) e (c d) so, por hiptese, ambos mltiplos de n, segue que cada uma das duas parcelas, (a b).c e b.(c d), so mltiplos de n. Assim, a .c - b.d um mltiplo de n. Portanto, a .c b.d (mod n). Para verificar a propriedade VIII, basta aplicar a propriedade VII usando a congruncia a b (mod n) k vezes. A propriedade IX, de fcil verificao, o cancelamento para a soma. Exemplificando, como 15 + 4 9 + 4 (mod 6), segue que 15 9 (mod 6).

Exemplo 2

77

Ana, Bernardo e Carla arrumam laranjas para vender na feira, colocando 12 laranjas em cada saco. Ana tinha 389 laranjas, Bernardo 188 e Carla 97. Depois de arrumar todas as laranjas nos sacos, quantas sobraram ao todo? Soluo Para responder, temos que observar que precisamos considerar, para cada um deles, a quantidade de laranjas mdulo 12. Como 389 5 ( mod 12); 188 8 (mod 12) e 97 1 (mod 12), quando Ana terminou de arrumar as laranjas nos sacos, sobraram 5 laranjas, das laranjas de Bernardo sobraram 5 e das de Carla sobrou 1. Portanto, no final sobraram 5 + 8 + 1 = 14 laranjas. Mas, 14 2 (mod 12). Isso significa que eles, em conjunto, poderiam completar mais um saco com 12 laranjas e sobrariam apenas 2 laranjas.
Exerccio 2 Encontre todos os inteiros n tal que -100 n 100, e n 7 (mod 19). Exemplo 3 Sejam a, b e c nmeros inteiros positivos cujos restos na diviso por 8 so 3, 5 e 1, respectivamente. Ache o resto da diviso de (a + b + c) por 8. Soluo

Temos que a 3 (mod 8), b 5 (mod 8) e c 1 (mod 8). Somando membro a membro as trs congruncias, obtemos (a + b + c) 3 + 5 + 1 (mod 8) . Ou seja, (a + b + c) 9 (mod 8). Como 9 1 (mod 8), segue que (a + b + c) 1 (mod 8) .
Uma pergunta: nas congruncias, vale o cancelamento para o produto?

Ou seja, se ac bc (mod n), ento, a b (mod n)?


Resposta: fcil obter exemplos onde no se verifica a propriedade. Por exemplo, 3 x 23 3 x 8 (mod 9) verdadeiro, enquanto 23 8 (mod 9) no verdadeiro, pois 23 8 = 15, que no divisvel por 9. Concluso: de uma maneira geral, no podemos aplicar o cancelamento numa situao como ac bc (mod n), para obter a b (mod n). O resultado correto seria:

Proposio 2 Se ac bc (mod n), ento, a b (mod n/d), onde d = MDC (c, n). Demonstrao

Se ac bc (mod n), existe um inteiro k tal que ac bc = kn. Agora, seja d = MDC(c, n). Dividindo por d ambos os lados da igualdade ac bc = kn, obtemos (a b)(c/d) = k(n/d). Por outro lado, como d = MDC(c, n), existem c n nmeros inteiros x e y tais que cx + ny = 1. Logo, x + y = 1 . Pelo Corolrio 2, da d d

78

aula 5 O mximo divisor comum, o mnimo mltiplo comum e as equaes c n e so primos entre si. diofantinas lineares , segue que d d Como MDC (c/d, n/d) = 1, ento, n/d divide a b, que o mesmo que dizer n a b (mod ). d

Corolrio Seja ac bc (mod n). Se c e n so primos entre si, ento, a b (mod n). Isto , nessas hipteses vale a lei do cancelamento para o produto. Demonstrao n n Se a e n so primos entre si, ento, MDC(n, c) = 1. Logo, = = n. MDC( n ,c ) 1 Exemplo 4 Verifique que 3 x 23 3 x 8 (mod 9) implica que 23 8 (mod 3). Soluo

Aplicando a propriedade anterior para 3 x 23 3 x 8 (mod 9), obtemos 23 8 (mod 9/3). Ou seja, 3 x 23 3 x 8 (mod 9) implica que 23 8 (mod 3).
Exerccio 2 Sejam a e b nmeros inteiros tais que 4a 4b (mod 15). Diga, justificando, se podemos concluir que a b (mod 15).

A seguir, vamos fazer alguns exemplos com os quais voc vai poder verificar como o uso do conceito de congruncia facilita a resoluo de certos problemas.

Exemplo 5 Sem efetuar a diviso, encontre o resto da diviso de 4100 por 5. Soluo

Como 4 (-1) = 4 + 1 = 5, podemos dizer que 4 -1 (mod 5). Aplicando a propriedade VIII, tomando k = 100, temos 4100 (-1)100 (mod 5). Ou seja, 4100 1 (mod 5). Portanto, o resto da diviso de 4100 por 5 o mesmo resto da diviso de 1 por 5. Mas, o resto da diviso de 1 por 5 1, pois 1 = 0 x 5 + 1. Logo, o resto da diviso de 4100 por 5 1.

Exemplo 6 A distribuio dos dias do ms num calendrio um exemplo do uso do conceito de congruncia. Vejamos o calendrio do ms de setembro do ano de 2008:

79

Setembro de 2008 D S T Q Q S 1 2 3 4 5 7 8 9 10 11 12 14 15 16 17 18 19 21 22 23 24 25 26 28 29 30

S 6 13 20 27

Observe a segunda coluna, a das segundas-feiras, ela comea com o 1 e todos os outros nmeros dessa coluna deixam resto 1 quando divididos por 7. A coluna da sexta-feira, comea com 5 e todos os outros nmeros deixam resto 2 quando divididos por 7. Ou seja, em todas as colunas os nmeros so cngruos entre si mdulo 7.

Exerccio 3 Encontre todos os inteiros n tal que: 100 n 200, e 3n 7 (mod 19). Exerccio 4 Descreva o modelo de congruncia para um relgio de ponteiros. Exemplo 7 Dado um inteiro qualquer n, podemos afirmar que o nmero (n2 + 1) no divisvel por 3. Soluo

De fato, na diviso de um nmero inteiro por 3, os possveis restos so 0, 1 ou 2. Desse modo, acontece uma, e s uma, das seguintes possibilidades: ou n 0 (mod 3), ou n 1 (mod 3), ou n 2 (mod 3). Se n 0 (mod 3), ento, n2 0 (mod 3), aplicando a propriedade VIII. Da, segue que n2 + 1 1 (mod 3). Portanto, nesse caso, n2 + 1 deixa resto 1 quando dividido por 3. Se n 1 (mod 3), ento, n2 + 1 2 (mod 3). Nesse caso, n2 + 1 deixa resto 2 quando dividido por 3. Se n 2 (mod 3), ento, n2 22 (mod 3), que o mesmo que n2 1 (mod 3). Logo, podemos dizer que n2 + 1 2 (mod 3). Nesse caso, na diviso por 3 o nmero n2 + 1 deixa resto 1. Portanto, para todo n, n2 + 1 no divisvel por 3.
Exemplo 8 Diga, justificando, se o nmero 3099 + 61100 um nmero divisvel por 31. Soluo Observe que 30 -1 (mod 31). Portanto, 3099 (-1)99 (mod 31). Logo, 3099 -1 (mod 31). Por outro lado, 61 -1 (mod 31). Portanto, 61100 (-1)100 (mod 31). Logo, 61100 1 (mod 31). Assim, 3099 + 61100 -1 + 1 (mod 31), que o mesmo que 3099 + 61100 0 (mod 31). Portanto, 3099 + 61100 um nmero divisvel por 31. Exemplo 9 Mostre que o nmero 43101 + 23101 divisvel por 66.

80

Soluo

De fato, como 66 = 6 x 11, ento, um nmero divisvel por 66 se, e somente se, divisvel simultaneamente por 6 e 11. Agora, 43 1 (mod 6) e 23 -1 (mod 6). Portanto, usando propriedades bsicas das congruncias, podemos dizer que: 43101 1 (mod 6) e 23101 -1 (mod 6). Somando ambas as congruncias, obtemos 43101 + 23101 1 + (-1) (mod 6), que o mesmo que 43101 + 23101 0 (mod 6). Assim, 43101 + 23101 divisvel por 6. Resta mostrar que 43101 + 23101 divisvel por 11. Para isso, observe que 43 -1 (mod 11) e 23 1 (mod 11). Portanto, podemos dizer que 43101 -1 (mod 11) e 23101 1 (mod 11). Logo, 43101 + 23101 0 (mod 11), que o mesmo que dizer que 43101 + 23101 divisvel por 11. Portanto, como 43101 + 23101 divisvel simultaneamente por 6 e por 11, ento, 43101 + 23101 divisvel por 66.
Exerccio 5 Sejam a e n dois nmeros inteiros. Prove que: (n a)2 a2 (mod n). Exemplo 10 Diga, justificando, se existe um nmero natural n tal que o nmero (n2 + n + 1) seja divisvel por 55. Soluo

Inicialmente, observe que um nmero divisvel por 55 se ele o for por 5 e 11 ao mesmo tempo, pois 55 = 5 x 11. Por outro lado, dado um nmero natural n, uma e s uma das afirmaes abaixo verdadeira: ou n 0 (mod 5) ou n 1 (mod 5) ou n 2 (mod 5) ou n 3 (mod 5) ou n 4 (mod 5). Agora, observe que: se se se se se n 0 (mod 5), ento, n 1 (mod 5), ento, n 2 (mod 5), ento, n 3 (mod 5), ento, n 4 (mod 5), ento, (n2 + n + 1) 1 (mod 5); (n2 + n + 1) 3 (mod 5); (n2 + n + 1) 2 (mod 5); (n2 + n + 1) 3 (mod 5); (n2 + n + 1) 1 (mod 5).

Portanto, em nenhuma dos casos possveis (n2 + n + 1) 0 (mod 5). Logo, no existe um nmero natural n tal que o nmero (n2 + n + 1) seja divisvel por 55.
Exemplo 11 Ache o dgito das unidades do nmero 3100. Soluo

Suponha que a representao decimal de 3100 seja c0 + c110 + c2 102 + c3103 + ... + cn10n, com c0, c1, c2, ..., cn inteiros no negativos, todos menores do que 10. O que queremos encontrar o valor de c0. Na linguagem das congruncias, 3100 c0 (mod 10). Agora, observe que 32 -1 (mod 10). Logo, 3100 = (32)50 (-1)50 (mod 10). Portanto, podemos escrever 3100 1 (mod 10) . Assim, o dgito das unidades de 3100 1.

Exerccio 6 Seja A = 3105 + 4105.

81

(a) Mostre que 7 divide A.

(b) Encontre o resto da diviso de A por 11.

Exemplo 12 Mostre que 7 divide o nmero 22225555 + 55552222. Soluo

fcil ver que: 2222 3 (mod 7), 5555 4 (mod 7) e 35 5 (mod 7). Usamos as propriedades bsicas das congruncias para escrever: 22225555 35555 (mod 7), 55552222 42222 (mod 7), 35555 = (35)1111 51111 (mod 7), 42222 = (42)1111 (-5)1111 (mod 7) - 51111 (mod 7). Portanto, 22225555 + 55552222 35555 + 42222 (mod 7) 51111 51111 (mod 7) 0 (mod 7). Logo, 7 divide o nmero 22225555 + 55552222.
Exemplo 13 Encontre todos os quadrados mdulo 13. Soluo

Pelo algoritmo da diviso, o resto da diviso de n por 13 varia de 0 a 12, como n2 (13 n)2 (mod 13), s precisamos nos preocupar com os quadrados dos nmeros no negativos de 0 at 6. Desse modo, 02 0 (mod 13); 12 1 (mod 13); 22 4 (mod 13); 32 9 (mod 13); 42 3 (mod 13); 52 12 (mod 13); 62 10 (mod 13). Portanto, os quadrados mdulo 13 so: 0, 1, 3, 4, 9, 10 e 12 .

Exemplo 14 Usando o conceito de congruncia, demonstre que: um nmero natural K divisvel por 9 se, e somente se, na sua representao na base 10, a soma de seus dgitos um nmero divisvel por 9. Soluo

Seja K = cn10n + cn-110n-1 + .... + c3103 + c2 102 + c110 + c0, com cn, cn-1, cn-2, ..., c0 inteiros no negativos, todos menores do que 10, a representao de K na base 10. Agora observe que: 10 1 (mod 9) e 10i 1 (mod 9). Dessa forma, temos que: K = cn10n + cn-110n-1 + .... + c3103 + c2 102 + c110 + c0 cn + cn-1 + cn-2 + ...+ c0 (mod 9), o que demonstra a afirmao.

82

Exerccio 7 Mostre que se a2 b2 (mod p), onde p um nmero primo, ento, ou p divide a + b ou p divide a b.

Um sistema completo de restos mdulo n um conjunto com n nmeros inteiros de tal forma que dois quaisquer deles no sejam congruentes mdulo n. Por exemplo, o conjunto S = {1, 2, 3, ...., n 2, n 1} um sistema completo de resto mdulo n.

Exemplo 15 Verifique que o conjunto {3, 13, 35} um sistema completo de restos mdulo 3. Soluo

Basta ver que 3 0 (mod 3), 13 1 (mod 3) e 35 2 (mod 3).


Exerccio 8 Verifique que o conjunto S = {-14, 20, 22, 32, 47, 86, 143} um sistema completo de restos mdulo 7. Congruncias lineares

Uma congruncia linear uma equao da forma ax b(mod n), onde a, b, n Z , com n > 1. Uma soluo de uma equao desse tipo um inteiro xo para o qual axo b (mod n). Agora, observe que axo b (mod n) se, e somente se, n divide axo - b. Ou seja,
axo b (mod n) axo b = ny, para algum y Z.

Desse modo, o problema de encontrar todas as solues de uma congruncia linear idntico ao de obter todas as solues da equao diofantina axo ny = b. Na prtica, tratamos como iguais duas solues quaisquer da congruncia ax b(mod n) que no so cngruas mdulo n, mesmo que elas no sejam iguais no sentido tradicional. Por exemplo, x = 2 e x = 7 satisfazem a congruncia linear 4x 3(mod 5). Como 2 7 (mod 5), tratamos 2 e 7 como a mesma soluo da congruncia linear 4x 3(mod 5). Ou seja, quando falamos do nmero de solues da congruncia linear axo b (mod n), estaremos contando somente aquelas que so incongruentes mdulo n.
Uma questo: quando a congruncia linear axo b (mod n) admite soluo? E,nesse caso, quantas so as solues incongruentes mdulo n? A resposta dada pelo teorema seguinte. Teorema 1 A congruncia linear axo b (mod n) admite soluo se, e somente se, d divide b, onde d = MDC(a, n). Alm disso, se d divide b, ento, a congruncia admite d solues mutuamente incongruentes mdulo n. Demonstrao

83

Observe que a congruncia linear dada equivalente equao diofantina ax ny = b. Foi visto na aula 5 que a equao diofantina admite soluo se, e somente se, d = MDC(a, n) divide b. Alm disso, as solues da equao diofantina so da forma n a x = xo + t e y = y o + t , com t um nmero inteiro. d d n Agora, dentre os inteiros x satisfazendo x = xo + t , considere aqueles para os quais t d toma os valores seguintes t = 1, 2, 3, ..., d 1. Ou seja, considere os d inteiros: xo , xo + 2n 3n 4n (d 1)n n , xo + , xo + , xo + , ..., xo + . d d d d n

Para concluirmos a prova, vamos mostrar que esses inteiros so dois a dois incongruentes mdulo n e qualquer inteiro x, que soluo da congruncia dada, congruente a algum dos inteiros acima. n n De fato, se xo + i xo + j (mod n) , com i, j {0, 1, 2, ...., d -1}, com i j. Assim, d d n n n n podemos escrever i j (mod n) . Agora, observe que MDC ( , n) = e, portanto, d d d d n pode ser cancelado na ltima congruncia obtida. Ou seja, depois do cancelamento, d teremos i j(mod d). Isso o mesmo que dizer que d divide (i j). Mas, isso impossvel, pois 0 < i j < d. Para concluir, resta mostrar que qualquer outra soluo da congruncia linear, n x = xo + t , congruente a algum dos d inteiros listados acima. Para isso, utilizamos o d Algoritmo da Diviso, dividindo t por d: t = qd + r, onde 0 r < d. Assim, podemos n n n n x = xo + t = xo + (qd + r ) = xo + nd + r xo + r (mod n) , onde escrever: d d d d n x = xo + r sendo um dos d nmeros acima. O que conclui a prova. d
Exemplo 16 Encontre todas as solues da congruncia linear 6x 21 (mod 51). Soluo

As solues da congruncia linear 6x 21 (mod 51) podem ser obtidas atravs da equao diofantina 6x 51y = 21, que admite soluo, pois MDC(6, 51) = 3 divide 21. O teorema anterior garante a existncia de 3 solues incongruentes mdulo 21. Por outro lado, uma soluo da equao diofantina seria xo = -56 e yo = -7. Ou seja, as solues da congruncia linear 6x 21 (mod 51) so: x = 56 + 51 t 46 + 17t (mod 51) , com t = 0, 1, 2. 3

Assim, as solues so: x 46 (mod 51), x 12 (mod 51) e x 29 (mod 51).

84

Exerccio 9 Encontre todas as solues da congruncia linear 3x 1 (mod 5).

Exerccios
1) Resolva as seguintes congruncias lineares (a) 10x 5 (mod 12) (b) 131x 21 (mod 77)

2) Qual o resto da diviso de 19385 por 31? 3) Encontre o resto da diviso de: (a) 15 + 25 + 35 + ... + 1005 por 4. (b) 1! + 2! + 3! + .... + 100! por 15.

4) Encontre todos os inteiros n tal que: 100 n 200, e 3n 7 (mod 19). 5) Ache um sistema completo de restos mdulo 11 composto de mltiplos de 7.

Resumo Nesta aula, estudamos a noo de congruncia, introduzida por Gauss para inovar o estudo da Aritmtica.

Referncias

BURTON, David M. Elementary number theory. New York: McGraw-Hill, 1998. COUTINHO, S. C. Nmeros inteiros e criptografia RSA. Rio de Janeiro: Instituto de Matemtica Pura e Aplicada IMPA/ Sociedade Brasileira de Matemtica SBM, 1997. DU SAUTOY, Marcus. A msica dos nmeros primos: a histria de um problema no resolvido. Rio de Janeiro: Jorge Zahar, 2008. HEFEZ, Abramo. Elementos de aritmtica. Rio de Janeiro: Sociedade Brasileira de Matemtica, 2005.

85

AULA 08 O Teorema Chins de Restos e o Pequeno Teorema de Fermat


Apresentao

Nesta aula, estudaremos a soluo de sistemas de congruncias lineares, baseado no que ficou conhecido como Teorema Chins de Restos. Tambm estudaremos as idias revolucionrias de Fermat, usadas hoje em dia para permitir o comrcio eletrnico confivel. Tente entender tudo que est sendo explicado na aula. Estude com caneta e papel ao lado. Leia com ateno. Se for preciso, leia vrias vezes uma linha ou um pargrafo. Seja paciente e procure ter certeza que voc entendeu o que (e por que) est fazendo.

Objetivos

Com esta aula espera-se que voc possa: Resolver sistemas de congruncias lineares, sob as condies do Teorema Chins de restos; Aplicar corretamente o Teorema de Fermat;

O TEOREMA CHINS DE RESTOS

Problemas antigos da astronomia, ligados aos movimentos peridicos dos corpos celestes, deram origem ao hoje conhecido como Teorema Chins de Restos. O nome veio do fato dos problemas terem sido originrios dos antigos matemticos chineses. H registros de problemas relacionados ao tema propostos no sculo terceiro depois de Cristo.
Teorema 2 (O Teorema Chins de Restos) Sejam n1, n2,n3, ..., nk nmeros inteiros positivos tais que MDC(ni, nj) = 1, para i j. O sistema de congruncias lineares

x a1 (mod n1) x a2 (mod n2) x a3 (mod n3) ....................... x ak (mod nk) admite uma soluo simultnea, que nica mdulo o inteiro n = n1n2n3...nk.

86

Demonstrao

Seja n = n1n2n3...nk. Para cada r = 1, 2, 3, ..., k, seja Nr =

n = n1n2n3..nr-1.nr+1...nk. Isto nr , Nr o produto de todos os ni, exceto o nr. Como MDC(ni, nj) = 1, a congruncia Nrx 1 (mod nr)

admite uma nica soluo, que chamaremos de xr, pois MDC(Nr, nr) = 1, e divide 1. A soluo do sistema ser:
x = a1 N 1 x1 + a 2 N 2 x 2 + a3 N 3 x3 + ... + a k N k x k

De fato, basta observar que: (i) Ni 0 (mod nr), para i r, pois nr divide Ni (ii) x = a1 N 1 x1 + a 2 N 2 x 2 + a3 N 3 x3 + ... + a k N k x k a r N r x r (mod nr ) (iii) Como escolhemos x a r .1 a r (mod nr ) xr satisfazendo Nrx 1 (mod nr), temos, necessariamente,

Resta-nos mostrar que a soluo nica mdulo n = n1n2n3...nk. Suponha que exista outra soluo x. Isto , x a r (mod nr ) x ' , para r = 1, 2, 3, ..., k. Assim, nr divide
x x ' , para cada valor de r. Como MDC(ni, nj) = 1, temos, obrigatoriamente, que n = n1n2n3...nk divide x x ' . Portanto, x x ' (mod n) , o que conclui a prova do Teorema Chins de Restos.

EXEMPLO 16 Use o Teorema Chins de Restos para resolver o seguinte sistema de congruncias lineares: x 2 (mod 3) x 3 (mod 5) x 2 (mod 7) Soluo Na notao do Teorema Chins de Restos, temos:

a1 = 2, a2=3, a3 = 2; n1 = 3, n2 = 5, n3 = 7; n = 3 . 5. 7 = 105; N1 = 5 . 7 = 35, N2 = 3 . 7 = 21, N3 = 3 . 5 = 15. As congruncias N1x1 1 (mod 3), N2x2 1 (mod 5) e N3x3 1 (mod 7), so: 35x1 1 (mod 3), que o mesmo que 2x1 1 (mod 3), cuja soluo x1 2 (mod 3); 21x2 1 (mod 5), que o mesmo que x2 1 (mod 5); 15x3 1 (mod 7), que o mesmo que x3 1 (mod 7). Portanto, a soluo do sistema dada por

87

x = 2. 35 . 2 + 3.21.1 + 2 .15 .1 233 (mod 105) 23 (mod 105).


ATIVIDADE 10 Use o Teorema Chins de Restos para resolver o seguinte sistema de congruncias lineares: x 1 (mod 12) x 1 (mod 5) x 0 (mod 7) EXEMPLO 17 (Antigo problema Chins) Uma senhora transportava um cesto de ovos. Assustada por um cavalo que galopava perto dela deixa cair o cesto e todos os ovos se partem. Quando lhe perguntaram quantos ovos tivera o cesto, respondeu dizendo que muito fraca em aritmtica, mas lembra-se de ter contado os ovos de dois em dois, de trs em trs, de quatro em quatro e de cinco em cinco, e tivera sobra de 1, 2, 3, e 4 ovos, respectivamente.

Ache a menor quantidade de ovos que o cesto inicialmente poderia ter. Soluo Seja x a quantidade de ovos que estavam inicialmente no cesto. Podemos escrever: x 1 (mod 2) x 2 (mod 3) x 3 (mod 4) x 4 (mod 5) Na notao do Teorema Chins de Restos, temos: a1 = 1, a2=2, a3 = 3, a4 = 4; n1 = 2, n2 = 3, n3 = 4, n4 = 5; No podemos aplicar diretamente o Teorema Chins de Restos, pois MDC(n1, n2) = MDC(2, 4) = 1. Para resolver o problema, inicialmente, trabalhamos somente com as congruncias lineares x 1 (mod 2) x 2 (mod 3) x 4 (mod 5) Agora, na notao do Teorema Chins de Restos o sistema acima tem os seguintes dados: a1 = 1, a2=2, a3 = 4; n1 = 2, n2 = 3, n3 = 5; n = 2 . 3. 5 = 30, N1 = 3 . 5 = 15, N2 = 2 . 5 = 10, N3 = 2 . 3 = 6. As congruncias N1x1 1 (mod 2), N2x2 1 (mod 3) e N3x3 1 (mod 5), so: 15x11(mod 2), que o mesmo que 1x1 1 (mod 2), cuja soluo x1 1 (mod 12), 10x2 1 (mod 3), que o mesmo que x2 1 (mod 3), cuja soluo x2 1 (mod 3) 6x3 1 (mod 5), que o mesmo que x3 1 (mod 5), cuja soluo x3 1 (mod 7)

88

Portanto a soluo do sistema dada por x = 1.15 .1 + 2.10.1 + 4.6 .1 59 (mod 30) 29 (mod 30) x = 29 + 30k , onde k um nmero inteiro. Agora, substitumos x na Ou seja, congruncia x 3 (mod 4). Assim, 29 + 30 k 3 (mod 4), que o mesmo que 1 + 2k 3 (mod 4). Ou ainda: 3 + 1 + 2k 3 + 3 (mod 4), que nos leva para 2k 2 (mod 4), que equivalente a dizer 2k 2 = 4t, onde t um inteiro. Ou seja, 2 (k 1) = 4t. Portanto, k tem de ser um nmero mpar, k = 2s + 1, onde s um nmero inteiro. Portanto, x = 29 + 30(2s + 1) = 59 + 60s, Deste modo, o nmero mnimo de ovos que a cesta inicialmente poderia conter 59.

O PEQUENO TEOREMA DE FERMAT

O mais famoso teorema de Fermat conhecido como o ltimo Teorema de Fermat: (ltimo Teorema de Fermat) Se n um inteiro maior do que 2, a equao xn + yn = zn no admite soluo x, y e z no conjunto dos nmeros inteiros maiores do que 1.
Pierre de Fermat (1601-1665) foi um matemtico e cientista francs. Seu pai, Dominique de Fermat, era um rico mercador de peles e lhe propiciou uma educao privilegiada, inicialmente no mosteiro franciscano de Grandselve e depois na Universidade de Toulouse. Ingressou no servio pblico em 1631. Em 1652 ele foi promovido para Juiz Supremo na Corte Criminal Soberana do Parlamento de Toulouse, todavia esta promoo se deu em ocorrncia da chegada da praga, que levou a vida de grande parte da populao da Europa. Neste mesmo ano Fermat tambm adoeceu e chegou-se a afirmar que ele havia morrido, entretanto ele se recuperou e permaneceu vivo por mais de uma dcada. Sua morte, de fato, deu-se a 12 de Janeiro de 1665, em Castres.

Em razo de seu cargo, Fermat no podia ter muitos amigos para no ser acusado de favoritismo em seus julgamentos, tambm em razo da tumultuada fase que passava a Frana de ento, com o Cardeal Richelieu sendo primeiro-ministro. Ao se investigar a produo matemtica de Fermat, percebe-se facilmente a caracterstica amadora predominante em seus trabalhos. Na verdade, com pouqussimas excees, ele no publicou nada em vida e nem fez qualquer exposio sistemtica de suas descobertas e de seus mtodos, tinha as questes da matemtica mais como desafios a serem resolvidos. Considerado o Prncipe dos amadores, Pierre de Fermat nunca teve formalmente a matemtica como a principal atividade de sua vida. Jurista e magistrado por profisso, dedicava Matemtica apenas suas horas de lazer e, mesmo assim, foi considerado por Pascal o maior matemtico de seu tempo. Contudo, seu grande gnio matemtico perpassou vrias geraes, fazendo com que vrias mentes se debruassem com respeito sob o seu legado, que era composto por

89

contribuies nas mais diversas reas das matemticas, as principais: clculo geomtrico e infinitesimal; teoria dos nmeros; e teoria da probabilidade. Entre os estudiosos com os quais mantinha contato postal, esto: Sir Kenelm Digby, John Wallis, Nicholas Hensius, alm de Blaise Pascal, Assendi, Roberval, Beaugrand e o padre Marin Mersenne. O interesse desperto em Fermat pela Matemtica, possivelmente, deu-se com a leitura de uma traduo latina, feita por Claude Gaspar Bachet de Mziriac, de Aritmtica de Diophante, um texto sobrevivente da famosa Biblioteca de Alexandria, queimada pelos rabes no ano 646 d.C., e que compilava cerca de dois mil anos de conhecimentos matemticos. A matemtica do sculo XVII estava ainda se recuperando da Idade das Trevas, portanto no de se admirar o carter amador dos trabalhos de Fermat. No entanto, se ele era um amador, ento era o melhor deles, devido preciso e importncia de seus estudos, que, diga-se ainda, estavam sendo realizados longe de Paris, o nico centro que abrigava grandes matemticos, mas at ento ainda no prestigiados estudiosos da Matemtica, como Pascal, Gassendi, Mersenne, entre outros. O padre Marin Mersenne teve um papel importante na histria da matemtica francesa do sculo XVII e tambm foi uma das poucas amizades de Fermat. Todavia, interessante observar mais de perto o desenvolvimento da Matemtica nesta poca. Diferentemente da famosa escola pitagrica, os franceses no tinham o costume de trocar com os colegas os avanos recentes de suas pesquisas, devido influncia dos cosistas do sculo XVI, italianos que utilizavam smbolos para representar quantidades desconhecidas. Mersenne tinha o costume, desagradvel para seus contemporneos matemticos, de divulgar os trabalhos dos pesquisadores. Em suas viagens pela Frana e por pases estrangeiros, acabou conhecendo Fermat e trocando com ele vrias correspondncias. No entanto, mesmo com a insistncia do padre, Fermat no publicou nada. (Fonte:http://pt.wikipedia.org/wiki/Pierre_de_Fermat) Contam os historiadores que, em 1637, Fermat afirmou que tinha uma prova para a proposio que ficou conhecida com o ltimo Teorema de Fermat. Ele escreveu sua afirmao nas margens do livro de Diofanto, Arithmeticae, uma verso feita por Claude Gaspar Bachet (15811683). Ele afirmou: Tenho uma prova maravilhosa para esta proposio, mas a margem muito pequena para cab-la. Muitos matemticos tentaram, sem sucesso, uma prova: Euler, Gauss, Dirichlet, Legendre, Lam, Kummer, Dedekind etc. Em setembro de 1994, o matemtico Andrew Wiles, de Princeton, e seu estudante Richard Taylor concluram uma prova usando fatos sobre curvas elpticas, que est muito acima do nvel desta aula. Portanto, nesta aula no trataremos do ltimo Teorema de Fermat. Estudaremos, em vez dele, o Pequeno Teorema de Fermat. Numa carta para Bernard Frenicle de Bessy (16051675), datada de 18 de outubro de 1640, Pierre de Fermat (1601 1665) deu sua verso do que hoje conhecemos como Pequeno Teorema de Fermat. Ele descobriu algo surpreendente e que foi usado para a criao do sistema RSA, j comentado na Aula 05.

90

Fermat descobriu que se voc, por exemplo, calcular as potncias de 2 em uma calculadora comum e verificar o resto na diviso por 7, estes restos tm um padro: comeando com 20, aps 6 clculos consecutivos o resto volta e ser 1, veja a tabela a seguir: Tabela 1 As potncias de 2 e seus restos na diviso por 7.
Potncia de 2 20 21 22 23 24 25 26 27 28 29 210 211 212 Visor da 1 2 4 8 16 32 64 128 256 512 1024 2048 4096 Calculadora 3 1 2 4 1 Resto da 1 2 4 1 2 4 1 2 Diviso por 7

Fermat, ainda viu que este padro se mantinha se ele substitusse 7 por qualquer nmero primo, enunciando o seguinte:
Teorema 3 (Pequeno Teorema de Fermat) Se p um nmero primo e a um inteiro que no divisvel por p, ento

ap-1 1 (mod p)
Demonstrao

Consideremos os primeiros p 1 mltiplos inteiros positivos de a: a, 2a, 3a, 4a, ..., (p -2)a, (p 1)a. Inicialmente, vamos provar que nenhum desses mltiplos de a cngruo a qualquer outro mdulo p, nem congruente a zero mdulo p. Para isso, vamos supor que existam nmeros inteiros r e s, com 1 r < s p 1 satisfazendo ra as (mod p). Como MDC(p, a) = 1, podemos cancelar a na congruncia acima e obter r s (mod p), com 1 r < s p 1, que uma contradio. Como os restos possveis na diviso por p so 1, 2, 3, ..., p 1, os nmeros a, 2a, 3a, 4a, ..., (p -2)a, (p 1)a tem de ser congruentes, em alguma ordem, a: 1, 2, 3, ..., p 1. Agora, multipliquemos essas congruncias membro a membro para obter a. 2a .3a. 4, ... (p -2)a. (p 1)a 1. 2. 3 ... (p 2).(p -1) (mod p) Que o mesmo que escrever: ap-1. 1. 2. 3 ... (p 2).(p -1) 1. 2. 3 ... (p 2).(p -1) (mod p)

91

Ou seja,

ap-1.(p 1)! (p 1)! (mod p).

Agora, observe que p no divide (p 1)!, pois no desenvolvimento de (p 1)! o nmero p no comparece. Portanto, na ltima congruncia, podemos cancelar (p -1 )!, obtendo ap-1 1 (mod p), como queramos.
EXEMPLO 18 Use o Pequeno Teorema de Fermat para verificar que 17 divide 11104 + 1. Soluo

17 um nmero primo e 11 no divide 17. Assim, pelo Pequeno Teorema de Fermat, 1116 1 (mod 17). Por outro lado, pelo Algoritmo da Diviso, 104 = 16 . 6 + 8. Assim, 11104 = (1116)6.118 (1)6.118 (mod 17) 118 (mod 17). Agora, observe que: 112 = 121 2 (mod 17) e 118 = (112)4 24 (mod 17) - 1 (mod 17). Portanto, podemos afirmar que 11104 118 (mod 17) -1 (mod 17), que o mesmo que afirmar que 17 divide 11104 + 1.
ATIVIDADE 11 Usando o Pequeno Teorema de Fermat, provar que 1812 deixa resto 1 quando dividido por 7. EXEMPLO 19 Encontre o resto da diviso de 3600 por 7. Soluo

Pelo Pequeno Teorema de Fermat, podemos escrever 36 1 (mod 7). Assim, 3600 = (36)100 (1)100 (mod 7) 1 (mod 7). Portanto, o resto da diviso de 3600 por 7 1. ATIVIDADE 12 Ache o menor inteiro positivo n para o qual o nmero 2n-1 1 divisvel por 41.

EXEMPLO 20 Existe um inteiro positivo c menor do que 40, para o qual 41 divide 2c -1? Soluo

Vamos supor que exista. Pelo Princpio da Boa Ordem ou Princpio da Boa Ordenao, existe o menor desses nmeros. Chamemos d o menor inteiro positivo para o qual 41 divide 2d -1. Isto , 2d = 1 + 41t, onde t um nmero inteiro. Pelo Algoritmo da Diviso, 40 = qd + r, com 0 r < d. Agora, podemos escrever 2qd = (1 + 41t)q e, aplicando o Binmio de Newton, podemos desenvolver (1 + 41t)q e verificar que 2qd = (1 + 41t)q = 41m + 1, onde m um nmero inteiro. Mas, como 240 = 2qd + r = 1 + 41k, temos 1 + 41k = 2qd .2r = (1 + 41m). 2r. Usando o Algoritmo da Diviso, 2r = 41v + s, com 0 s < 41. Da segue que 1 + 41k = 2qd .2r = (1 + 41m). (41v + s) = 41b + s, o que implica s = 1. Portanto, 2r = 41v + s = 41v + 1 e 2r 1 divisvel por 41. Mas, 0 r < d e d o menor inteiro positivo com esta propriedade. Contradio. Portanto, r = 0 e d divide 40. Ento o nmero inteiro d tem de pertencer ao conjunto {1, 2, 4, 5, 8, 10, 20, 40}. Verificando esses valores, temos

92

21 1 = 1, que no divisvel por 41; 22 1 = 3, que no divisvel por 41; 4 2 1 = 15, que no divisvel por 41; 25 1 = 31, que no divisvel por 41; 28 1 = 255, que no divisvel por 41; 210 1 = 1023, que no divisvel por 41; 220 1 = 1048575 = 41 x 25575, que , portanto, divisvel por 41; 240 1 (mod 41). Deste modo, 20 o menor nmero tal que 220 1 divisvel por 41. O Pequeno Teorema de Fermat nos garante que: se p = 41, ento 240 1 divisvel por 41. O Exemplo 20 nos mostra que possvel que algum divisor d de p 1 tenha tambm esta propriedade: 2d 1 divisvel por 41. O Pequeno Teorema de Fermat pode ser enunciado de uma forma mais compacta, eliminando-se a condio de que o nmero primo p no divide a:
Corolrio 1 Se p um nmero primo, ento ap a (mod p), para todo nmero inteiro a. Demonstrao

De fato, se p divide a, ento ap 0 (mod p) a (mod p). Agora, se p no divide a, segue, pelo Pequeno Teorema de Fermat, que ap-1 1 (mod p). Multiplicando cada membro da congruncia por a, obtemos ap a (mod p).
EXEMPLO 21 Encontre o resto da diviso de 3102 por 101. Soluo

Pelo Corolrio do Pequeno Teorema de Fermat, temos 3101 3 (mod 101). Por outro lado, 3102 = 3101.3 3 . 3 (mod 101). Portanto, o resto da diviso de 3102 por 101 9.
ATIVIDADE 13 Encontre o resto da diviso de 8900 por 29. O TEOREMA DE WILSON Em 1770, Eduard Waring (17341798), matemtico ingls, afirmou em seu livro Medidationes algebraicae, que um de seus estudantes, John Wilson (1741 1793), conjecturou que, se p um nmero inteiro primo, ento p divide (p 1)! + 1. Mas, Wilson no conseguiu provar. O resultado foi provado por Legendre, em 1771, que provou tambm a recproca. Teorema 4 (Wilson) Se p um nmero primo, ento (p -1)! -1(mod p) Demonstrao

Se p = 2, teremos (2 1)! = 1 -1 (mod 2). Se p = 3, teremos (3 1)! = 2! = 2 -1 (mod 3).

93

Assim, nos casos em que p = 2 ou p = 3 a afirmao bvia. Suponha que p seja um primo qualquer maior do que 3. Suponha que a {1, 2, 3, ..., p - 2, p-1}. Considere a congruncia linear ax 1 (mod p). Como MDC(a, p) = 1, a congruncia admite uma nica soluo mdulo p. Assim, existe um nico inteiro b, com 1 b p 1, satisfazendo ab 1 (mod p) (*) Ou seja, existe o inverso de a mdulo p. Como p primo, temos que a = b, se e somente se, a = 1 ou a = p -1 . Para verificarmos este fato, basta observar que: a2 1 (mod p) a2 - 1 0 (mod p) (a 1).(a + 1) 0 (mod p). Portanto, ou a -1 0 (mod p) ou a + 1 0 (mod p), que equivalente a dizer: ou a 1(mod p) ou a p 1 (mod p). Para cada a pertencente ao subconjunto {2, 3, 4, ...., (p 2)}, existe b, seu p 3 inverso mdulo p, com a distinto de b. Deste modo, existem congruncia do tipo 2 (*). Multiplicando membro a membro todas elas, obtemos: 2.3.4....(p 3).(p-2) 1 (mod p) Ou ainda, (p 2)! 1 (mod p). Agora, multiplicando (p 1) de cada lado, obtemos: (p 1).(p 2)! (p 1).1 (mod p) -1 (mod p), como queramos.

EXERCCIOS

1) Um bando de 19 piratas roubam uma sacola com moedas de ouro. Quando eles tentaram dividir a fortuna em partes iguais, sobraram 3 moedas. Na discusso sobre quem ficava com as trs moedas que sobraram, um pirata foi morto. A seguir, na diviso das moedas em partes iguais entre os sobreviventes, sobraram 10 moedas. Novamente, surgiu uma disputa pela posse das dez moedas que sobraram e um pirata foi morto. Agora, o total das moedas foi distribudo, igualmente, entre os sobreviventes sem sobrar qualquer moeda. Qual o menor nmero de moedas que a sacola poderia conter? (Sugesto: Use o teorema Chins de Restos)) 2) Mostre que: (a) 7 divide 19411963 + 19631991. (b) 39 divide 53103 + 10353. 3) Qual o resto da diviso de 19385 por 31? 4) Mostre que: (a) 7 divide 19411963 + 19631991. (b) 39 divide 53103 + 10353.

94

5) Trs fazendeiros cultivavam junto todo o seu arroz e o dividiam igualmente entre si no tempo da colheita. Certo ano, cada um deles foi a um mercado diferente vender seu arroz. Cada um destes mercados s comprava arroz em mltiplos de um peso padro, que diferia em cada um dos mercados. O primeiro fazendeiro vendeu o seu arroz em um mercado onde o peso padro era 87 kg. Ele vendeu tudo o que podia e voltou para casa com 18 kg. O segundo fazendeiro vendeu todo o arroz que podia ser vendido em um mercado cujo peso padro era 170 kg e voltou para casa com 58 kg. O terceiro fazendeiro vendeu todo o arroz que podia em um mercado cujo peso era de 143 kg e voltou (ao mesmo tempo em que os outros dois) com 40 kg. Qual a quantidade mnima de arroz que eles podiam ter cultivado, no total? (Sugesto: Use o Teorema Chins de Restos) 6) Encontre o menor inteiro a maior do que 2 tal que 2 divide a, 3 divide a +1, 4 divide a + 2, 5 divide a + 3 e 6 divide a + 4. 7) Numa ilha tropical, 5 homens e um macaco passam o dia todo recolhendo coco. noite, quando todos dormem, um dos homens levanta-se e, sem avisar aos outros, resolve tomar sua parte. Para isso, divide os cocos em cinco pilhas iguais, restando um coco, que ele d ao macaco. Depois de esconder a sua parte, colocar os cocos restantes numa s pilha, vai dormir. Cada um dos homens levanta-se, sem avisar aos outros, e procede da mesma maneira, sendo que, toda vez que fazem a diviso da pilha em cinco partes iguais, sobra um coco, que dado ao macaco. Na manh seguinte, todos os homens levantam-se e dividem os cocos que restaram em cinco partes iguais, tendo sobrado, tambm, um coco, que dado ao macaco. Ache o nmero mnimo de cocos que poderia ter na pilha original. (Sugesto: Seja n a quantidade de cocos que cada homem recebeu na diviso da manh seguinte, onde a pilha era formada por 5n + 1 cocos. O quinto 5n + 1 homem que acessou a pilha na noite anterior pegou cocos. 4 Quantos cocos existia na pilha quando o quinto homem interveio?) 8) O mgico senta-se numa cadeira, de costas voltadas para a audincia. Algum pensa num nmero natural qualquer no superior a 105. Divide o nmero por 3 e diz o resto da diviso ao mgico. Em seguida, divide o nmero inicialmente pensado por 5 e fala o resto da diviso ao mgico. E, finalmente, divide o nmero pensado por 7 e diz o resto. O mgico, conhecendo apenas os trs restos, advinha o nmero pensado. Qual o truque? (Sugesto: Observe que MDC(3, 5, 7) = 1. Use o Teorema Chins de Restos) 9) Mostre que 39 divide o nmero 53103 + 10353 .

RESUMO

Nesta aula, estudamos a soluo de sistemas de congruncias lineares e vimos tambm s idias de Fermat, que so usadas hoje em dia, para tornar o comrcio eletrnico confivel.

95

REFERNCIAS

Burton, David M. Elementary Number Theory. The McGrawHill Companies, Inc. New York. USA. 1998 Coutinho, S. C. Nmeros Inteiros e Criptografia RSA. Instituto de Matemtica Pura e Aplicada IMPA & Sociedade Brasileira de Matemtica SBM. Rio de Janeiro. 1997 Du Sautoy, Marcus A Msica dos Nmeros Primos: A Histria de um Problema no Resolvido. Zahar. Rio de Janeiro. 2008 Hefez, Abramo Elementos de Aritmtica. Sociedade Brasileira Matemtica. Rio de Janeiro. 2005

96

AULA 09 A Funo de Euler

Apresentao
Nesta aula, estaremos interessados em determinar a quantidade de nmeros naturais relativamente primos com um nmero natural n e menores do que n. Em seguida, veremos como usar este fato para generalizar o Pequeno Teorema de Fermat, estudado na Aula 08. Tente entender tudo que est sendo explicado na aula. Estude com caneta e papel ao lado. Leia com ateno. Se for preciso, leia vrias vezes uma linha ou um pargrafo. Seja paciente e procure ter certeza que voc entendeu o que (e por que) est fazendo.

Objetivos
Espera-se que ao trmino desta aula voc seja capaz de: Saber calcular os valores da funo de Euler em um nmero inteiro positivo, conhecida sua decomposio em fatores primos;

1 2

Usar o Teorema de Euler em problemas do tipo: encontrar o resto da diviso de um nmero inteiro quando dividido por outro.

97

1. A FUNO DE EULER

Dado um nmero natural n importante saber a quantidade de nmeros naturais menores do que n e relativamente primos com n. Essa curiosidade nos remete definio da chamada funo de Euler: : N N, tal que n N (n) = a quantidade de nmeros naturais k < n, tais que k e n so primos entre si. Vejamos os seguintes exemplos de (n) para alguns valores particulares de n.
EXEMPLO 1 (1) = 1, (2) = 1 e (n) 2, para todo nmero natural n 3. Soluo

O nico nmero k 1 relativamente primo com 1 o prprio 1. Logo, (1) = 1. O mesmo vale para 2. Ou seja, o nico nmero relativamente primo com 2 e menor do que 2 o 1. Isso nos diz que (2) = 1. Agora, para n 3, temos que n - 1 2. Como dois nmeros consecutivos so sempre primos entre si, segue que, para n 3, n e n - 1 so relativamente primos e o mesmo acontece com 1 e n , o que d (n) 2.

EXERCCIO 1 Encontre os valores: (a) (8) (b) (11)

(c) (14)

Agora podemos ver algumas proposies de carter mais geral.


PROPOSIO 1 Dado um nmero natural n, ento: (n) = n 1 se, e somente se, n um nmero primo. Demonstrao

Suponha que (n) = n 1. Isso significa que todos os n 1 nmeros naturais menores do que n so relativamente primos com n . Logo, n no pode ser de composto num produto de fatores primos menores do que n . Ou seja, n um nmero primo. Reciprocamente, supondo n primo, ento todos os nmeros naturais menores do que n so relativamente primos com n. Mas, os nmeros naturais menores do que n so precisamente 1, 2, 3, 4, ...., n 1. Logo, (n) = n 1. Seguindo essa linha de raciocnio, a prxima pergunta saber se existe tambm uma frmula para calcular (n), onde n a potncia de um nmero primo. Tal frmula existe e dada pelo seguinte:
PROPOSIO 2 Se n = pr, onde p um nmero primo e r 1, ento (n) = pr-1(p 1).

98

Demonstrao

Como p um nmero primo, ento todos os nmeros inteiros positivos menores do que ou iguais a n = pr que no so relativamente primos com n so todos aqueles que tm alguma potncia de p como fator, a saber: os pr-1 nmeros 1.p, 2.p, 3.p, ..., pr-1.p = pr. Portanto, todos os outros nmeros inteiros variando de 1 a pr so relativamente primos com n, o que nos d (n) = pr - pr-1. Isto , (n) = pr-1(p 1).

COROLRIO Sejam r, s inteiros maiores do que ou iguais a 1 e p um nmero primo. Ento (pr). (ps) < (pr+s). Demonstrao

Pela Proposio, temos: (pr). (ps) = pr-1(p 1). ps-1(p 1) = pr+s- 2(p 1)2 = p 1 = p r + s 1 . p .( p 1) . Ora, p 1 < 1 . Logo, (pr). (ps) < pr+s- 1.(p 1) = (pr+s). p

EXEMPLO 2 Como 19 um nmero primo, ento (19) = 19 1 = 18. Enquanto (125) = (53) = 52.(5 1) = 25 x 4 = 100. Do mesmo modo, (81) = (34) = 33.(3 1) = 27 x 2 = 54.

EXERCCIO 2 Calcule a soma 1 + (11) + (112) + (113) + ... + (11n). Qual a resposta se substituirmos 11 por um nmero primo p qualquer?

Continuando com o mesmo tipo de curiosidade de antes, poderamos indagar: possvel encontrar uma frmula para (n) em funo dos fatores primos da decomposio de n? A resposta sim, o que explicaremos a seguir. Mas, antes dessa explicao, h um caminho a percorrer que comea com o seguinte resultado:
LEMA 1 Dado um nmero natural n > 1, todo sistema completo de restos mdulo n est em correspondncia biunvoca com o conjunto S0 = {0, 1, 2, ...., n 1}. Nota: Por isso S0 = {0, 1, 2, ...., n 1} dito um sistema fundamental completo de restos mdulo n. Demonstrao

99

Na Aula 07 foi definido que um conjunto S = {x1, x2, ..., xn} de n nmeros naturais dito um sistema completo de restos mdulo n, se dois quaisquer de seus elementos no so congruentes mdulo n, isto , xi no congruente a xj mdulo n, se i j. Pelo Algoritmo da Diviso, para cada j, com 1 j n, existem inteiros qj e rj tais que: xj = qj.n + rj, onde 0 rj n. Como xj xi = (qj qi).n + (rj ri), se fosse rj = ri, teramos xj xi = (qj qi).n. Isto , xj xi (mod n), o que contrrio hiptese. Logo, a funo que a cada xj associa o resto rj da diviso de xj por n, define uma correspondncia biunvoca entre os conjuntos S e S0, pois S possui n elementos e a cada elemento de S associado a um nico elemento de S0 que possui tambm n elementos. O prximo passo estabelecer o seguinte:
TEOREMA 1 Se m e n so nmeros inteiros positivos primos entre si, ento (m.n) = (m).(n). Demonstrao

Seja 0 z < mn um inteiro. A diviso de z por n pode ser expressa como z = nq + r, onde 0 r < n. Sendo 0 z < mn , segue que 0 q m 1. De fato, se fosse q > m 1, teramos q m e teramos z = nq + r mn + r mn. Ou seja, z mn, o que uma contradio. O fato crucial para nossa prova que, para cada 0 r < n, o conjunto com m elementos Sr = {r, n + r, 2n + r, 3n + r, ... , (m 1).n + r} um sistema completo de restos mdulo m quando m e n so primos entre si, que o nosso caso. Para tanto, sejam kn + r e jn + r dois elementos de Sr. Se tivssemos kn + r jn + r (mod m) a lei do cancelamento implicaria que kn jn (mod m). Como m e n so primos entre si, ento n pode ser cancelado e obteramos k j (mod m). Mas, k e j esto entre 0 e m 1, logo k = j, o que nos d kn + r = jn + r. A concluso que dois elementos distintos quaisquer de Sr no so congruentes mdulo m. Como Sr possui m elementos, segue que o mesmo um sistema completo de restos mdulo m. Na Aula 05 vimos que, se b = q. a + r, com 0 r < a, ento MDC(a, b) = MDC(a, r). Desse modo, se 0 z < mn um nmero relativamente primo com mn e como m e n so primos entre si, ento z tambm relativamente primo com m e com n. Como z = qm + r, onde 0 r < m e z = kn +s, onde 0 s < n, ento z relativamente primo com r e s, de acordo com o que vimos na Aula 05. Observando isso, sejam:

100

A = {nmeros naturais relativamente primos com mn e menores do que mn} B = {nmeros naturais relativamente primos com m e menores do que m} C = {nmeros naturais relativamente primos com n e menores do que n}. Veja que A possui (mn) elementos, B possui (m) elementos e C tem (n) elementos. Dado z A, ento z = qm + r, onde 0 r < m. Como z Sr, existe 0 s < n tal que z = kn + s. Como MDC(z, m) = 1, segue que MDC(r, m) = 1. Logo, r B. Analogamente, conclumos que s C. Agora, definimos a funo f : A B x C, pondo f(z) = (r, s).
Mostraremos que f uma bijeo. Como o resto e o quociente obtidos pelo Algoritmo da Diviso so nicos, segue que f uma funo injetiva. Por outro lado, dado (r, s) B x C, como Sr um sistema completo de

restos mdulo m, escolha z Sr tal que z = kn + s e como z um elemento de Sr, segue que z = qm + r. Como MDC(r, m) = 1 e MDC(s, n) = 1, ento MDC(z, Mn) = 1, pois m e n so primos entre si, isso nos diz que z A, f(z) = (r, s) e logo f sobrejetiva. Como A possui (mn) elementos, B possui (m) elementos e C tem (n) elementos, ento B x C possui (m). (n) elementos. Portanto, (m.n) = (m).(n).

COROLRIO 1 Se m1, m2, m3, ..., mn so nmeros inteiros positivos, dois a dois primos entre si, ento

(m1 m2 m3... mn) = (m1).(m2).(m3).....(mn)


Demonstrao

A demonstrao ser feita por induo sobre o nmero n de fatores. Para n = 1, o primeiro e o segundo membro da expresso so iguais a (m1). Suponha que a expresso vlida para n = k. Isto , (m1 m2 m3... mk) = (m1).(m2).(m3).....(mk). Mostraremos que ela vlida para n = k + 1. Tomemos m1, m2, m3, ..., mk,, mk+1 so nmeros inteiros positivos, dois a dois primos entre si. Isto MDC(mi, mj) = 1, se i j. Desse modo, chamando a = m1 m2 m3... mk, temos MDC(a, mk+1) = MDC(m1 m2 m3... mk mk+1) = 1, pois cada fator primo de mj, para cada 1 j k, diferente de cada fator primo de mk+1. Como a e mk+1 so primos entre si, pelo Teorema 1, ( m1 m2 m3... mk mk+1) = (amk+1) = (a).(mk+1) = (m1 m2 m3... mk).(mk+1) = = (m1).(m2).(m3).....(mk) ).(mk+1). Isso garante que a frmula proposta vlida para n = k + 1. Portanto, ela vlida para todo nmero natural n. Uma propriedade interessante de dada no seguinte:
COROLRIO 2

101

Para todo nmero inteiro n > 2, (n) par. Demonstrao Se n contm um fator primo p 3 na sua decomposio, seja pr a maior potncia de p nesta decomposio. Ento podemos escrever n = pr.b, onde p e b so primos entre si. Pelo Teorema 1, segue-se que (n) = (pr). (b) = pr-1.(p 1). (b). Como p 3 um nmero primo, em particular p 1 um nmero par. Logo, (n) par. Por outro lado, se n > 2 no contm fator primo impar na sua decomposio em fatores primos, ento n = 2k, para k > 1, donde (n) = (2k) = 2k-1.(2 -1) = 2k-1. Como k > 1, segue que (n) par.

OBSERVAO 1 Para m e n primos entre si, o fato de (m.n) = (m).(n), conforme assegura o Teorema 1, no significa que os nmeros relativamente primos com mn sejam obtidos como produto dos nmeros relativamente primos com m com os nmeros relativamente primos com n. O que existe uma correspondncia biunvoca entre esses conjuntos, conforme ficou evidenciado na prova do referido teorema. A ttulo de ilustrar essa observao vejamos o

EXEMPLO 3 Sejam m = 6 e n = 5, nmeros primos entre si. Logo, (30) = (6 x 5) = (6).(5) = 2 x 4 = 8. Note que os o conjunto dos nmeros inteiros positivos menores do que 6 e do que 5, relativamente primos com 6 e 5, respectivamente, so {1, 5} e {1, 2, 3, 4}. Enquanto que o conjunto dos nmeros inteiros positivos menores do que 30 e relativamente primos com 30 {1, 7, 11, 13, 17, 19, 23, 29}. Observe que nenhum nmero deste ltimo conjunto, exceto o 1, produto de dois nmeros dos outros dois conjuntos anteriores.

EXERCCIO 3 Se p e p + 2 so dois primos, chamados de primos gmeos, mostre que (p + 2) = (p) + 2.

O teorema anterior, juntamente com seu corolrio, simplifica bastante o clculo de (n). Por exemplo, para calcular (420) poderamos fazer uso do Teorema calculando (420) = (7 x 60) = (7) x (60). Mas, o uso do corolrio anterior facilita ainda mais, pois 420 = 22 x 3 x 5 x 7 e como 22 = 4, 3, 5 e 7 so dois a dois primos entre si, podemos escrever (420) = (22) (3) (5) (7) = 2 x 2 x 4 x 6 = 96. Uma aplicao direta do corolrio anterior demonstra o seguinte:
TEOREMA 2 r r r Seja n = p1 1 . p 2 2 ... p k k a decomposio de n em fatores primos. Ento

102

(n) = p1 r 1 . p 2 r 1 ... p k r 1 ( p1 1).( p 2 1)....( p k 1)


1 2 k

Demonstrao

Como p1 , p 2 , ..., p k so nmeros primos distintos, ento eles so dois a dois primos entre si. Logo, tambm quaisquer de suas potncias so duas a duas primas entre si. Em r r particular, pi i e p j j , para i j, so primos entre si. Usando o corolrio do Teorema 1, podemos escrever (n) = ( p1 1 . p 2 2 ... p k k ) = ( p1 1 ).( p2 2 )( pk k ) Mas, a Proposio 2 diz que se p primo, ento ( p r ) = p r 1 .(( p 1) . Logo, para cada j = 1, 2, ... , k, temos ( p j j ) = p j
1 2

r j 1

( p j 1) . Desse modo,
k

(n) = p1 r 1 ( p1 1). p 2 r 1 ( p 2 1).... p k r 1 ( p k 1)


= p1
r 11

. p 2 2 ... p k

r 1

rk 1

( p1 1).( p 2 1)....( p k 1)

EXERCCIO 4 (a) Calcule (360) (b) Encontre todos os inteiros positivos n para os quais (n) mpar.

O Teorema 3, a seguir, garante que a recproca do Teorema 1 tambm verdadeira.


TEOREMA 3 Se m e n so nmeros naturais que no so primos entre si, ento (m.n) (m).(n). Na realidade, neste caso, (m).(n) < (m.n). Demonstrao

Como MDC(m, n) > 1, ento m e n possuem fatores primos em comum. Sejam p1 , p 2 , ..., pt esses fatores primos comuns. Escrevemos

m = p1 1 . p 2 2 ... pt t .a e n = p1 1 . p 2 2 ... pt t .b , onde MDC(a, b) = 1 e nem a nem b possui qualquer fator primo p j , com j = 1, 2, 3, ..., t
[Por exemplo, m = 22.32.5.91 e n = 22.3.52.121, tem-se t = 3, p1 = 2, p2 = 3, p3 = 5, r1 = 3, r2 = 2, r3 = 1, a = 91, s1 =2, s2 = 1, s3 = 2, b = 121] Como (m.n) = ( p1 1 1 . p 2 2
1 1 2 2 t t

r +s

r + s2

.... pt
1 1

rt + st

). (a). (b) , pelo Teorema 2, temos que


2 2 t t

( p1 r + s . p 2 r + s .... pt r + s ) = ( p1 r + s ). ( p 2 r + s ).... ( pt r + s ) . Como tambm temos que ( p1 r . p 2 r ... pt r ) = ( p1 r ). ( p 2 r ).... ( pt r ) e


1 2 t 1 2 t

103

( p1 s . p 2 s ... pt s ) = ( p1 s ). ( p 2 s ).... ( pt s )
1 2 t 1 2 t

Pelo corolrio da Proposio 2, para cada j = 1, 2, 3, ..., t, temos ( p j r j ). ( p j s j ) < ( p j r j + s j ) Ao realizarmos a multiplicao de (m) por (n), aparecem os pares ( p j j ). ( p j j )
r s

e ao calcular (m.n), aparecem os fatores ( p j

rj +s j

) . Portanto, como ( a ) e

(b )

aparecem em ambas as expresses de (m.n) e (m).(n), segue que (m).(n) < (m.n).

COLORRIO Se m e n so inteiros positivos, ento (m.n) = (m).(n) se, somente se, m e n so primos entre si. Demonstrao

Suponha que existam m e n inteiros positivos com (m.n) = (m).(n). Ento a igualdade no pode acontecer se m e n no so primos entre si, pois, pelo Teorema 3, teramos (m).(n) < (m.n), contrariando a hiptese da igualdade. Por outro lado, se m e n so primos entre si, pelo Teorema 1, (m.n) = (m).(n). Os resultados anteriores estabelecem propriedades multiplicativas da funo sob certas condies. A pergunta que se impe se admite propriedades aditivas. Pela fato de um nmero inteiro puder se escrever como soma de dois outros inteiros de vrias maneiras diferentes, contrariamente ao fato de ele ser decomposto em fatores primos de modo nico no se espera que tenha propriedades aditivas. Vejamos o exemplo que se segue para nos convencer dessa suspeita.
EXEMPLO 4 Sabemos que (9) = 6. Enquanto que a decomposio 9 = 7 + 2, fornece (7) + (2) = 7 > (9) = 6, a decomposio 9 = 6 + 3 fornece (6) + (3) = 4 < (9) e a decomposio 9 = 5 + 4, fornece (5) + (4) = 6 = (9).

Observao 2 Podemos afirmar, em carter geral, que se n um nmero primo, ento para qualquer decomposio aditiva p + q de n tem-se (p) + (q) < (p + q) = (n). De fato, como (n) = n 1, (p) p 1, (q) q 1, ento (p) + (q) (p 1) + (q 1) < p +q 1 = n 1 = (n). Isto , (p) + (q) < (n) = (p + q).

EXERCCIO 5 Caracterize todos inteiros positivos n para os quais: (a) (n) = 2k (b) (n) = n/2

104

2. A CONJECTURA DE GOLDBACH

Existe uma conjectura devida a Goldbach (*) de que qualquer inteiro par a soma de dois nmeros primos. Por exemplo: 10 = 7 + 3, 24 = 17 + 7, 36 = 31 + 5, 102 = 97 + 5 etc.
(*)A conjectura de Goldbach, proposta pelo matemtico prussiano Christian Goldbach, um dos problemas no resolvidos da Matemtica, mais precisamente da Teoria dos Nmeros, mais antigos atualmente. Ela diz que todo nmero par maior ou igual a 4 a soma de dois primos. Por exemplo: 4 = 2 + 2; 6 = 3 + 3; 8 = 5 + 3; 10 = 3 + 7 = 5 + 5; 12 = 5 + 7; etc. Verificaes por computador j confirmaram a conjectura de Goldbach para vrios nmeros. No entanto, a efetiva demonstrao matemtica ainda no ocorreu. O melhor resultado at agora foi dado por Olivier Ramar em 1995: todo nmero par a soma de at 6 nmeros primos. Em 7 de junho de 1742, o matemtico prussiano Christian Goldbach escreveu uma carta a Leonhard Euler, onde ele props seguinte conjectura: Todo inteiro par maior que 2 pode ser escrito como a soma de 3 nmeros primos Ele considerava o nmero 1 como sendo primo, que uma conveno posterior (e presente at hoje) abandonou. Uma viso moderna da Conjectura (e a mais aceita) : Todo inteiro par maior que 5 pode ser escrito como a soma de 3 nmeros primos. Euler, se interessado pelo problema, respondeu que a conjectura era equivalente outra: Todo inteiro par maior que 2 pode ser escrito como a soma de 2 nmeros primos. Euler adicionou, ainda, que estava absolutamente certo sobre isso, porm no era capaz de prov-lo. A verso de Euler a mais conhecida e divulgada atualmente, tambm a mais aceita, por ser mais simples e abrangente. Para valores pequenos de n, a conjectura de Goldbach pode ser testada diretamente (mtodo conhecido jocosamente pelos matemticos como fora bruta e ignorncia). Em 1938, N. Pipping testou todos os nmeros at 105. (FONTE: http://pt.wikipedia.org/wiki/Conjectura_de_Goldbach)

Supondo verdadeira esta conjectura, podemos provar que dado um nmero n par, existem nmeros primos r e s tais que (r) + (s) = n. De fato, se n par ento n + 2 par. Sejam, portanto, r e s nmeros primos tais que r + s = n + 2. Mas, (r) = r 1 e (s) = s 1, logo (r) + (s) = (r + s) 2 = (n + 2) 2 = n. Isto , (r) + (s) = n. Este resultado diz que se um nmero par n pode ser escrito como (r) + (s) para r e s primos, ento n + 2 no pode ser escrito como soma de dois nmeros primos. E, nesse caso, no valeria a conjectura de Goldbach.
3. O TEOREMA DE EULER

O Pequeno Teorema de Fermat, estudado na Aula 07, afirma que se p um nmero primo que no divide um inteiro a, ento a p 1 1 (mod p) . Euler (*) observou que p 1 exatamente igual a (p), pois p primo. Alm disso, conseguiu uma generalizao do Teorema de Fermat. Antes de precisar essa generalizao, vamos precisar do seguinte
LEMA 2 Sejam x, m e k inteiros positivos, com k e m primos entre si. Seja r um inteiro tal que x r (mod m) e x r (mod k), ento x r (mod mk). Demonstrao

Pela hiptese, existem inteiros p e q tais que x = qm + r e x = pk + r. Logo, qm + r = pk + r, donde qm = pk. Como m e k so primos entre si, essa igualdade s pode

105

acontecer se p e q so do tipo, q = qm e p = pk. Desse modo, x = qmk + r. Ou seja, x r (mod mk).


TEOREMA 4 [Teorema de Euler (*) ] Sejam n e a inteiros positivos primos entre si. Ento a(n) 1 (mod n). Demonstrao

Tomemos inicialmente a = pr, com r inteiro positivo e p primo e no divisor de a. Pelo Pequeno Teorema de Fermat, temos ap-1 1 (mod p). De acordo com a propriedade bsica VIII, das congruncia, Aula 07, segue que

(a )

r 1 p 1 p

1(mod p ) . Ou seja, a ( p 1) p
p r 1

r 1

1(mod p) .

Desse modo, existe um inteiro m tal que (a p 1 )

1 = mp . Ou ainda,

ap

p r 1

1 = mp .

Multiplicando ambos os membros da ltima igualdade por p r 1 , vem que

ap ap
r 1

r 1

= mp r , que implica em a p a p (mod p r ) .


r

r 1

Como p r e a p so primos entre si, pois p e a o so, ento a p que o mesmo que escrever a(n) 1 (mod n).

p r 1

1(mod p r ) ,

Para o caso geral, suponha que existe um inteiro positivo n tal que a (n ) no seja congruente a 1 mdulo n. Seja n0 o menor inteiro com esta propriedade. Pelo caso anterior, n0 no pode ser escrito como uma potncia de um nmero primo. Logo, podemos escrever n0 = mk, com m, k inteiros maiores do que 1 e primos entre si. Como 1 < m < n0, pela escolha de n0, tem-se a(m) 1 (mod m) e a(k) 1 (mod n). Pela propriedade bsica VIII, das congruncia, Aula 07, podemos escrever a(m).(k) = [a(m) ](k) 1 (mod m) e a(m).(k) = [a(k) ](m) 1 (mod k) Mas, como (m).(k) = (mk) = ( n0). Logo, as igualdades anteriores podem ser escritas a( n0) 1 (mod m) e a( n0) 1 (mod n).

Como m e k so primos entre si, pelo Lema 2, segue que a( n0) 1 (mod mk) 1 (mod n0), em contradio com a hiptese inicial.

106

(*) Leonhard Euler nasceu em 15 de Abril de 1707, em Basil, na Sua. Foi sem dvida o maior matemtico do sculo dezoito. Com 886 trabalhos publicados, a maioria deles no final de sua vida, quando j estava completamente cego, Euler foi to importante no apenas para a matemtica, mas tambm a fsica, engenharia e astronomia, que termos como: Nmero de Euler, Nmeros Eulerianos, Frmula de Euler, significam coisas diferentes de acordo com o contexto. Seu pai era um padre calvinista que nutria esperanas de que seu filho o precedesse no clericato. Ele ensinou a Euler a matemtica. Quando seu filho entrou na Universidade de Basel, estudou Teologia e a lngua Hebraica, e atendia a uma aula de uma hora por semana com Johannes Bernoulli. Ele fez amizade com Daniel e Nicolaus Bernoulli, e recebeu seu primeiro mestrado aos dezessete anos. Os Bernoullis, ento, tiveram de persuadir seu pai a deix-lo continuar com a carreira acadmica. Aos dezenove anos, Euler recebeu meno honrosa por uma soluo que enunciou a um problema posto pela academia de Paris. Mais tarde, ele ganhou o primeiro prmio nesta mesma competio doze vezes. Os Bernoullis conseguiram para Euler uma posio de pesquisa na Academia de So Petersburgo, mas em Medicina, sob o reinado de Catarina I. Porm, ela morreu logo aps, e uma regime de condies caticas se seguiu, com Euler passando seo de matemtica da Academia. Euler quis por muito tempo retornar Europa, mas os constantes nascimentos de seus filhos o impediram. Porm, este foi um perodo extremamente produtivo para ele - era perigoso falar ou at mesmo sair s ruas, portanto Euler concentrou seus esforos na pesquisa e desenvolveu hbitos que manteve pelo resto de sua vida. Euler tambm escreveu livros didticos para escolas russas, supervisionou o departamento de geografia do governo e ajudou a revisar o sistema de pesos e medidas. Ele permaneceu na Rssia at 1740, quando aceitou o convite de Frederico O Grande para entrar na academia de Berlin, onde passou os prximos 24 anos. Euler, porm, no era to sofisticado quanto os outros membros da corte de Frederico e estes anos no foram totalmente agradveis para ele. Porm, ele viveu relativamente bem e manteve uma casa em Berlin assim como uma fazenda. A situao na Rssia melhorou muito durante este perodo, e em 1766 Catarina A Grande o trouxe de volta a So Petersburgo. Ela deu a ele (e a seus 18 dependentes) uma casa mobiliada, e at mesmo um cozinheiro prprio. Em 1735, Euler perdeu a viso de um de seus olhos, e, logo aps seu retorno Rssia, a viso em seu outro olho comeou a deteriorar. Euler sempre teve uma memria excepcional, e era capaz de fazer enormes clculos de cabea, logo ele se preparou para sua futura cegueira aprendendo a escrever frmulas em uma tbua e ditar matemtica a seus filhos ou secretria. Ele foi cego pelos ltimos 17 anos de sua vida, e durante este tempo sua produtividade somente aumentou. Euler foi um cristo por toda a sua vida e frequentemente lia a Bblia a sua famlia. Uma histria sobre sua religio durante sua estada na Rssia envolve o dito filsofo ateu Diderot. Diderot foi convidado corte por Catarina, mas tornou-se inconveniente ao tentar converter todos ao atesmo. Catarina pediu a Euler que ajudasse, e Euler disse a Diderot, que era ignorante em matemtica, que lhe daria uma prova matemtica da existncia de Deus, se ele quisesse ouvir. Diderot disse que sim, e, conforme conta De Morgan, Euler se aproximou de Diderot e disse, srio, em um tom de perfeita convico: "( a + bn ) / n = x, portanto, Deus existe". Diderot ficou sem resposta, e a corte caiu na gargalhada. Diderot voltou imediatamente Frana. Euler teve contribuies a vrias reas da cincia, incluindo dinmica dos fluidos, teoria das rbitas lunares (mars), mecnica, "A teoria matemtica do investimento" (seguros, anuidades, penses), bem como essencialmente todas as reas da matemtica que existiam naquela poca. Ele permaneceu so e alerta at o fim da sua vida, quando morreu de um derrame aos 76 anos. O trabalho ativo de Euler provocou uma tremenda demanda da academia de So Petersburgo, que continuou publicando seus trabalhos por mais de 30 anos aps sua morte. A memria de Euler era lendria, assim como seus poderes de concentrao. Chamado de "Anlise Encarnada", ele era capaz de recitar toda a Eneida de cor, e nunca foi atrapalhado por interrupes ou distraes, de modo que muito de seu trabalho foi realizado tendo suas crianas sua volta. Ele era capaz de realizar clculos prodigiosos de cabea, uma necessidade depois que ele ficou cego. Seu matemtico contemporneo, Condorcet, conta uma histria onde dois dos estudantes de Euler estavam calculando independentemente uma complicada srie infinita, e chegaram a uma discusso depois de somarem dezessete termos, por uma diferena na qinquagsima casa decimal. Euler resolveu a disputa fazendo a soma de cabea. As funes e frmulas de Euler so muito comuns na matemtica. Duas das mais

107

famosas so: e^(ix) = cos(x) + i sin(x), e V - A + F = 2 para qualquer poliedro simples com Vrtices, A arestas e F faces. (FONTE: http://www.exatas.com/matematica/euler.html)

A seguir mostraremos exemplo do uso do Teorema de Euler.


EXEMPLO 5 Encontre o resto da diviso do nmero 393602 por 14. Soluo

O nmero ao qual 393602 congruente o resto solicitado. Ora, 39 e 14 so primos entre si e (14) = (2.7) = (2).(7) = 1.6 = 6 e 3602 = 600 . 6 + 2. Pelo Teorema de Euler, temos (39)(14) = 396 392 392 (mod 14)., donde 393.600 1 ( mod 14), o que nos d 393602 = 393.600 . 392 392 (mod 14). Como 39 11 (mod 14), segue que 392 112 (mod 14) = 121 (mod 14) 9 ( mod 14). Portanto, 9 o resto solicitado.

EXERCCIO 6 Prove que o nmero 22225555 + 55552222 divisvel por 7.

Nessa altura cabe a seguinte pergunta: Sendo (n) um nmero par, para todo inteiro positivo n > 2, ser que para todo nmero par k existe um inteiro positivo x tal que (x) = k? A resposta no. Por exemplo, para k = 14 a equao (x) = 14 no admite soluo. Seno vejamos, na decomposio de x em seus fatores primos deve comparecer 7r, para r 2, pois (x) = 2.7. Desse modo, (x) (7 1).7r-1 = 6. 7r-1 6 . 7 = 42. Portanto, a equao (x) = 14 no admite soluo. O exemplo a seguir ilustra uma aplicao interessante do Teorema de Euler.
EXEMPLO 6 Se a e n so primos entre si, ento x = ba(n)-1 a nica soluo mdulo n da congruncia linear ax b (mod n). Soluo

De fato, ax = a(ba(n)-1) = ba(n). Mas, a(n) 1 (mod n), pelo Teorema de Euler. Logo, ba(n) b (mod n), donde o afirmado. Agora suponha x e x duas solues mdulo n. Isto , ax b (mod n) e ax b (mod n).

Logo, ax ax (mod n), donde x x (mod n), pois MDC(a, n) = 1.

EXEMPLO 7

108

Encontre todas as solues mdulo 35 da congruncia linear 6x 13 (mod 35). Soluo Como 6 e 35 so primos entre si, pelo exemplo anterior, temos x = 13.6(35)-1 = 13.623 = 13.620.63 = 13.(62)10.63 13.1.6 (mod 35) = = 78 (mod 35) = 8 (mod 35). Logo, x 8 (mod 35) a nica soluo da equao inicial.

4. EXERCCIOS
1) Mostre que no uma funo crescente. No entanto, a restrio de ao conjunto dos nmeros primos crescente. 2) Encontre o algarismo das unidades do nmero 310.007. 3) Prove que se m divide n, ento (m) divide (n). r r r [Sugesto: Se p1 1 . p 2 2 ... pt t a decomposio de m em seus fatores primos, conclua que n = p1 1 . p 2 2 ... pt t .a , onde sk rk e MDC(a, pi) = 1, para i = 1, 2, 3,..., k. ] 4) Encontre os valores x para os quais (x) = 16. O mesmo para (x) = 18. 5) Seja p um nmero primo. Mostre que a equao (x) = 2p, tem soluo se, e somente, 2p + 1 um nmero primo. [Sugesto: Para a recproca, suponha que a equao tenha um nmero composto x como soluo e conclua que x = pr, com r 2] 6) Calcule (n) para os seguintes valores de n: a. n = 694 575 b. n = 1 308 736 7) Faa uma tabela dos valores de (n) para n 36. 8) Para todo nmero par n 12, encontre dois valores de x tais que (x) = n. Mostramos anteriormente que a equao (x) = 14 no tem soluo. Portanto, n = 14 o menor valor de n par tal que a equao (x) = n no admite soluo.
s s s

6. RESUMO
Nesta aula introduzimos a funo de Euler como sendo a funo que conta os nmeros positivos, relativamente primos com um inteiro n, e menores do que n. Isso nos permitiu estabelecer um teorema, devido a Euler, o qual generaliza o Pequeno Teorema de Fermat, estudado na Aula 07.

109

7. REFERNCIAS
[1] Burton, David M. Elementary Number Theory. The McGrawHill Companies, Inc. New York. USA. 1998

[2]

Coutinho, S. C. Nmeros Inteiros e Criptografia RSA. Instituto de Matemtica Pura e Aplicada IMPA & Sociedade Brasileira de Matemtica SBM. Rio de Janeiro. 1997

[3]

Hefez, Abramo Elementos de Aritmtica. Sociedade Brasileira de Matemtica. Rio de Janeiro. 2005

[4]

Oliveira, Jos Plnio, de Introduo Teoria dos Nmeros. Publicao IMPA. Rio de Janeiro. 2000

110

AULA 10 Sequncias de Fibonacci

Apresentao

Nesta aula, estudaremos as Seqncias de Fibonacci, assim denominadas em homenagem ao italiano Leonardo de Pisa, mais conhecido como Fibonacci, que significa filho de Bonacci. Ele nasceu em 1180 na cidade de Pisa, na poca do incio da construo da famosa Torre de Pisa, e introduziu na Europa o sistema de numerao hindu-arbico atravs do seu famoso livro Lber Abaci (1202). Fibonacci considerado o maior matemtico da Idade Mdia. Seu livro Lber Abaci contm um problema famoso sobre coelhos, cuja soluo agora conhecida como a Seqncia de Fibonacci. Surpreendentemente, os nmeros de Fibonacci, isto , os nmeros que comparecem na Seqncia de Fibonacci, servem para representar modelos da natureza, como o nmero de espirais em determinadas rosas, frutas, como os girassis, a pinha, o abacaxi etc. Tente entender tudo que est sendo explicado na aula. Estude com caneta e papel ao lado. Leia com ateno. Se for preciso, leia vrias vezes uma linha ou um pargrafo. Seja paciente e procure ter certeza que voc entendeu o que (e por que) est fazendo.

Objetivos

Com esta aula espera-se que voc possa: 1) Identificar uma sequncia de Fibonacci; 2) Perceber relaes entre nmeros de Fibonacci; 3) Usar seqncias de Fibonacci para resolver problemas prticos;

111

1. A SEQNCIA DE FIBONACCI

Figura 1 - Leonardo de Pisa (1180-1250) OS NMEROS DE FIBONACCI Fibonacci - que significa filho de Bonacci- era o pseudnimo de Leonardo de Pisa, que considerado o maior matemtico da Idade Mdia. Como mercador, viajou pelo Oriente. No seu regresso, escreveu os livros Liber Abaci (1202) e Practica Geometricae (1220). No primeiro livro, descreveu fatos de aritmtica e lgebra recolhidos durante sua viagem. No segundo, descreveu o que tinha descoberto na geometria e na trigonometria, [3] pgina 138. O Liber Abaci foi um instrumento que permitiu difundir na Europa ocidental o sistema de numerao hindu-rabe, que era usado ocasionalmente j alguns sculos antes de Leonardo de Pisa, e que foi trazido pelos mercadores, embaixadores, eruditos, peregrinos e soldados vindo da Espanha e do Oriente, [3] pgina 139. Fibonacci ficou conhecido entre ns no exatamente por seus livros, mas porque no sculo XIX o matemtico francs F. Edouard A. Lucas, na sua coleo Rcreations mathmatique (4 volumes, Gauthier-Villars, Paris 1891-1896; reeditado em Paris 1960), ligou o nome de Fibonacci seqncia que aparece num problema do livro Liber Abaci. O problema, relacionado com o nmero de casais de coelhos obtidos a partir de um nico casal, era: Quantos casais de coelhos podem ser produzidos a partir de um nico casal durante um ano se: (a) um casal de coelhos colocado num cercado; (b) Os coelhos precisam de dois meses at chegar idade adulta e poder reproduzirse;

112

(c) cada casal origina um novo casal em cada ms, o qual se torna frtil a partir do segundo ms; (d) nenhum coelho mais pode vir de fora, nenhum coelho pode sair do cercado e no ocorrem mortes. Nessas condies, um casal nasce no primeiro ms, totalizando-se assim 2 casais. Durante o segundo ms, o casal original produz um novo casal. Um ms depois, o casal original e o que nasceu imediatamente aps o seu acasalamento, produzem novos casais. Nessa altura, j existem 3 casais adultos e dois casais filhotes. E, assim por diante. Veja o quadro a seguir: Quadro 1 O crescimento dos casais de Coelhos Ms 1 2 3 4 5 6 7 8 Casais Adultos 1 2 3 5 8 13 21 34 Casais Jovens 1 1 2 3 5 8 13 21 TOTAL 2 3 5 8 13 21 34 55

A seqncia de Fibonacci constituda pelos totais de casais, isto , os nmeros 1, 1, 2, 3, 5, 8, 13, 21, 34, 55, 89, 144, 233,..... Algumas perguntas ocorrem naturalmente: - possvel encontrar uma relao simples entre os termos da seqncia de Fibonacci? - Que relao existe entre os termos consecutivos da seqncia de Fibonacci? - Quantos casais de coelhos haver no final de doze meses? - possvel encontrar uma frmula simples para a soma dos n primeiros termos da seqncia de Fibonacci? - Existe uma frmula para descrever os termos da seqncia de Fibonacci? Essas perguntas sero abordadas nas prximas sees.
2. PROPRIEDADES ELEMENTARES 2.1 A Igualdade Fundamental

Vamos denotar por (fn) a seqncia de Fibonacci e seus termos por: f1 = 1, f2 = 1, f3 = 2, f4 = 3, f5 = 5, f6 = 8, f7 = 13, ...... Com essa notao, a seqncia de Fibonacci exibe uma propriedade interessante:

113

f3 = f1 + f2;

f4 = f2 + f3; f5 = f3 + f4; f6 = f4 + f5

De uma maneira geral, os termos de uma seqncia de Fibonacci satisfazem a relao: f1 = 1, f2 = 1, fn = fn-1 + fn-2, para n 3. Isto , cada termo da seqncia, a partir do terceiro, a soma dos dois imediatamente inferiores. Desse modo, descreve-se a seqncia de Fibonacci como uma seqncia recursiva, ou seja, uma seqncia na qual todo termo pode ser representado como uma combinao linear dos termos precedentes. Isto , f1 = 1, f2 = 1, fn = fn-1 + fn-2, para n 3. A seqncia de Fibonacci a primeira seqncia recursiva conhecida na literatura matemtica (Por volta de 1634, a partir dos trabalhos de Albert Girard [1], pgina 287).

EXEMPLO 1 Numa faixa 1 x 10, veja a figura 2, a seguir, cada quadrado pintado ou de azul ou de vermelho, mas dois quadrados adjacentes no podem ser pintados de azul.

Figura 2 Faixa de quadrados 1 x 10 De quantas maneiras distintas podemos realizar a pintura? Soluo Certamente existem muitas maneiras diferentes de realizarmos a pintura. Mas, como contar todas essas maneiras? Construamos, mentalmente, uma faixa menos complicada, com um nmero menor de quadrados. Comeamos pensando na faixa tendo um s quadrado. Neste caso, existem duas maneiras de pintar: ou pintamos o quadrado de azul (A) ou pintamos o quadrado de vermelho (V). Se a faixa possui dois quadrados. Neste caso, se o primeiro quadrado for pintado de vermelho (V), temos duas possibilidades para pintar o segundo quadrado: azul (A) ou vermelho (V). Se o primeiro quadrado for pintado de azul (A), s podemos pintar o segundo de vermelho (V). Portanto, com as regras dadas, podemos pintar uma faixa com dois quadrados de trs modos distintos: VA; VV; AV. Se a faixa possui trs quadrados. Neste caso, se o primeiro quadrado for pintado de vermelho (V), temos trs possibilidades para pintar os dois quadrados seguintes VA, VV e AV . Se o primeiro quadrado for pintado de azul (A), temos duas possibilidades

114

para pintar os dois quadrados seguintes: VV e VA. Portanto, com as regras dadas, podemos pintar de cinco modos diferentes uma faixa com trs quadrados: VVA; VVV; VAV; AVV; AVA. E se a faixa possui quatro quadrados? Seguimos o raciocnio anterior. Para isso, escolhemos a pintura do primeiro quadrado e pintamos os quadrados restantes olhando para as pinturas j feitas. Se pintarmos o primeiro quadrado de vermelho (V), com as regras dadas, os trs quadrados restantes podem ser pintados de cinco maneiras distintas, como se fosse o caso de a faixa ter dois quadrados, e teremos as seguintes pinturas possveis: VVVA; VVVV; VVAV; VAVV; VAVA Se pintarmos o primeiro quadrado de azul (A), de acordo com as regras dadas, o prximo quadrado s pode ser pintado de vermelho (V) e os dois quadrados restantes podem ser pintados de dois modos distintos, como se fosse o caso da faixa ter dois quadrados. Assim, a pintura seria: AVVA; AVVV; AVAV. Portanto, no caso de a faixa ter quatro quadrados, podemos pint-la de 5 + 3 = 8 modos distintos. Para o caso da faixa ter cinco quadrados, usamos o mesmo argumento do caso da faixa ter quatro quadrados. Ou seja, se pintarmos o primeiro quadrado de vermelho (V), com as regras dadas, existem 8 possibilidades para efetuar a pintura dos quatro quadrados restantes. E se o primeiro quadrado for pintado de azul (A), com as regras dadas, o segundo s pode ser pintado de vermelho (V) e os trs restantes de 5 modos distintos, dando um total de 8 + 5 = 13 possibilidades de pintura. Portanto, seguindo este raciocnio, o nmero de maneiras distintas de pintarmos uma faixa de comprimento 1 x n dada, de acordo com o nmero de quadrados, n, pela seqncia de nmeros 2, 3, 5, 8, 13, 21, 34, 55, 89, 144, ...., onde cada termo a soma dos dois termos precedentes. Esta a Seqncia de Fibonacci. Portanto, a resposta, para n = 10, 144.
EXERCCIO 1 Seja {f1 = 1, f2 = 1, f3 = 2, f4 = 3, f5 = 5, f6 = 8, f7 = 13, fn, ...... } a seqncia de Fibonacci, satisfazendo de uma maneira geral: fn = fn-1 + fn-2, para n 3. Verifique que os termos f3, f6, f9, .... so todos nmeros pares.

[Sugesto: Usando que fn = fn-1 + fn-2, para n 3, mostre que fn+3 fn (mod 2)]

115

2.2 A Soma dos primeiros n termos da seqncia de Fibonacci

A soma dos n primeiros termos da seqncia de Fibonacci tem uma frmula fcil de gravar, dada na
PROPOSIO 1 Sejam f1 = 1, f2 = 1, fn = fn-1 + fn-2, para n 3. A soma, Sn, dos primeiros n termos da seqncia de Fibonacci dada por:

Sn = f1 + f2 + f3 +...+ fn = fn+2 -1
Demonstrao

Comeamos escrevendo a igualdade fundamental: fk+2 = fk + fk+1, para k 1. Da segue que fk = fk+2 - fk+1. Agora, fazendo k = 1, 2, 3, 4,..., n e somando membro a membro, obtemos Sn = f1 + f2 + f3 + ...+ fn = (f3 - f2) + (f4 - f3) + (f5 - f4) + ...+ (fn+2 - fn+1) = = (f3 - 1) + (f4 - f3) + (f5 - f4) + ...+ + (fn+1 fn) + (fn+2 - fn+1) = fn+2 - f2 = fn+2 -1. Veja que os termos intermedirios se cancelam, resultando somente os termos extremos: fn+2 -1.

EXEMPLO 2 Mostre que a soma de quaisquer 10 termos consecutivos da seqncia de Fibonacci igual a 11 vezes o stimo desses termos. Soluo

Considere os 10 termos da seqncia Fibonacci como sendo: a, b, a + b, a + 2b, 2a + 3b, 3a + 5b, 5a + 8b, 8a + 13b, 13a + 21b, 21a + 34b. O stimo termo da seqncia 5a + 8b e a soma dos 10 termos igual a S = 55a + 88b = 11(5a + 8b), com queramos mostrar.

EXERCCIO 2 Sejam f1 = 1, f2 = 1, fn = fn-1 + fn-2, para n 3.Verifique que a seguinte igualdade: f1 + f2 + f3 + f4 + f5 + f6 + f7 + f8 = f10 1.

2.3 Termos Consecutivos da Seqncia de Fibonacci so Relativamente Primos

Observando-se os nmeros iniciais da seqncia de Fibonacci, podemos notar que termos consecutivos (f1 e f2; f2 e f3; f3 e f4; assim por diante) so relativamente primos (i.e. tm Mximo Divisor Comum igual a 1). Uma pergunta ocorre naturalmente:

116

Isso se verifica para todo n? Isto , fn e fn-1 so relativamente primos? A resposta afirmativa: Teorema 1 Na seqncia, (fn), de Fibonacci, temos que MDC (fn, fn-1) = 1, para todo n 2. Demonstrao O caso em que n for igual a 1 ou 2 trivialmente verdadeiro. Para n 3, faremos um argumento por reduo ao absurdo. Isto , supe-se que MDC(fn, fn-1) seja um inteiro d maior do que 1 e vamos chegar a uma contradio. Assim, vamos supor, pelo Princpio do Menor Inteiro, que existe n o menor inteiro positivo tal que MDC(fn, fn-1) = d > 1. Nesse caso, d divide fn e d divide fn-1. Como fn = fn-1 + fn-2, para n 3, segue que d divide fn-2, contrariando a escolha de n. Uma pergunta ocorre naturalmente: Como f3 = 2, f5 = 5, f7 = 13 e f11 = 89 so todos primos, podemos concluir que fn primo sempre que n seja primo? A resposta no. Para ilustrar, veja o contra-exemplo: 19 primo, mas, no entanto, f19 = 4181 = 37 x 113 um nmero composto. Voc pode encontrar outro contra-exemplo?

EXEMPLO 3 Verifique se os nmeros de Fibonacci f13 e f17 so primos. Soluo

Usando a relao fn = fn-1 + fn-2, para n 3, temos: f1 = 1, f2 = 1, f3 = 2, f4 = 3, f5 = 5, f6 = 8, f7 = 13, f8 = 21, f9 = 34, f10 = 55, f11 = 89, f12 = 144, f13 = 233, f14 = 377, f15 = 610, f16 = 987, f17 = 1597. Agora, basta observar que: f13 = 233 primo, pois no divisvel por qualquer inteiro positivo que seja maior do que 1 e menor do que 233 15,264 . Do mesmo modo, f17 = 1597 primo, pois no tem qualquer divisor entre 1 e 1597 39,96 .
EXERCCIO 3 Calcule: (a) MDC (f15, f20)

(b) MDC (f16, f24) .

No se conseguiu ainda determinar quais so todos os nmeros inteiros positivos n para os quais fn seja primo. Voc pode resolver esse problema e tornar-se um matemtico conhecido! Tambm no se sabe ainda se o nmero de primos na seqncia de Fibonacci infinito. Outro problema intrigante! Voc pode tambm ficar famoso se conseguir resolv-lo!

117

2.4 O Algoritmo da Diviso e a Seqncia de Fibonacci

um fato conhecido, estudado na Aula 05, O Mximo Divisor Comum, O Mnimo Mltiplo Comum e as Equaes Diofantinas Lineares, que o Mximo Divisor Comum de dois inteiros positivos pode ser calculado a partir do Algoritmo da Diviso, depois de um nmero finito de divises. Por exemplo, para calcular o MDC(32,12) comeamos dividindo 32 por 12: 32 = 2 x 12 + 8, onde 8 o resto da diviso e 2 o quociente. Em seguida, fazemos a diviso do quociente pelo resto da diviso anterior: 12 = 1 x 8 + 4. Finalmente, dividimos 8 por 4: 8 = 2 x 4 + 0. Como o resto da ltima diviso zero, dizemos que MDC(32,12) = 4. Nesse caso, foi preciso 3 divises para encontrar o Mximo Divisor Comum. Por uma escolha conveniente dos inteiros, o nmero de divises pode ser arbitrariamente grande. Voc capaz de dar exemplos dessa situao? Nessa altura, uma pergunta aparece naturalmente: Dado um inteiro positivo n, existem inteiros positivos a e b tais que, para se calcular MDC(a, b) usando o Algoritmo da Diviso necessita-se de exatamente n divises? A resposta afirmativa e foi dada, em 1844, por Gabriel Lam (1825-1871), um matemtico francs. Lam, ao responder a pergunta, descobriu uma surpreendente ligao entre os nmeros de Fibonacci e o Algoritmo da Diviso. A resposta foi dada tomando a = fn+2 e b = fn+1. Nesse caso, o uso do Algoritmo da Diviso para a obteno do MDC(fn+2, fn+1) leva-nos ao sistema de equaes: fn+2 = 1 x fn+1 + fn fn+1 = 1 x fn + fn-1 fn = 1 x fn-1 + fn-2 .............................. f4 = 1 x f3 + f2 f3 = 2 x f2 + 0 Do sistema de equaes acima, fica claro que MDC(fn+2, fn+1) = f2 = 1 e que o nmero de divises n. Uma conseqncia surpreendente dada na seguinte
Proposio 2 O nmero de divises necessrias para achar o Maior Divisor Comum de dois inteiros positivos, a e b, usando o Algoritmo da Diviso, no excede a cinco vezes o nmero de algarismos (na base decimal) do menor nmero.

118

Demonstrao Seja a b, com o nmero a possuindo k algarismos na base dez. Nesse caso, a < 10k. Se, usando o Algoritmo da Diviso para calcular o MDC(a, b), necessitamos de n divises, ento a fn+1. Portanto, de acordo com o argumento anterior de Lam, 10k > fn+1. Usando induo, podemos mostrar que fm > (8/5)m-2, para todo m > 2, e, portanto 10k > (8/5)n-1 Elevando cada lado a potncia 5, obtemos 105k > [(8/5)5]n-1 > 10n-1 Assim, 5k > n-1, e, como k inteiro, n 5k, como queramos. Sobre essa questo uma interessante referncia em portugus [2]. Outro fato interessante a respeito da seqncia de Fibonacci : PROPOSIO 3 Os termos da seqncia, (fn), de Fibonacci satisfazem a identidade: fm+n = fm-1 fn + fm fn+1
Demonstrao

Um exemplo fcil de achar. Seno vejamos, para calcular f9 basta aplicar a identidade: f9 = f6+3 = f5 f3 + f6 f4 = 5 x 2 + 8 x 3 = 34. Mas, como sabemos se a identidade acima vlida para todo inteiro m e n? Para responder a essa questo, vamos fazer uma demonstrao por induo sobre n. Para isso, fixemos m. Se n = 1, a identidade torna-se fm+1 = fm-1 f1 + fm f2 = fm-1 .1 + fm .1 = fm-1 + fm , que verdadeira, por ser a relao fundamental da seqncia de Fibonacci. Suponha que a identidade acima seja verdadeira quando n um dos inteiros: 1, 2, 3, 4,..., k. Assim, tem-se fm+k = fm-1 fk + fm fk+1 e tambm fm+ (k-1) = fm-1 fk-1 + fm fk Somando-se as duas igualdades e aplicando-se a igualdade fundamental mostra-se que fm+ (k+1) = fm-1 fk+1 + fm fk+2. O que conclui a prova.

EXEMPLO 4 Mostre a seguinte identidade: fm+3 = 3fm+1 - fm-1, para m 2. Soluo

119

Da Proposio 3, temos fm+3 = fm-1 f3 + fm f4 = 2fm-1 + 3fm , pois f3 = 2 e f4 = 3. Agora, usando a relao fm = fm-1 + fm-2, reescrevemos a ltima expresso obtida como: fm+3 = 2fm-1 + 3fm =2fm-1 + 2fm + fm = 2(fm-1 + fm) + fm = 2fm+1 + fm = 2fm+1 + (fm+1 - fm1) = = 3fm+1 - fm-1, como queramos mostrar.

ATIVIDADE 4 Demonstre a identidade de Cassini: O quadrado de qualquer termo da seqncia de Fibonacci difere do produto dos termos adjacentes por 1 ou -1. Isto , fn-1 fn+1 - fn2 = (-1)n

[Sugesto: Em vez de fazer contas, verifique o resultado surpreendente n 1 1 f n+1 f n = f n1 1 0 f n e use o fato de que para duas matrizes quadradas A e B, de mesma ordem, tem-se det(AB) = det(A).det(B) ] Nota: Jean-Dominique Cassini descobriu essa identidade em 1680, veja J.D. Cassini, Une nouvelle progression de nombres, Histoire de lAcademie Royale des Sciences, Paris, 1 (1733) 496-201.

2.5 O Teorema de Lucas

Em 1876, o matemtico francs F. Edouard A. Lucas provou que o Mximo Divisor Comum de dois nmeros de Fibonacci era outro nmero de Fibonacci. Mais precisamente,

Teorema 2 (Lucas) Se (fn) a seqncia de Fibonacci, ento MDC (fm, fn) = fMDC(m, n).

Antes de demonstrarmos o Teorema de Lucas, vamos provar os seguintes lemas:


Lema 1 Para m, n inteiros maiores do que ou iguais a 1, fmn divisvel por fm. Demonstrao

A prova por induo sobre n. Para n = 1 o resultado verdadeiro, pois fmn = fm. Suponha que para n = 1, 2, 3..., k, fmn seja divisvel por fm. O caso (n+1) verificado usando a frmula fundamental: fm (k+1) = fm(k-1) fm + fmk fm+1 Como, por hiptese de induo, fm divide fmk, o lado direito da expresso acima divisvel por fm, e, portanto, fm (k+1) divisvel por fm.

120

Lema 2 Se m e n so inteiros positivos (m n) com m = qn + r, 0 r < n, ento MDC(fm, fn) = MDC(fn, fr). Demonstrao

Pela igualdade fundamental temos: MDC(fm, fn) = MDC(fqn+r, fn) = MDC(fq(n-1) fr + fqn fr+1, fn). Usando o Lema 1 e o fato de que MDC(a+b, c) = MDC(a, c), sempre que c dividir b temos MDC ( f q ( n 1) f r + f qn f r +1 , f n ) = MDC ( f q ( n 1) f r , f n ) Agora, vamos mostrar que d = MDC(fqn-1, fn) = 1. As relaes: d divide fn e fn divide fqn, implicam que d divide fqn. Portanto, d um inteiro positivo que divide dois termos consecutivos, fqn e fqn-1, da seqncia de Fibonacci. Logo d = 1. Por outro lado, MDC ( f m , f n ) = MDC ( f q ( n )1 f r , f n ) = MDC ( f r , f n ) A ltima igualdade decorre do fato: sempre que MDC(a,b) = 1, temos MDC( a, bc ) = MDC(a,c). O que finaliza a demonstrao. Agora, estamos em condies de demonstrar o Teorema de Lucas.
Demonstrao do Teorema de Lucas

Suponha que m n. Aplicando o Algortmo da Diviso para m e n, obtemos o seguinte sistema de equaes :
m = q1 n + r1 , 0 r1 < n n = q 2 r1 + r2 , 0 r2 < r1 r1 = q3 r2 + r3 , 0 r3 < r2 ....................................................... r n 2 = q n rn 1 + rn , 0 rn < rn 1

rn 1 = q n +1 rn + 0 , De acordo com o Lema 2, temos


MDC(f m , f n ) = MDC(f n , f r1 ) = .... = MDC(f rn , f rn 1 )

Como rn divide rn-1, o Lema 1 garante que f rn divide f rn 1 . Portanto, temos que: MDC(f rn , f rn 1 ) = f rn . Mas, rn sendo o ltimo resto no nulo no Algortmo Euclides para m e n, ele o MDC(m, n). O que encerra a prova.

121

2.5 A Frmula de Binet

A pergunta: Existe uma frmula geral que expresse o n-simo termo da seqncia de Fibonacci? tem resposta afirmativa. A frmula foi descoberta em 1718 pelo matemtico francs De Moivre. Mas, a frmula ficou conhecida pelo nome de Frmula de Binet, em homenagem ao matemtico francs que a resdescobriu mais de um sculo depois, em 1843. Antes de apresentarmos a frmula de Binet, vamos fazer algumas observaes. A seqncia de Fibonacci no a nica seqncia que satisfaz a frmula recursiva
f n + 2 = f n + f n +1

(*)

De fato, a chamada seqncia de Lucas 1, 3, 4, 7, 11, 18, ... tambm satisfaz a relao (*). Na verdade, existe uma infinidade de seqncias satisfazendo a relao (*). O lema seguinte mostra como podemos produzir novas solues de (*), isto , outras seqncia satisfazendo a relao f n + 2 = f n + f n +1 (*).
Lema 3 (a) Se A = (a1, a2, a3,...) uma soluo de (*) e c um nmero real qualquer, ento a seqncia

cA = (ca1, ca2, ca3,...) tambm soluo de (*). (b) Se as seqncias A = (a1, a2, a3,...) e B = (b1, b2, b3,...) satisfazem (*), ento a soma A + B = (a1 + b1, a2 + b2, a3 + b3, ... ) tambm satisfaz a relao (*) Demonstrao (a) Se A satisfaz a relao fundamental (*), temos que an+ 2 = an + an+1 . Agora multiplicando por c ambos os membros da ltima relao, temos can+ 2 = can + can +1 . Portanto a sequncia cA uma soluo de (*). (b) Se as seqncias A e B satisfazem (*), ento an+ 2 = an + an+1 e bn+ 2 = bn + bn +1 . Assim, somando membro a membro as duas igualdades, obtemos an+ 2 + bn+ 2 = (an + bn ) + (an+1 + bn+1 ) . O que mostra que a seqncia A + B satisfaz (*).

122

TEOREMA 3 (A Frmula de Binet) O n-simo termo da seqncia de Fibonacci dado pela frmula
f
n

1 1 5 n 1 5 n + = 5 2 2

Demonstrao

interessante observar que, uma seqncia extremamente fcil de definir recursivamente tenha, para cada termo, uma frmula complicada. Observe que, apesar de no ser fcil visualizar, o lado direito da frmula acima um inteiro! Observe que, se r a raiz positiva da equao x2 = x + 1, ento r2 = r + 1. Logo, 1+ 5 r= . 2 Multiplicando cada lado de r2 = r + 1 por rn, obtemos rn+2 = rn+1 + rn, para todo n = 1, 2, 3, 4,.... Assim, (r, r2, r3,....) uma seqncia satisfazendo (*). Seja u = 1 r = 1 5 a outra 2 soluo da equao x2 = x + 1. Logo, (u, u2, u3,....) uma seqncia satisfazendo (*). Pelo Lema 3, a seqncia (r - u, r2 - u2, r3 - u3,...) satisfaz (*). Mas, os dois primeiros termos (pois r e u so solues de x2 = x + 1) so iguais, pois r2 - u2 = (r - u) (r + u) = 1. (r - u) = r - u, e, portanto, a seqncia Lema 3, satisfaz a n + 2 rn un = a n , com n = 1, 2, 3, ...... Isto , a1 = a 2 = 1 , e pelo r u rn un = a n + a n 1 , tem de ser a seqncia de Fibonacci. E ru
n n n

1+ 5 1 5 , un = exatamente o valor da frmula de Binet, uma vez que r = 2 2 e r u= 5 . Isso conclui a prova.

Observe, a partir da frmula de Binet, que fn aproximadamente igual a


rn 5

com erro absoluto tendendo a zero quando n tende ao infinito. De fato, temos que

123

fn

rn 5

1 r 5

porque 1 - r < 1. Ento os nmeros de Fibonacci crescem exponencialmente e, quando n cresce, eles se aproximam dos termos da progresso geomtrica
r r2 r3 ,... , , 5 5 5

O nmero r, soluo positiva da equao x2 = x + 1, chamado razo urea.

EXEMPLO 5 Encontre, diretamente da Frmula de Binet, o vigsimo nmero de Fibonacci. Soluo


1 1 5 n 1 5 n + . Desse modo, basta substituir na frmula de Binet Temos que n = 5 2 2 2 0 1 1 5 1 5 20 f + . Agora, desenvolvemos n por 20. Assim, 2 0 = 5 2 2 2 0 1 5 1 5 20 20 20 20 20 + 1+ 5 1+ 5 1 5 1 5 . e 2 = 2 = = = 262144 262144 220 220 f

) (

) (

Pelo Binmio de Newton, o desenvolvimento de 1 + 5

(1 + 5 ) (1 5 )
20

20

20 = 2 1

) (1 5 ) 20 20 5 + ( 5 ) + ... + ( 5 ) . 3 19
20
3 19

20

igual a

Assim,
f
2 0

1 1 5 20 1 5 20 20 + 20 3 20 19 1 = 5 2 2 = 262144 5 2 1 5 + 3 5 +...+ 19 5 = .

( )

( )

2 18 20 1 20 20 + 5 + ... + 5 = 1 3 19 131072

( )

( )

20 2 20 9 1 20 20 + .5 + .5 + ... + .5 = 6765 1 3 3 19 131072

EXERCCIO 5 Usando a Frmula de Binet, mostre que f2n+1.f2n-1 f2n. f2n+1 = 1, para todo n 1.

124

2.6 A Frmula de Lucas para fn

Os coeficientes do Binmio de Newton (Isaac Newton-1642-1727) (x + y)n, onde n um nmero natural, so os (n+1) inteiros da forma =

n n! , k = 0, 1, 2, 3,..., n k (n k )! k !

O tringulo de Pascal (Blais Pascal- 1623-1662) formado a partir desses nmeros, fazendo n = 0, 1, 2,... , com k variando de 0 at n, e colocando-se esses nmeros como segue:

Figura 3 - Tringulo de PASCAL

Se olharmos para o tringulo de Pascal, agora formando um tringulo retngulo, veja a figura 4, a seguir, vamos ver que, surpreendentemente, aparecem a os nmeros de Fibonacci ([4] pg.135):

Figura 4 . Tringulo de Pascal, formando um tringulo retngulo

Soma dos elementos das diagonais da figura 4 so os nmeros de Fibonacci: 1, 1, 2, 3, 5, 8, 13, ......... Em 1876, F. Edouard A. Lucas descobriu a seguinte frmula para os termos de Fibonacci, empregando os coeficientes binomiais:

125

Teorema 3 (Lucas)

n n 1 n 2 n j fn+1 = + + +...+ , 0 1 2 j

onde j o maior inteiro menor do que ou igual a n/2.


Demonstrao

Por induo sobre n. fcil ver para os casos n = 0, 1, 2. Suponhamos que a frmula seja verdadeira para os inteiros 0, 1, 2, 3,..., k-1. Da identidade fundamental e da hiptese de induo temos:

k 1 k 2 k 3 k j 1 fk+1 = fk + fk-1 = + + +...+ + j 0 1 2


k 2 k 3 k 4 k j 1 + , +...+ + + j 1 0 1 2
que pode ser reescrito como:

k j 1 k j 1 k 1 k 2 k 2 k 3 k 3 fk+1 = + +...+ + + + + j j 1 0 1 0 2 1
m m 1 m 1 Agora, aplicando a relao = + , obtemos i i i 1

k 1 k 1 k 2 k 3 k j fk+1 = + + + +...+ 0 1 2 3 j

Para concluir a prova, basta observar que a primeira parcela da soma acima igual seguinte expresso: , para k > 1.
k 0

2.7 A Razo urea

Considere a razo

rn =

f n +1 , com n = 1, 2, 3, 4,..., entre os nmeros de Fibonacci fn

consecutivos. A seqncia, rn, dada por:


propriedades fascinantes:

1 2 3 5 8 13 21 34 55 , , , , , , , , ,.... , possui 1 1 2 3 5 8 13 21 34

(i) os termos de ordem par so decrescentes: r2 > r4 > r6 > r8 > r10 >>....

126

(ii) os termos de ordem mpar so crescentes: r1 < r3 < r5 < r7 < r9 <<.... (iii) os termos consecutivos aparecem em ordem alternada: r1 < r2, r2> r3, r3 < r4, r4 > r5,....... (iv) a seqncia dos intervalos fechados: [r1, r2], [r3, r4], [r5, r6], [r7, r8],........, encaixante, isto , cada um dos intervalos, a partir do segundo, est inteiramente contido no anterior: [r1, r2] [r3, r4] [r5, r6] [r7, r8] ........ . Alm disso, o limite do comprimento desses intervalos tende a zero quando n tende ao infinito. De fato, pela ( 1) n , que identidade de Cassini (veja a Atividade 2 desta Aula) temos: rn - rn-1 = f n f n 1 tende para zero quando n tende para infinito. O Princpio dos Intervalos Encaixantes (que voc estudar na disciplina Anlise Real) afirma:
Se I1, I2, I3,,... uma seqncia de intervalos fechados e limitados, e se o comprimento de In tende a zero quando n tende ao infinito, ento existe um, e somente um, nmero real que pertence a todos os intervalos da seqncia.

Em outras palavras, O Princpio dos Intervalos Encaixantes afirma que o sistema de nmeros reais completo, isto , sem furo ou brecha ou lacuna. No caso da seqncia de intervalos fechados definidos acima, conclumos que existe um nmero real L comum a todo intervalo fechado [r2n-1, r2n], para n = 1, 2, 3, 4,...., e, portanto, L = lim f n + 1 . Sabendo-se que fn+2 = fn+1 + fn e dividindo-se ambos os lados
n

fn

por fn+1 temos: rn+1 = 1 + 1/rn. Agora, fazendo o limite quando n tende ao infinito, 1 obtemos L = 1+ . Portanto, L a raiz positiva da equao L2 = L + 1, ou seja, L 1+ 5 , que a razo urea - costumeiramente denotada por - e que j havamos L= 2 encontrado antes quando estudamos a frmula de Binet para os nmeros de Fibonacci. Uma aproximao para a razo urea: = 1, 6180.... A razo urea tem origem na antiguidade clssica. Euclides (matemtico grego, diretor do famoso Museu de Alexandria e autor de Os Elementos) chamou-a de mdia e extrema razo, que significa a razo obtida quando um segmento de reta est dividido em duas partes desiguais de modo que a razo do todo para o mais largo igual razo da maior para a menor:

Figura 5 - Segmento dividido em mdia e extrema razo


AB AC + CB CB 1 = 1+ = 1+ = AC AC AC

127

Na Renascena, a razo urea era chamada a divina proporo. A construo clssica de um polgono regular usando somente as ferramentas proposta por Euclides, rgua sem marcao e o compasso, depende da diviso de um segmento de reta na razo : 1. Vamos explicar esse fato. Inicialmente, observe que, para construir um pentgono regular suficiente construirmos um decgono regular inscrito num circulo, pois o pentgono pode ser formado conectando os vrtices do decgono alternadamente. Seguindo o mtodo dos gregos antigos, suponha que o decgono j est construdo- portanto o ngulo central 2 AOB, veja Figura 6, a seguir, mede = . Seja C o ponto sobre o raio OA tal que 10 5 BC a bissetriz do ngulo OBA . Como o tringulo OBA isscele, os ngulos da base medem 1/2(-/5) = 2/5. E, portanto, o ngulo OBC mede /5. O ngulo ACB, que externo, mede /5 + /5= 2/5. Logo, os tringulos ABC e OBC so tambm issceles. Portanto: OC = BC = AB.

Figura 6 - Construo de um decgono inscrito num crculo

Tomando o raio do crculo medindo 1, o lado do decgono AB = x, e como os tringulos ABO e ABC so semelhantes, por terem os mesmos ngulos, temos:
1 x . Portanto, o ponto C divide o raio em mdia e extrema razo. Segue que: = x 1 x

1 1 2 2 5 1 5 1 . = x = x = = = . Como o segmento de x 2 5 +1 5 +1 5 1 comprimento 5 pode ser construdo, usando somente a rgua e o compasso, tambm podemos construir o lado do decgono.

Exerccios

1) Mostre que a soma fn2 + f2n+1 sempre um nmero de Fibonacci. 2) Mostre que f1 f2 + f2 f3 ++... + f2n-1 f2n = f22n 3) Encontre frmulas simples para as somas (a) f1 + f3 + f5 + ... + f2n-1

128

(b) f2 + f4 + f6 +... + f2n 4) De quantas maneiras possvel subir uma escada com n degraus pisando em um ou dois degraus de cada vez? 5) Prove que: (a) se 2 divide fn+1 ento 4 divide (fn2 - fn-12) (b) se 3 divide fn, ento 9 divide (fn+13 - fn-13) 6) Prove que: (fn fn+3)2 + (2fn+1 fn+2)2 = (f2n+3)2 e use isso para gerar 5 triplas Pitagricas, isto , nmeros que satisfazem ao Teorema de Pitgoras (o quadrado do maior nmero igual soma dos quadrados dos outros dois). 7) Prove que o produto fn fn+1 fn+2 fn+3, de quaisquer quatro nmeros de Fibonacci consecutivos igual rea de um tringulo retngulo de lados inteiros (tringulo pitagrico). 8) Mostre que a soma dos quadrados dos primeiros n nmeros de Fibonacci igual fnfn+1. 9) Mostre que a diferena f2n+1 - f2n-1 um nmero de Fibonacci, para todos n 2. 10) Verificar que: (a) 2 divide fn se, e somente se, 3 divide n. (b) 3 divide fn se, e somente se, 4 divide n. (c) 4 divide fn se, e somente se, 6 divide n. (d) 5 divide fn se, e somente se, 5 divide n.
RESUMO

Nesta aula, introduzimos as seqncias de Fibonacci, que foram usadas no sculo VIII para descrever mtricas na poesia snscrita e que apareceram pela primeira vez na Europa em 1202, atravs do livro Lber Abaci, de Leonardo de Pisa, mais conhecido como Fibonacci. Posteriormente, em 1634, Albert Girard, matemtico alemo e aluno de Vite, definiu a seqncia de Fibonacci recursivamente.

129

REFERNCIAS

BURTON, DAVID M., Elementary number theory 4 th ed. The McGraw-Hill Companies, Inc. New York. 1998 de CARVALHO, JOO. B. P., Euclides, Fibonacci e Lam, RPM- SBM, No. 24, 32-40. Rio de Janeiro. 1993 STRUICK, DIRK, J., Histria Concisa das Matemticas. Gradiva. Lisboa. 1989 YOUNG, ROBERT M., Excursions Calculus : An Interplay of The Continous and the Discrete. Dolciani Mathematical Exposition, MAA, Washington. 1992 ZECKENDORF, E., Representation des nombres natureals par une somme de nombres de Fibonacci ou de nombres de Lucas. Bull. de la Soc. Royale des Sci. de Liege, 41 (1972) 179-182. Frana.

130

Aula 11 Noes sobre o processo e o mtodo de criptografar

Apresentao

Esta a Aula 09 da disciplina Mtodos e Modelos Matemticos, que estuda aplicaes da Matemtica no cotidiano das pessoas. Aqui voc estudar como usar fatos simples da disciplina de Teoria dos Nmeros para criar seus prprios cdigos secretos de comunicao. Tambm, veremos como os sites de venda de produtos pela INTERNET fazem para tornar as vendas seguras, para quem usa o carto de crdito. Nesta aula, damos uma idia de como se usou a Matemtica no passado e como se usa nos dias atuais para codificar e descodificar mensagem, visando atender necessidades do dia-a-dia. Tente entender tudo que est sendo explicado na aula. Estude com caneta e papel ao lado. Seja paciente e procure ter certeza de que voc entendeu o que (e por que) est fazendo.

Objetivos

Usar a Matemtica para resolver problemas do cotidiano. Usar a Teoria dos Nmeros para resolver problemas de tornar senhas ou mensagens seguras, de modo que sejam somente conhecidas pelos seus donos ou usurios credenciados.

131

1. Noes bsicas de Criptografia

Criptografia, palavra que vem do grego e krypts, "secreto", e grphein, "escrita", o estudo dos princpios, tcnicas e implementao de sistemas sigilosos pelos quais a informao pode ser transformada da sua forma original para outra ilegvel, de maneira que possa ser conhecida apenas por seu destinatrio (detentor da "chave secreta"), o que a torna difcil de ser lida por algum no autorizado, de modo que s o receptor da mensagem pode ler a informao com facilidade. A procura pelo uso da criptografia proporcional a necessidade de passar ou receber informaes consideradas sigilosas para um indivduo, grupo de pessoas, comrcio, organizaes civis ou militares etc. Este tipo de procedimento tem suas origens nos babilnios, egpcios e hindus. Pesquisadores encontraram inscries cuneiformes, datadas de 1500 a. C., contendo procedimentos criptografados para fazer cermica esmaltada. O historiador grego Herdoto, em seu A Histria, que um relato sobre a guerra entre gregos e persas, conta que, no ano de 499 a. C., Histiaeus, tirano de Miletos, raspou a cabea de um escravo e tatuou uma mensagem para ser usada na guerra contra a Prsia. Depois que o cabelo do escravo cresceu, Histiaeus enviou o mesmo para seu genro Aristgoras, em Miletos, que aps raspar a cabea do escravo encontrou a mensagem. Polybius, poltico, diplomata e historiador grego do perodo helenstico, no segundo sculo antes de Cristo, criou um sistema criptogrfico que substitua as letras de um texto por um par de nmeros. Olhando com nosso alfabeto atual, o procedimento de Polybius seria dispor as letras do alfabeto da esquerda para direita e de cima para baixo em um tabuleiro 5 por 5, de modo que as letras I e J so combinadas para ficar numa mesma posio. Cada letra era substituda por um par de dgitos, de maneira que o primeiro representava a linha em que a letra se encontrava e o segundo a coluna, veja Tabela I, a seguir.
Tabela I 1 A F L Q V 2 B G M R W 3 C H N S X 4 D IJ O T Y 5 E K P U Z

1 2 3 4 5

132

De acordo com o mtodo de Polybius, a mensagem O HOMEM VAI HOJE seria enviada como: 34 23 34 32 15 32 51 11 24 23 34 24 15.

Outro exemplo de mensagem criptografada, um criptograma, baseado na transformao de cada letra do texto em outra letra diferente, para produzir assim a escrita secreta. Pesquisadores descobriram um criptograma usado por Jlio Csar, imperador romano, que se usado com o nosso alfabeto atual, com 26 letras, substitua cada letra pela letra que estivesse, na ordem usual do alfabeto, trs letras depois, sendo que as trs ltimas X, Y e Z eram substitudas por A, B e C, respectivamente, veja na Tabela II, a seguir.
Tabela II
A D B E C F D G E H F I G J H K I L J M K N L O M P N Q O R P S Q T R U S V T W U X V Y W Z X A Y B Z C

Deste modo, a mensagem IHOLCQDWDO

FELIZ NATAL seria criptografada como:

EXEMPLO 1 Usando o mtodo de Polybius, faa o criptograma da mensagem seguinte:

ENVIE TROPAS
SOLUO

Usando a Tabela 1, temos: E N V I E T R O P A S 15 33 51 24 15 44 42 34 35 11 43


EXERCCIO 1 Usando o mtodo de Polybius, decifre a mensagem:

45 43 11 42 15 24 34 12 11 42 13 34

Associando a cada letra do alfabeto um nmero, de acordo com a Tabela III, a seguir, podemos construir um criptograma baseado na noo de congruncia.

Tabela III
A 0 B 1 C 2 D 3 E 4 F 5 G 6 H 7 I 8 J 9 K 10 L 11 M 12 N 13 O 14 P 15 Q 16 R 17 S 18 T 19 U 20 V 21 W 22 X 23 Y 24 Z 25

Seja Y o correspondente numrico de uma letra num texto, como na Tabela III acima, e seja X o equivalente numrico da letra correspondente no texto criptografado. No

133

sistema usado por Jlio Csar, teramos X Y + 3 (mod 26). No momento de decifrar uma mensagem, basta usar a congruncia Y X - 3 (mod 26).

EXEMPLO 2 Seja Y o correspondente numrico de uma letra num texto, como na Tabela III acima, e seja X o equivalente numrico da letra correspondente no texto criptografado, de tal modo que X Y +15 (mod 26). Use este criptograma para transformar a mensagem:

ENVIE REFOROS numa mensagem numrica. SOLUO Para criptografar a mensagem, transformando-a numa mensagem numrica, temos que saber qual ser o nmero que corresponde a cada letra do texto-mensagem dado. Assim, o nmero que corresponder letra E ser o nmero da Tabela II acima, representado pela incgnita X, satisfazendo: X E +15 (mod 26) = 4 +15 (mod 26) = 19 (mod 26). Ou seja X = 19. O nmero que corresponder letra N ser aquele da Tabela II acima, representado pela incgnita X, tal que: X N +15 (mod 26) = 13 + 15 (mod 26) = 28 (mod 26) = 2 (mod 26), ou seja X = 2. O nmero que corresponder letra V ser aquele da Tabela II acima, representado pela incgnita X, tal que X V +15 (mod 26) = 21 + 15 (mod 26) = 36 (mod 26) = 10 (mod 26), ou seja X = 10. O nmero que corresponder letra I ser aquele da Tabela II acima, representado pela incgnita X, tal que X I +15 (mod 26) = 8 + 15 (mod 26) = 23 (mod 26), ou seja X = 23. O nmero que corresponder letra R ser aquele da Tabela II acima, representado pela incgnita X, tal que X R +15 (mod 26) = 17 + 15 (mod 26) = 32 (mod 26) = 6 (mod 26), ou seja X = 6. O nmero que corresponder letra F ser aquele da Tabela II acima, representado pela incgnita X, tal que X F +15 (mod 26) = 5 + 15 (mod 26) = 20 (mod 26) , ou seja X = 20. O nmero que corresponder letra O ser aquele da Tabela II acima, representado pela incgnita X, tal que

134

X O +15 (mod 26) = 14 + 15 (mod 26) = 29 (mod 26) = 3 (mod 26), ou seja X = 3. O nmero que corresponder letra (usamos o mesmo que a letra C) ser aquele da Tabela II acima, representado pela incgnita X, tal que X C +15 (mod 26) =3 + 15 (mod 26) = 18 (mod 26), ou seja X = 18. O nmero que corresponder letra S ser aquele da Tabela II acima, representado pela incgnita X, tal que X S +15 (mod 26) = 18 + 15 (mod 26) = 33 (mod 26) = 7 (mod 26), ou seja X = 7. Portanto, a mensagem numrica ser: 192102319 6192031837.

EXERCCIO 2 Seja Y o correspondente numrico de uma letra num texto, como na Tabela III acima, e seja X o equivalente numrico da letra correspondente no texto criptografado, de tal modo que X Y +19 (mod 26). Use este criptograma para transformar a mensagem:

HOJE CHOVEU MUITO numa mensagem numrica.

Generalizando o que vimos acima, podemos construir criptogramas usando a congruncia X aY + b (mod 26) onde a, b so inteiros no negativos e menores do que ou iguais a 25, tendo a condio MDC (a, 26) = 1. A condio MDC (a, 26) = 1 para garantir que podemos encontrar o Y, quando estivermos no processo de descodificao da mensagem, ou seja, transformando-a num modo legvel usual. Vejamos como isso feito. O fato de que MDC (a, 26) = 1, implica que existem inteiros m e n tais que am + 26n = 1. Portanto, mdulo 26, a equao diofantina am + 26n = 1 d origem a congruncia am 1 (mod 26), o que significa dizer que a invertvel mdulo 26, para todo inteiro a {1, 2, 3, ...., 25}. Deste modo, se voc est lendo uma mensagem codificada segundo a congruncia X aY + b (mod 26), voc pode obter o valor de Y da forma seguinte: X aY + b (mod 26) X b aY (mod 26) aY X b (mod 26) Y a 1 (X b) (mod 26) .

135

Portanto, para trabalhar com a congruncia X aY + b (mod 26), temos 26 escolhas para o inteiro b, de 0 at 25, e (26) = 12 escolhas para o nmero inteiro a, onde a funo de Euler. Veja na Tabela IV, a seguir, os possveis valores de a e seus inversos.
Tabela IV
a a mdulo 26
-1

1 3 1 9

5 21

7 15

9 13

11 19

15 7

17 23

19 11

21 5

23 17

25 25

EXEMPLO 3 Transformar a mensagem-texto ESTAREI NA PONTE numa mensagem numrica usando a congruncia X 9Y + 15 (mod 26). SOLUO

Para criptografar a mensagem, transformando-a numa mensagem numrica, temos que saber qual ser o nmero que corresponde a cada letra do texto-mensagem dado. Como a letra E na Tabela II corresponde ao nmero 4, o nmero que vai corresponder letra E ser o nmero X tal que X 9.4 + 15 (mod 26) = 36 + 15 (mod 26) = 51 (mod 26) 25(mod 26). Portanto, a letra E ser representada pelo nmero 25. Como a letra S na Tabela II corresponde ao nmero 18, o nmero que vai corresponder letra S ser o nmero X tal que X 9.18+ 15 (mod 26) = 162 + 15 (mod 26) = 177 (mod 26) 21(mod 26). Portanto, a letra S ser representada pelo nmero 21. Como a letra T na Tabela II corresponde ao nmero 19, o nmero que vai corresponder letra T ser o nmero X tal que X 9.19 + 15 (mod 26) = 171 + 15 (mod 26) = 186 (mod 26) 4(mod 26). Portanto, a letra T ser representada pelo nmero 4. Como a letra A na Tabela II corresponde ao nmero 0, o nmero que vai corresponder letra A ser o nmero X tal que X 9.0 + 15 (mod 26) = 0 + 15 (mod 26) = 15 (mod 26) 15(mod 26). Portanto, a letra A ser representada pelo nmero 15. Como a letra R na Tabela II corresponde ao nmero 17, o nmero que vai corresponder letra R ser o nmero X tal que X 9.17 + 15 (mod 26) = 153 + 15 (mod 26) = 168 (mod 26) 22(mod 26). Portanto, a letra R ser representada pelo nmero 22.

136

Como a letra I na Tabela II corresponde ao nmero 8, o nmero que vai corresponder letra I ser o nmero X tal que X 9.8 + 15 (mod 26) = 72 + 15 (mod 26) = 87 (mod 26) 9(mod 26). Portanto, a letra I ser representada pelo nmero 9. Como a letra N na Tabela II corresponde ao nmero 13, o nmero que vai corresponder letra N ser o nmero X tal que X 9.13 + 15 (mod 26) = 117 + 15 (mod 26) = 132 (mod 26) 2(mod 26). Portanto, a letra N ser representada pelo nmero 2. Como a letra P na Tabela II corresponde ao nmero 15, o nmero que vai corresponder letra P ser o nmero X tal que X 9.15 + 15 (mod 26) = 135 + 15 (mod 26) = 150 (mod 26) 20(mod 26). Portanto, a letra T ser representada pelo nmero 20. Como a letra O na Tabela II corresponde ao nmero 14, o nmero que vai corresponder letra O ser o nmero X tal que X 9.14 + 15 (mod 26) = 136 + 15 (mod 26) = 151 (mod 26) 21(mod 26). Portanto, a letra O ser representada pelo nmero 21. Assim, a mensagem ESTAREI NA PONTE ser transformada na seguinte mensagem numrica: 25 21 4 15 2 25 9 2 15 20 21 24 25.

EXERCCIO 3 (a) Transformar a mensagem-texto FELIZ NATAL numa mensagem numrica usando a congruncia X 21Y + 7 (mod 26). (b) Verifique que sua resposta est correta, ou seja, confirme que a mensagem numrica encontrada no subitem (a) de fato FELIZ NATAL.

Um criptograma semelhante aos estudados acima oferece a possibilidade para uma pessoa decifr-lo sem grandes investimentos. Isto acontece porque a freqncia mdia com que cada letra (de uma dada lngua, no nosso caso o portugus) aparece em algum texto mais ou menos constante. De modo que, contando a freqncia de aparecimento de cada smbolo no texto criptografado, possvel descobrir a que letra ele corresponde. bem verdade que, existem maneiras de complicar bastante o processo de identificao de cada letra na mensagem codificada. Por exemplo, usar um processo que destri a estrutura da frase, subdividindo a mensagem em blocos de letras e embaralhando esses blocos. possvel fazer isso eliminando os espaos entre blocos de palavras, subdividindo a mensagem em blocos de duas letras, permutando os blocos, trocando o primeiro pelo ltimo, o segundo pelo penltimo etc. Mas, estes passos que dificultam a identificao dos dados por estranhos, concorrentes ou pessoas com ms intenes,

137

apresentam desvantagem nas aplicaes comerciais, que normalmente so feitas atravs de computadores. O fato que estes cuidados deixariam o processo extremamente lento. Em 1977, trs pesquisadores que trabalhavam no Massachussets Institute of Tecnology MIT, nos Estados Unidos, R. L. Rivest, A Shamir e L. Adleman, idealizaram um mtodo de codificar nmeros, que ficou conhecido como RSA, que simples de fazer, mas muito difcil de desfazer, adaptando-se perfeitamente para as transaes comercias, com segurana e rapidez. No prximo pargrafo, vamos ter uma noo de como esta brilhante idia pode ser colocada em prtica, com constantes aperfeioamentos.
2. CRIPTOGRAFIA RSA

Um problema importante para os nossos propsitos o seguinte:


Problema Dado um nmero natural K, K primo ou no?

Voc aprendeu, na disciplina de Teoria dos Nmeros, o conhecido Pequeno Teorema de Fermat: K um nmero primo aK a (mod K) , para todo inteiro a, com 1 a K 1. Podemos aplicar o Pequeno Teorema de Fermat para decidir se K primo ou no. Como faremos isto? Inicialmente, poderamos fazer as divises de K por um dos primos 2, 3, 5, 7, ....,. Se K for divisvel por um desses, ento K no primo. Caso contrrio, continuaria o processo de diviso pelos primos seguintes. Este mtodo pode ser longo e no talvez no consigamos uma resposta a curto ou em mdio prazo. Outra maneira de verificar se K primo, seria verificar se 2K 2 (mod K). Se no acontece isso, poderemos concluir, pelo Pequeno Teorema de Fermat, que o nmero K composto. Se, por outro lado, se tivermos 2K 2 (mod K), ento h uma chance de K ser primo. Neste caso, verificaramos se 3K 3 (mod K). Se isto no acontece, ento K composto. Caso contrrio, h uma chance de K ser primo. Continuando este processo para 5, 7, 11, 13, 17, .... , podemos tirar uma concluso: ou K composto ou K primo. Observe que, podemos escolher, aleatoriamente, um nmero inteiro a < K e verificar a congruncia aK a (mod K) . Se esta congruncia no ocorre, ento K no primo. Um computador pode fazer este teste em poucos minutos, se o nmero no for arbitrariamente grande. Agora, o computador, mesmo os mais velozes, no entanto, sabendo que K no primo, levaria anos para conhecer seus fatores primos. Ou seja, podemos em pouco tempo saber se o nmero K primo, mas, se ele no primo, levaramos muito tempo para conhecer seus fatores primos, p e q, de modo que K = p.q. Esta a idia da criptografia RSA.

138

Escolhe-se dois nmeros primos grandes, p e q, de preferncia com mais de cem dgitos, define-se K = p.q. Cada pessoa que utiliza o sistema escolhe um inteiro positivo s, tal que MDC(s, (K)) = 1, e dois inteiros t e u tal que st = 1 + u(K). Portanto, st 1 (mod (K)). O par (K, s) o par que tornado pblico para os usurios do sistema, mas o nmero t, a chave que decifra, fica secretamente guardada. O que vimos acima, s tratou de nmeros. Mas, muitas mensagens so mensagenstexto. Ento, para usar o sistema RSA, temos que estabelecer uma maneira de converter a mensagem-texto em uma sequncia de nmeros. No sistema RSA, cada letra convertida em um par de nmeros, de acordo com a Tabela IV, a seguir.
Tabela V A 00 N 13 B 01 O 14 C 02 P 15 D 03 Q 16 E 04 R 17 F 05 S 18 G 06 T 19 H 07 U 20 I 08 V 21 J 09 W 22 K 10 X 23 L 11 Y 24 M 12 Z 25

Para usar o sistema RSA, transformamos a mensagem-texto numa mensagem numrica, usando a Tabela V. Os blocos numricos Y, obtidos pela transformao da mensagemtexto em mensagem-numrica, usando a Tabela V, so codificados, como X, usando o nmero s e a congruncia X Ys (mod K), onde 0 X < K, e o texto criptografado enviado. Usando o Teorema de Fermat, na verso do Teorema de Euler, temos que Y(K) 1 (mod K). Portanto, podemos escrever: Xt (Ys)t (mod K) = Yst (mod K) = Y1 + u(K) (mod K) = Y.Y u(K) (mod K) Y (mod K) com 0 < Y < K. Podemos escolher o nmero s como sendo qualquer primo maior do que K = p.q, tal que 2s > K = p.q, e seria impossvel recuperar a mensagem-texto, Y, simplesmente calculando a raiz s-sima de Xt. Logo, o conhecimento do par (K, s) no leva uma pessoa a conhecer o par decifrador (t, K). Na verdade, para obter t, o inverso de s mdulo (K), temos primeiro que determinar (K) = (p.q) = (p 1).(q 1), que requer que o decifrador conhea a fatorao de K, que praticamente impossvel sem conhecer p e q.
EXEMPLO 4 (a) Usando o sistema RSA, para p = 61, q = 47, K = p.q = 61. 47 = 2867 e s = 17, codifique a mensagem: O ENCONTRO SER HOJE (b) Imagine que voc recebeu a seguinte mensagem 1404 1562 0425 2251 0408 2137 0078 1819 codificada pelo mtodo RSA, com os mesmos parmetros do subitem (a) e voc conhece o parmetro t = 2273. Decifre a mensagem recebida SOLUO

Temos que (K) = (61).(47) = 60.46 = 2760.

139

Agora, temos que encontrar t, que o inverso de s = 17 mdulo K = 2867. Para isso, temos que st 1 (mod (K)), ou seja 17t 1 (mod 2760), que o mesmo que a equao diofantina 17t 2760u = 1. Assim, usando o Algoritmo da Diviso, temos: -2760 = (-163).17 + 11; 17 = 1.11 + 6; 11 = 1.6 + 5; 6 = 1.5 + 1. Assim, 1 = 6 + 5(-1) = 6 + [11 + 6(-1)] (-1) = 11(-1) + 6.2 = 11.(-1) + [17 +11(-1)].2 , que mesmo que : 1 = 17.2 + 11(-3) = 17.2 + [-2760 +17.163](-3) = 17.2 + 2760.3 +17(-489) , que nos d: 1 = 17. (-487) + 2760.3 = 17. (-487) 2760 (-3) . Assim, mdulo 2760, temos 17.(-487) 1 (mod 2760). Mas, -487 2273 (mod 2760) . Portanto, 17. 2273 1 (mod 2760), e o inverso de s = 17 t = 2273. Agora, usando a Tabela V, transformamos a mensagem texto na mensagem numrica equivalente, agrupando os nmeros em blocos de 4: 1404 1502 1413 1917 1418 0417 0014 0904, Usamos a congruncia X Ys (mod K), que neste caso X Y17 (mod 2867), para codificar a mensagem numrica. Logo, basta fazer o clculo Y17 (mod 2867), para Y {1404, 1502, 1413, 1917, 1418, 0417, 0014, 0904}. Assim, temos: 140417 150217 141317 191717 141817 041417 001417 090417 1526 (mod 2867) 1562 (mod 2867); 425 (mod 2867); 2251 (mod 2867); 408 (mod 2867); 2137 (mod 2867); 780 (mod 2867); 1819 (mod 2867).

Portanto, a mensagem O ENCONTRO SER HOJE ser codificada como: 1526 1562 0425 2251 0408 2137 0078 1819 (b) Pelo exposto acima, para ler a mensagem dada, vamos usar a congruncia: Xt (Ys)t (mod K) = Yst (mod K) = Y1 + u(K) (mod K) = Y.Y u(K) (mod K) Y (mod K) com t = 2273 e 0 < Y < K = 2867, para decifrar a mensagem. No caso, X {1526, 1562, 0425, 2251, 0408, 2137, 0780, 1819}. Assim, temos: 15262273 1404 (mod 2867); 15622273 1562(mod 2867); 04252273 1413 (mod 2867); 22512273 1917 (mod 2867); 04082273 1418 (mod 2867); 21372273 0414 (mod 2867); 07802273 0014 (mod 2867); 18192273 0904 (mod 2867); E a mensagem numrica seria 1404 1502 1413 1917 1418 0417 0014 0904, que corresponde, exatamente, a mensagem do subitem (a). Na verdade, o que fizemos foi

140

mostrar que o mtodo realmente funciona, verificando como decifrar uma mensagem codificada que j sabamos qual era.

EXERCCIO 4 (a) Usando o sistema RSA, para p = 83, q = 97, K = p.q = 83. 97 = 8051 e s = 29, codifique a mensagem: O LIVRO CHEGOU. (b) Verifique que a sua resposta est correta, fazendo os clculos semelhantes s do subitem (b) do Exemplo 4, acima.

oportuno observar que, o fato de algum conhecer K e s, no o leva a encontrar t, pois para ter o valor de t, que o inverso de s mdulo (K), teria que determinar (K), que sabemos ser (K) = (p.q) = (p - 1).(q 1). Mas, para isso, precisaria conhecer a fatorao de K, o que praticamente impossvel sem conhecer os fatores p e q. Agora, se algum conhece K e (K), os fatores p e q podem ser conhecidos. De fato, usando a identidade: (p q)2 - (p + q)2 = -4pq e, tambm, o fato de que p + q = pq (p 1).(q 1) + 1 = pq - (K) + 1, e a igualdade (p - q) = [(p + q)2 4pq]1/2 = [(p + q)2 4K]1/2, teremos:
p= ( p + q)+( p q) 2 e q= ( p + q)( p q) 2

EXEMPLO 5 Determine os primos p e q, usados no sistema RSA dado por K = 4386607 e (K) = 4382136. SOLUO

Temos que K = p.q e (K) = 4382136. Pelos clculos feitos acima, temos que:
p + q = pq - (K) + 1 = 4386607 4382136 + 1 = 4472 e p q = [(p + q)2 4K]1/2 = [44722 4 x 4386607]1/2 = 1566.

Portanto, temos que: ( p + q ) + ( p q ) 4472 + 1566 = = 3019 p= 2 2


q= ( p + q ) ( p q ) 4472 1566 = = 1453 2 2

FIM DO EXEMPLO EXERCCIO 5 Usando o sistema RSA usado no Exemplo 5, a cima, e sabendo que s = 5, calcule o valor de t.

141

EXERCCIOS
1) Use o sistema de Polybius para codificar a mensagem ASA BRANCA. 2) Use o sistema de Julio Csar, imperador romano, para codificar a mensagem-texto: O SOL BRILHAR PARA TODOS. 3) Transformar a mensagem-texto ESTADO DO RIO GRANDE DO NORTE numa mensagem numrica usando a congruncia X 21Y + 7 (mod 26). (b) Verifique que sua resposta est correta, ou seja, confirme que a mensagem numrica encontrada no subitem (a) de fato ESTADO DO RIO GRANDE DO NORTE. 4)Determine dois nmeros primos p e q usados no sistema RSA, sabendo-se que K = 4386607 e (K) = 4382136. Se s = 5 determine t.

Resumo
Nesta aula estudamos alguns sistemas usados para criptografar mensagens-textos. Particularmente, vimos qual a idia do sistema RSA.

Referncias
COUTINHO, S. C. Nmeros Inteiros e Criptografia RSA. Rio de Janeiro, IMPA/SBM, 1997. COUTINHO, S. C. Criptografia. Rio de Janeiro, IMPA/SBM, Programa de Iniciao Cientfica da OBMEP. 2007. HEFEZ, Abramo. Elementos de aritmtica. Rio de Janeiro: Sociedade Brasileira de Matemtica, 2005. TATTERSALL, James J. Elementary Number Theory in Nine Chapters. Cambridge University Press. Cambridge. New York. 1999.

142

Você também pode gostar